You are on page 1of 137

www.baoshi2011.

com

第 1 讲 计算综合(一)
繁分数的运算,涉及分数与小数的定义新运算问题,综合性较强的计算问题.
1.繁分数的运算必须注意多级分数的处理,如下所示:

甚至可以简单地说:“先算短分数线的,后算长分数线的”.找到最长的分数线,将其上视为分子,
其下视为分母.
2.一般情况下进行分数的乘、除运算使用真分数或假分数,而不使用带分数.所以需将带分数化为
假分数.
3.某些时候将分数线视为除号,可使繁分数的运算更加直观.
4.对于定义新运算,我们只需按题中的定义进行运算即可.
5.本讲要求大家对分数运算有很好的掌握,可参阅《思维导引详解》五年级
[第 1 讲 循环小数与分数].

7 1 1
×4 +
1.计算:
18 2 6 ×2 7
1 3 5 8
13 − 3 ÷
3 4 16
7 1 23
+
【分析与解】原式=
4 6 × 2 = 12 × 23 = 4 17
7
1 8 4 8 128
13 − 12
3 3

2.计算:
5
【分析与解】 注意,作为被除数的这个繁分数的分子、分母均含有19 .于是,我们想到改变运算顺
9
5
序,如果分子与分母在 19 后的两个数字的运算结果一致,那么作为被除数的这个繁分数的值为 1;
9
如果不一致,也不会增加我们的计算量.所以我们决定改变作为被除数的繁分数的运算顺序.
而作为除数的繁分数,我们注意两个加数的分母相似,于是统一通分为 1995×0.5.
具体过程如下:
5 9
19 (+3 − 5.22)
9 10 1993 × 0.4 1.6
原式= ÷( + )
5 27 1995 × 0.5 1995
19 ( −6 + 5.22)
9 50

学而思奥数网 www.aoshu.cn Page 1 of 137


www.baoshi2011.com

5
19 − 1.32
9 1993 × 0.4 4 × 0.4 × 0.5
= ÷( + )
5
19 − 1.32 1995 × 0.4 1995 × 0.5
9
1993 + 2 0.4 0.4 1
=1 ÷ ( × ) =1 ÷ =1
1995 0.5 0.5 4

1
1−
1
3.计算: 1+
1
1−
1987
1
1− 1986 1987
【分析与解】原式= 1987 = 1 − =
1+ 3973 3973
1986

1 8
=
1 11
1+
4.计算:已知= 1 ,则 x 等于多少?
2+
1
x+
4
1 1 1 8x + 6 8
= = = =
1 1 4x + 1 12x + 7 11
1+ 1+ 1+
【分析与解】方法一: 1 4 8x + 6
2+ 2+
1 4x + 1
x+
4
交叉相乘有 88x+66=96x+56,x=1.25.
1 11 3
1+ = = 1+ 1 8 2
1 8 8 ,所以 2+ = = 2+ 1 3
方 法 二 :有 2+
1 x+
1 3 3 ;所以 x + = ,那么 x =
x+ 4 2
4
4
1.25.

5.求
4,43,443,..., 44...43
1 2 3 这 10 个数的和.
9个 4

【分析与解】方法一:
4+43+443 + ... + 44...43
123
9个 4

= 4 + (44 − 1) + (444 − 1) + ... + (44...4


{ − 1)
10个 4

学而思奥数网 www.aoshu.cn Page 2 of 137


www.baoshi2011.com

4
= 4 + 44 + 444 + ... + 44...4
{ −9 = × (9 + 99 + 999 + ... + 999...9)
12 3 −9
10个 4 9 10个 9

4
= × [(10 − 1) + (100 − 1) + (1000 − 1) + ... + (1000...0
14 2 43 − 1)] − 9
9 10个 0

4
= ×111.100 − 9=4938271591 .
9 14 92个143

方法二:先计算这 10 个数的个位数字和为 3 × 9+4=31 ;

再计算这 10 个数的十位数字和为 4×9=36,加上个位的进位的 3,为 36 + 3 = 3 9 ;

再计算这 10 个数的百位数字和为 4×8=32,加上十位的进位的 3,为 32 + 3 = 3 5 ;

再计算这 10 个数的千位数字和为 4×7=28,加上百位的进位的 3,为 28 + 3 = 31 ;

再计算这 10 个数的万位数字和为 4×6=24,加上千位的进位的 3,为 24 + 3 = 2 7 ;

再计算这 10 个数的十万位数字和为 4×5=20,加上万位的进位的 2,为 20 + 2 = 2 2 ;

再计算这 10 个数的百万位数字和为 4×4=16,加上十万位的进位的 2,为 16 + 2 = 1 8 ;

再计算这 10 个数的千万位数字和为 4×3=12,加上百万位的进位的 1,为 12 + 1 = 1 3 ;

再计算这 10 个数的亿位数字和为 4×2=8,加上千万位的进位的 1,为 8 + 1 = 9 ;

最后计算这 10 个数的十亿位数字和为 4×1=4,加上亿位上没有进位,即为 4 .

所以,这 10 个数的和为 4938271591.

6.如图 1-1,每一线段的端点上两数之和算作线段的长度,那么图中 6 条线段的长度之和是多少?

【分析与解】 因为每个端点均有三条线段通过,所以这 6 条线段的长度之和为:


1 1 7
3 × ( + + 0.6 + 0.875) = 1+0.75+1.8+2.625=6.175=6
3 4 40

学而思奥数网 www.aoshu.cn Page 3 of 137


www.baoshi2011.com

7. 我们规定,符号 “○” 表示选择两数中较大数的运算,例如: 3.5○2.9=2.9○3.5=3.5.符号


“ △ ” 表示选择两数中较小数的运算,例如: 3.5△2.9=2.9△3.5=2.9 . 请 计 算 :
23 155
(0.625V ) × ( d 0.4)
33 384
1 235
( d 0.3) + ( V2.25)
3 104
【分析与解】原式
155
0.625 ×
384 = 5 × 155 ÷ 2 7 = 25
1 8 384 12 256
+ 2.25
3

8 . 规 定 ( 3 ) =2×3×4 , ( 4 ) =3×4×5 , (5)=4×5×6 , (10)=9×10×11 , … . 如 果

1 1 1
− = × ,那么方框内应填的数是多少?
(16) (17) (17)

1 1 1 (17) 16 ×17 ×18 1


【分析与解】 =( − )÷ = −1= −1 = .
(16) (17) (17) (16) 15 ×16 × 17 5

1 1 1 1 1 1
9.从和式+ + + + + 中必须去掉哪两个分数,才能使得余下的分数之和等于 1?
2 4 6 8 10 12
1 1 1 1 1 1 1 1 1
【分析与解】 因为 + = ,所以 , , , 的和为 l,因此应去掉 与 .
6 12 4 2 4 6 12 8 10

10.如图 1-2 排列在一个圆圈上 10 个数按顺时针次序可以组成许多个整数部分是一位的循环小数,


例如 1.892915929.那么在所有这种数中。最大的一个是多少?

【 分 析 与 解 】 有整数部分尽可能大,十分位尽可能大,则有 92918……较大,于是最大的为
g g
9.29189 2915 .

学而思奥数网 www.aoshu.cn Page 4 of 137


www.baoshi2011.com

11.请你举一个例子,说明“两个真分数的和可以是一个真分数,而且这三个
分数的分母谁也不是谁的约数”.
1 1 4 1 1 1 1 1 1
【分析与解】 有 + = , + = , + =
6 10 15 10 15 6 35 14 10
评注:本题实质可以说是寻找孪生质数,为什么这么说呢?
1 1 c+a 1 1 c+a 1
注意到 + = ,当 a + c = b 时,有 + = = .
a ×b c×b a × b×c a ×b c×b a × b×c a ×c
当 a、b、c 两两互质时,显然满足题意.
显然当 a、b、c 为质数时一定满足,那么两个质数的和等于另一个质数,必定有一个质数为 2,不妨
设 a 为 2,那么有 2 + c = b ,显然 b、c 为一对孪生质数.

1 1 1
即可得出一般公式: + = ,c 与 c+2 均为质数即可.
2 × (c + 2) c × (c + 2) 2 × c

1 1 1
12.计算: (1 − × 1−
)() × ... × (1 − )
2× 2 3× 3 10 ×10
【分析与解】
(2 − 1) × (2 + 1) (3 − 1) × (3 + 1) (10 − 1) × (10 + 1)
原式= × × ... ×
2× 2 3× 3 10 × 10
1× 3 × 2 × 4 × 3 × 5 × 4 × 6 × 5 × 7 × 6 × 8 × 7 × 9 × 8 ×10 × 9 ×11
=
2 × 2 × 3 × 3 × 4 × 4 × ... ×10 ×10
1× 2 × 3 × 3 × 4 × 4 × 5 × 5 × ... × 9 × 9 ×10 × 11
=
2 × 2 × 3 × 3 × 4 × 4 × ... × 9 × 9 × 10 ×10
1× 2 ×10 ×11 11
= = .
2 × 2 × 10 × 10 20

11× 66 + 12 × 67 + 13 × 68 + 14 × 69 + 15 × 70
13.已知 a= × 100 .问 a 的整数部分是多少?
11× 65 + 12 × 66 + 13 × 67 + 14 × 68 + 15 × 69
【分析与解】
11× 66 + 12 × 67 + 13 × 68 + 14 × 69 + 15 × 70
a= × 100
11× 65 + 12 × 66 + 13 × 67 + 14 × 68 + 15 × 69
11× (65 + 1) + 12 × (66 + 1) + 13 × (67 + 1) + 14 × (68 + 1) + 15 ×()69 + 1
= ×100
11× 65 + 12 × 66 + 13 × 67 + 14 × 68 + 15 × 69
11 + 12 + 13 + 14 +15
=()1+ ×100
11× 65 + 12 × 66 +13 ×67 +14 ×68 +15 ×69
11 + 12 + 13 + 14 + 15
= 100 + ×100 .
11× 65+12 × 66 + 13 × 67 + 14 × 68 + 15 × 69
11 + 12 + 13 + 14 + 15 11 + 12 + 13 +14 +15 100
因为 ×100 < × 100 =
11× 65+12 × 66 + 13 × 67 + 14 × 68 + 15 × 69 ()11 + 12 + 13 +14+15 ×65 65

学而思奥数网 www.aoshu.cn Page 5 of 137


www.baoshi2011.com

100 35
所以 a < 100+ = 101 .
65 65
11 + 12 + 13 + 14 + 15 11 + 12 +13 +14 +15 100
同时 ×100 > × 100 =
11× 65 + 12 × 66 + 13 × 67 + 14 × 68 + 15 × 69 11 + 12 + 13 +14+15 ×69
() 69
100 31
所以 a> 100 + =101 .
69 69
31 35
综上有 101 <a< 101 .所以 a 的整数部分为 101.
69 65

1 3 5 7 99 1
14.问 × × × × ... × 与 相比,哪个更大,为什么?
2 4 6 8 100 10
1 3 5 7 99 2 4 6 8 100
【分析与解】方法一:令 × × × × ... × =A , × × × × ... × =B ,
2 4 6 8 100 3 5 7 9 101
1 3 5 7 99 2 4 6 8 100 1
有 A × B==× × × × ... × × × × × × ... × .
2 4 6 8 100 3 5 7 9 101 101
而 B 中分数对应的都比 A 中的分数大,则它们的乘积也是 B>A,
1 1 1 1 1 1 1
有 A×A<4×B(=) < = × ,所以有 A×A< × ,那么 A< .
101 100 10 10 10 10 10
1 3 5 7 99 1 1
即 × × × × ... × 与 相比, 更大.
2 4 6 8 100 10 10
1 3 5 7 97 99
方法二:设 A= × × × × ... × × ,
2 4 6 8 98 100
1 1 3 3 5 5 99 99
则 A = × × × × × × ... × ×
2

2 2 4 4 6 6 100 100
1× 3 × 3 × 5 × 5 × 7 × 7 × ... × 97 × 97 × 99 × 99 ×1
= ,
2 × 2 × 4 × 4 × 6 × 6 × 8 × ... × 96 × 98 × 98 × 100 × 100
1× 3 3 × 5 5 × 7 97 × 99 99 1 1
显然 、 、 、…、 、 都是小于 1 的,所以有 A2< ,于是 A< .
2× 2 4× 4 6× 6 98 × 98 100 100 10

15.下面是两个 1989 位整数相乘: 14 2 4


3 111...11×111...11
14 2 43 .问:乘积的各位数字之和是多少?
1989 个个
1 1989 1

【分析与解】在算式中乘以 9,再除以 9,则结果不变.因为 1


4 2 43 能被 9 整除,所以将一个 14 2 43 111...11 111...11
1989个1 1989个1

乘以 9,另一个除以 9,使原算式变成:

14 2 43 ×123456790......012345679
999......99 1 4 4 4 42 4 4 4 43
1989个共位数
9 1988

()1000......00 − 1 ×123456790......012345679
= 1 4 2 43 1 4 4 4 42 4 4 4 43
1989个共位数
0 1988

123456790......012345679 000......00 − 123456790......012345679


= 1 4 4 4 4 2 4 4 4 4 3 14 2 43 1 4 4 4 4 2 4 4 4 4 3
共位数个共位数
1988 1989 0 1988

123456790......012345679123456789876543209......987654320987654321
= 1 4 4 4 42 4 4 4 43 1 4 4 4 44 2 4 4 4 4 43
共位数共位数
1988 1980

学而思奥数网 www.aoshu.cn Page 6 of 137


www.baoshi2011.com

得到的结果中有 1980÷9=220 个 “ 123456790” 和 “ 987654320” 及一个 “ 12345678”和一个


“987654321”,所以各位数之和为:
1 + 2 + 3 + 4 + 5 + 6 + 7 + 9 × 220 + 9 +8 +7 +6 +5 +4 +3 +2 ×220
()()
1 + 2 + 3 + 4 + 5 + 6 + 7 + 8 + 9 +8 + 7 + 6 +5 +4 +3 +2 +1 =17901
+()()
评注:111111111÷9=12345679;
999...9
M× 1 2 3 的数字和为 9×k.(其中 M≤ 1 2 3 ).可以利用上面性质较快的获得结果.
999...9
k个 9 k个 9

第 2 讲 计算综合(二)
本讲主要是补充[计算综合(I)]未涉及和涉及不深的问题,但不包括多位数的运算.
1.n×(n+1)=[n×(n+1)×(n+2)-(n-1)×n×(n+1)]÷3;
2.从 1 开始连续 n 个自然数的平方和的计算公 a 式:
1
12 + 22 + 32 + L + n2 = × n × ( n + 1) × ( 2n + 1)
6
3.平方差公式:a2-b2=(a+b)(a-b).

1 1
,b = ,
1 1
2+ 2+
1 1
1. 已知 a= 3+ 3+ 试比较 a、b 的大小.
1 1
ggg+ ggg+
99 1
99 +
100
【分析与解】

学而思奥数网 www.aoshu.cn Page 7 of 137


www.baoshi2011.com

1 1
a= ,b = ,
1 1
2+ 2+
1 1
3+ 3+
1 1
ggg+ ggg+
1 1
98 + 98 +
A B
1 1 1
其中 A=99,B=99+ . 因为 A<B,所以 98+ >98+ ,
100 A B
1 1 1 1
97 + < 97 + ,96 + > 96 + ,
1 1 1 1
98 + 98 + 97 + 97 +
A B 1 1
98 + 98 +
A B
L
1 1
2+ > 2+ ,
1 1
3+ 3+
1 1
4+ 4+ 所以有 a < b.
1 1
ggg+ ggg+
1 1
98 + 98 +
A B

1 1
+
1 1
2+ 1+
1 1
3+ 1+
2.试求 1 1 的和?
4+ 3+
1 1
ggg+ 4+
2005 1
ggg+
2005
1
x= ,
1
3+
【分析与解】 记 1 则题目所要求的等式可写为:
4+
1
ggg+
2005
1 1 1 1 1 1+ x
+ , + = + = 1.
2 + x 1+ 1 而 2 + x 1+ 1 2+ x 2+ x
1+ x 1+ x
所以原式的和为 1.
评注:上面补充的两例中体现了递推和整体思想.

2. 试求 1+2+3+4+…4+100 的值?
【分析与解】 方法一:利用等差数列求和公式,(首项+末项)×项数÷2=(1+100)×100÷2=5050.
方法二:倒序相加,1+ 2+ 3+ 4+ 5+… 97+ 98+ 99+ 100
100+ 99+ 98+ 97+ 96+…4+ 3+ 2+ 1,
上下两个数相加都是 101,并且有 100 组,所以两倍原式的和为 101×100,那么原式的和为
10l×100 ÷2=5050.

学而思奥数网 www.aoshu.cn Page 8 of 137


www.baoshi2011.com

方法三:整数裂项(重点),
原式=(1×2+2×2+3×2+4×2+…+100×2)÷2

= [ 1× 2 + 2 × (3 − 1) + 3 × (4 − 2) + 4 × (5 − 3) + ggg+100 × (101 − 99)] ÷ 2

= (1× 2 + 2 × 3 − 1× 2 + 3 × 4 − 2 × 3 + 4 × 5 − 3 × 4 +ggg+100 ×101 − 99 × 100 ) ÷ 2

= 100 ×101 ÷ 2
=5050.

3. 试求 l×2+2×3+3×4+4×5+5×6+…+99×100.
【分析与解】方法一:整数裂项
原式=(1×2×3+2×3×3+3×4×3+4×5×3+5×6×3+…+99×100×3)÷3
=[1×2×3+2×3×(4-1)+3×4×(5-2)+4×5×(6-3)+5×6×(7-4)+…+99×100×(101-98)]÷3

(1× 2 × 3 + 2 × 3 × 4 − 1× 2 × 3 + 3 × 4 × 5 − 2 × 3 × 4 + 4 × 5 × 6 − 3 × 4 × 5 + 5 × 6 × 7 − 4 × 5 × 6 +ggg+99 ×100 × 101 −


98 × 99 ×100 ) ÷ 3
= 99 ×100 ×101 ÷ 3
= 33 ×101× 100
= 3333 ×100
= 333300.
n × (n + 1) × (2n + 1)
方程二:利用平方差公式 12+22+32+42+…+n2= n =
2
.
6
原式:12+l+22+2+32+3+42+4+52+5+…+992+99
=12+22+32+42+52+…+992+1+2+3+4+5+…+99
99 ×100 ×199 99 ×100
= +
6 2
=328350+4950
=333300.

5.计算下列式子的值:
0.1×0.3+0.2 × 0.4+0.3×0.5+0.4×0.6+…+9.7×9.9+9.8 × 10.0
【分析与解】这个题看上去是一个关于小数的问题,实际上我们可以先把它们变成整数,然后再进行
计算.即先计算 1×3+2 × 4+3×5+4 × 6+…+97 × 99+98×100。再除以 100.
方法一:再看每一个乘法算式中的两个数,都是差 2,于是我们容易想到裂项的方法.
0.1×0.3+0.2 × 0.4+0.3×0.5+0.4×0.6+…+9.7×9.9+9.8 × 10.0
=(1×3+2×4+3×5+4×6+…+97×99+98×100)÷100
=[(l×2+1)+(2×3+2)+(3×4+3)+(4×5+4)+…+(97×98+97)+(98×99+98)]÷100
=[(1×2+2×3+3×4+4×5+…+97×98+98×99)+(1+2+3+4+…+97+98)]÷100
1 1
=( ×98×99×100+ ×98×99)÷100
3 2
=3234+48.51
=3282.51
方法二:可以使用平方差公式进行计算.
0.1×0.3+O.2×0.4+0.3×0.5+0.4×0.6+…+9.7×9.9+9.8×10.0

学而思奥数网 www.aoshu.cn Page 9 of 137


www.baoshi2011.com

=(1×3+2×4+3×5+4×6+…+97×99+98×l00)÷100
=(12-1+22-1+32-1+42-1+52-1+…+992-1)÷100
=(11+22+32+42+52+…+992-99)÷100
1
=( ×99×100×199-99)÷100
6
=16.5×199-0.99
=16.5×200-16.5-0.99
=3282.51
评注: 首先,我们要清楚数与数之间是相通的,小数的计算与整数的计算是有联系的.下面简单介
绍一下整数裂项.
1×2+2×3+3×4+…+(n-1)×n
1
= ×[1×2×3+2×3×3+3×4×3+…+(n-1)×n×3]
3
1
= ×{1×2×3+2×3×(4-1)+3×4×(5-2)+…+(n-1)×n[n+1-(n-2)]}
3
1 1× 2 × 3 − 2 × 3 × 1 + 2 × 3 × 4 − 3 × 4 × 2 + 3 × 4 × 5 + ggg
= × 
3  −(n − 1) × n × (n − 2) + (n − 1) × n × (n + 1) 
1
= × (n − 1) × n × (n + 1)
3

6.计算下列式子的值:
1 1 1 1 1 1
24 × (+ +ggg+ )−( 2 + 2 + ggg 2 )
2×3 4×5 20 × 21 1 1 +2 2
1 + 2 +ggg+102
2

1 1 1 1 1 1
【分析与解】 虽然很容易看出 = − , = − ⋅⋅⋅⋅⋅⋅ 可是再仔细一看,并没有什么效果,
2×3 2 3 4×5 4 5
因为这不像分数裂项那样能消去很多项.我们再来看后面的式子,每一项的分母容易让我们想到公式
1
12+22+32+…+n2= ×n×(n+1)×(2n+1) , 于 是 我 们 又 有
6
1 6
= .
1 + 2 + 3 +ggg+ n
2 22 2
n × ( n + 1)(2n − 1)
减号前面括号里的式子有 10 项,减号后面括号里的式子也恰好有 10 项,是不是“一个对一个”呢?
1 1 1 1 1 1
24 × ( + +ggg )−( 2 + 2 +ggg+ 2 )
2×3 4×5 20 × 21 1 1 +2 2
1 + 2 +ggg+102
2

1 1 1 1 1 1
= 24 × ( + +ggg ) − 6× ( + + ggg+ )
2×3 4×5 20 × 21 1× 2 × 3 2 × 3 × 5 10 ×11× 12
1 1 1 1 1 1
= 24 × ( + +ggg ) − 24 × ( + + ggg+ )
2×3 4×5 20 × 21 2× 4×3 4× 6×5 20 × 22 × 21
 1 1 1 1 1 1 
= 24 × ( − )+( − ) +ggg+( − )
 2×3 2× 4×3 4×5 4× 6×5 20 × 21 20 × 22 × 21 
1 1 1
= 24 × ( + +ggg )
2× 4 4× 6 20 × 22

学而思奥数网 www.aoshu.cn Page 10 of 137


www.baoshi2011.com

1 1 1
=6×( + +ggg )
1× 2 2 × 3 10 × 11
1
= 6 × (1 − )
11
60
=
11

7.计算下列式子的值:

1 1 1 1 1 1 1 1 1 1 1 1 1 1
(1 + + + + +ggg+ ) 2 + ( + + + +ggg+ )2 + ( + + +ggg+ )2
2 3 4 5 198012 2 3 4 5 198012 3 4 5 198012
1 1 1 1 1 1 1 1 1 1 1 1
+( + +ggg+ ) 2 + ( + +ggg+ )2 +ggg+( )2 + (1 + + + + +ggg+ )
4 5 198012 5 6 198012 198012 2 3 4 5 198012
【分析与解】显然直接求解难度很大,我们试着看看是否存在递推的规律.
显然 12+1=2;

1 1 1
(1 + ) 2 + ( )2 + (1 + ) = 4;
2 2 2
1 1 1 1 1 1 1
(1 + + ) 2 + ( + )2 + ( )2 + (1 + + ) = 6;
2 3 2 3 3 2 3
1 1 1 1 1 1 1 1 1 1 1 1
(1 + + + ) 2 + ( + + )2 + ( + )2 + ( )2 + (1 + + + ) = 8;
2 3 4 2 3 4 3 4 4 2 3 4
所以原式=198012×2=396024.
习题
计算 17×18+18×19+19×20+…+29×30 的值.
提示:可有两种方法,整数裂项,利用 1 到 n 的平方和的公式.
答案:(29×30×31-16×17×18)÷3=29×10×31-16×17×6=7358.

第 3 讲 多位数的运算

多位数的运算,涉及利用 14 2 4
999L 9
3 =10k-1,提出公因数,递推等方法求解问题.
k个9

学而思奥数网 www.aoshu.cn Page 11 of 137


www.baoshi2011.com

999L 9
一、 14 2 4
3 =10k-1 的运用
k个9

在多位数运算中,我们往往运用 14 2 4
3 =10k-1 来转化问题; 999L 9
k个9

333L 3
如: 1
4 2 43 ×59049
2004 个3

我们把 1
333L 3
4 2 43 转化为 14 2 43 ÷3, 999L 9
2004 个3 2004 个9

4 2L433 ×59049= ( 999


333
于是原式为 1 14 2L439 ÷3 ) ×59049= 999
14 2L439 ×59049= ( 1000
14 2 L430 -
2004 个3 2004 个9 2004 个9 2004 个0

1000L 0
1)×19683=19683× 14 2 43 -19683
2004 个0

而对于多位数的减法,我们可以列个竖式来求解;
6 4 442004
7 个49 4 48 +1
1968299L 999999
6 4 4420047 个49 4 48
1968299L 999999 + 1
−19683
如: 6 4 441999
1968299L 980317
,于是为 1 4 44 2 4 4 43 .
7个49 4 48
1999 个 9
1968299L 980316 + 1
6 4 441999
7个49 4 48
1968299L 980317

简便计算多位数的减法,我们改写这个多位数.
333L 3
原式= 14 2 4
3 ×2×3×3× 14 2 43 333L 3
2004 个3 2008个3

333L 3
= 14 2 4
3 ×2×3× 14 2 43 999L 9
2004 个3 2008 个9

1999L 98 1000L 0
= 14 2 43 ×( 14 2 43 -1)
2003 个9 2008 个0

1999L 98 1000L 0 1999L 98


= 14 2 43 × 14 2 43 - 14 2 43
2003 个9 2008个0 2003 个9

642003
7个个48
9
6 442008
7 49 48
1999L 97 9999999L 99 + 1
− 1999
14 2L43 98
=
2003个9 1999L 979998000L 02
,于是为 1 44 2 4 43 14 2 43 .
6 44 7 4 48 64 7 48
2003个个9 2003 0
2003个个
9 2003 0
1999L 979998000L 01 + 1
1 44L2979998000
1999 4 43 14 2L 43
02
2003个个
9 2003 0

学而思奥数网 www.aoshu.cn Page 12 of 137


www.baoshi2011.com

111L 1
2.计算 1 2 3 - 14 2 4
3 =A×A,求 A.222L 2
2004个1 1002个2

【分析与解】 此题的显著特征是式子都含有 1 2 3 ,从而找出突破口.


111L 1
n个1

1 2L31 - 222
111 14 2L432 = 111
1 2L31 000
14 2L430 - 111
1 2L31
2004个1 1002个2 1002个1 1002个0 1002个1

111L 1 1000L 0
= 1 2 3 ×( 14 2 43 -1)
1002个1 1002个0

111L 1
= 1 2 3 ×( 14 2 4
3 ) 999L 9
1002个1 1002个9

111L 1
= 1 2 3 ×( 1 2 3 ×3×3)=A2
111L 1
1002个1 1002个1

所以,A= 14 2 4
3 . 333L 3
1002个3

666L 6 666L 6
3.计算 14 2 4
3 × 14 2 43 ×25 的乘积数字和是多少?
2004个6 2003个6

14 2 L430 − 1 来简便计算,但是不同于上式的是不易得出凑成
999L39 = 1000
【分析与解】我们还是利用 14 2 4
k个9 k个0

14 2L439 ,于是我们就创造条件使用:
999
k个9

14 2L436 × 666
666 14 2L 43 67 ×25=[ 2 ×( 99914 2L439 )]×[ 2 ×( 999
14 2L439 )+1]×25
2004个6 2003个6 3 2004 个9 3 2004 个9

2 1000L 0 − 1 )]×[ 2 ×( 1000 14 2 L430 )+1]×25


=[ ×( 14 2 43
3 2004个0 3 2004个0

1 1 1000L 0 1000L 0
= × ×[2× 14 2 43 -2]×[2×( 14 2 43 )+1]×25
3 3 2004个0 2004个0

25 1000L 0 1000L 0
= ×[4× 14 2 43 -2× 14 2 43 -2]
9 4008个0 2004个0

100 999 L39 - 50 × 999


14 2L439
= × 14 2 4
9 4008个9 9 2004 个9

111L 1
=100× 1 2 3 -50× 1 2 3
111L 1
4008个1 2004 个1

111L 100 − 555L 50


= 14 2 43 14 2 43 (求差过程详见评注)
4008 个个
1 2004 5

111L 3 14 2L 43
10555
= 14 2 4
50
2004个个
1 2004 5

111L 3 14 2L 43
10555
所以原式的乘积为 14 2 4
50
2004个个
1 2004 5

那么原式乘积的数字和为 1×2004+5×2004=12024.
111L 100 − 555L 50
评注:对于 14 2 43 14 2 43 的计算,我们再详细的说一说.
4008 个个
1 2004 5

学而思奥数网 www.aoshu.cn Page 13 of 137


www.baoshi2011.com

14 L
111 43 − 555
2 100 14 2L 43
50
4008个个
1 2004 5

111L 1000L30 + 111


= 1 2 3 14 2 4 1 2L3100 − 555
14 2L 43
50
2005 个个
1 2005 个个
0 2003 1 2004 5

111L 10999L 9 + 1 + 111


= 1 2 3 14 2 43 1 2L3100 − 555
14 2L 43
50
2004 个个
1 2005个个
9 2003 1 2004 5

111L 10444L 49 + 111


= 1 2 3 1 4 2 43 1 2L3101
2004 个个
1 2004 个 4 2003 1

111L 10555L 5
= 1 2 3 14 2 43
2004 个1 2004 个 5

4.计算 14 2 4
3 222L 2 × 222
14 2L432 的积?
1998 个个
2 1998 2

【分析与解】 我们先还是同上例来凑成 14 2 4
3 ; 999L 9
k个 9

14 2L432 × 222
222 14 2L432
1998 个个
2 1998 2

2  
= ×  999 L439  × 222
2L432
9  1998个9  141998
14 2
个2

2  
= × 1000 L 0 − 1  × 222
14 2L432
9  141998
2 43
个0  1998个2

1  
= ×  1000
 14 2 L430 − 1 × 444
9  14 2L434
1998个 0  1998 个 4

1  
= ×  444 14 2L430 − 444
 14 2L434 000
9  14 2L434 
1998 个个
4 1998 个 0 1998 4 

1
= × 444
14 2L4343555
14 2L4356 (求差过程详见评注)
9 1997 个 4 1997 个5

我们知道 14 2 4
444L 4
3 能被 9 整除,商为:049382716.
9个 4

又知 1997 个 4,9 个数一组,共 221 组,还剩下 8 个 4,则这样数字和为 8×4=32,加上后面的


3,则数字和为 35,于是再加上 2 个 5,数字和为 45,可以被 9 整除.

1 4L2 4355
444 4 3 能被 9 整除,商为 04938271595;
8个4

我们知道 14 2 4
555L 5
3 能被 9 整除,商为:061728395;
9个5

这样 9 个数一组,共 221 组,剩下的 1995 个 5 还剩下 6 个 5,而 6 个 5 和 1 个、6,数字和 36,


可以被 9 整除.

14 2L 43
555 56 能被 9 整除,商为 0617284.
6个5

于是,最终的商为:

学而思奥数网 www.aoshu.cn Page 14 of 137


www.baoshi2011.com

1 4 4 4 4L
49382716 049382716 049382716
24 1 4 4 4 4L
4 4 43 04938271595061728395 0617283950617284
24 4 4 43
220 个个
049382716 221 061728395

评注:对于 1
4 2 43 444L 4 000
14 2L430 - 444
14 2L434 计算,我们再详细的说一说.
1998 个 4 1998 个 0 1998个 4

14 2L434 000
444 14 2L430 - 444
14 2L434
1998个 4 1998个 0 1998个 4

444L 43
= 14 2 14 2L439 +1- 444
43999 14 2L434
1997 个个9
4 1998 1998 个 4

444L 43
= 14 2 14 2L435 +1
43555
1997 个个5
4 1998

444L 43
= 14 2 14 2L 43
43555 56 .
1997 个个5
4 1997

二、提出公因式
有时涉及乘除的多位数运算时,我们往往需提出公因式再进行运算,并且往往公因式也是和式或
者差式等.

5. 计 算 : (
19981998L 1998
1998+19981998+199819981998+… 1 4 44 2 4 4 4
3
1998个1998

19991999L 1999
)÷(1999+19991999+199919991999… 1 4 44 2 4 4 4
3 )×1999
1998 个1999

19981998L 1998
3 =1998× 10011001
【分析与解】 1 4 44 2 4 4 4 14424 L 4
1001
3
1998个1998 1998 个1001

原 式 = 1998 (
10011001L 1001
1+10001+100010001+… 1 4 4 2 4 4 3
1998 个1001

10011001L 1001
)÷[1999×(1+10001+100010001+… 1 4 4 2 4 4 3 )]×1999=1998÷1999×1999
1998 个1001

=1998.

6.试求 1993×123×999999 乘积的数字和为多少?


【 分 析 与 解 】 我们可以先求出 1993×123 的乘积,再计算与(1000000—1)的乘积,但是
1993×123 还是有点繁琐.
设 1993×123=M,则(1000×123=)123000<M<(2000×123=)246000,所以 M 为 6 位
数,并且末位不是 0;

令 M= abcdef

则 M×999999=M×(1000000-1)=1000000M-M

= abcdef 000000 - abcdef

= abcdef ( f − 1) 999999 +1- abcdef

= abcdef ( f − 1) ( 9 − a ) ( 9 − b ) ( 9 − c ) ( 9 − d ) ( 9 − e ) ( 9 − f ) +1

= abcdef ( f − 1) ( 9 − a ) ( 9 − b ) ( 9 − c ) ( 9 − d ) ( 9 − e ) ( 9 − f + 1)

学而思奥数网 www.aoshu.cn Page 15 of 137


www.baoshi2011.com

那么这个数的数字和为:a+b+c+d+e+(f-1)+(9-a)+(9-b)+(9-c)+(9-d)+(9-e)+(9-
f+1)=9×6=54.
所以原式的计算结果的数字和为 54.
999L 9
评注:M× 14 2 4
3 的数字和为 9×k.(其中 M 的位数为 x,且 x≤k).
k个 9

7.试求 9×99×9999×99999999×…× 14 2 4
999L 9 999L 9 999L 9
3 × 14 2 43 × 14 2 43 乘积的数字和为多少?
256个9 512个9 1024个9

【分析与解】 通过上题的计算,由上题评注:

设 9×99×9999×99999999×…× 14 2 4
999L 9 999L 9 999L 9
3 × 14 2 43 × 14 2 43 =M,
256个9 512个9 1024个9

999L 9
于是 M× 14 2 4
3 类似 的情况,于是,确定好 M 的位数即可;
1024个9

注意到 9×99×9999×99999999×…× 14 2 4
999L 9 999L 9
3 × 14 2 43 =M,
256个9 512个9

1000L 0 1000L 0 1000L 0


则 M<10×100×100013×100000000×…× 14 2 43 × 14 2 43 = 14 2 43
256 个 0 512个0 k个0

其中 k=1+2+4+8+16+…+512=1024-l=1023;
1000L 0
即 M< 14 2 43 ,即 M 最多为 1023 位数,所以满足
999L 9
的使用条件,那么 M 与 14 2 4
3 乘
1023个0 1024个9

积的数字和为 1024×9=10240—1024=9216.
原式的乘积数字和为 9216.
三、递推法的运用
有时候,对于多位数运算,我们甚至可以使用递推的方法来求解,也就是通常的找规律的方法.

8 . 我 们 定 义 完 全 平 方 数 A2=A×A , 即 一 个 数 乘 以 自 身 得 到 的 数 为 完 全 平 方 数 ; 已 知 :
1234567654321×49 是一个完全平方数,求它是谁的平方?
【分析与解】 我们不易直接求解,但是其数字有明显的规律,于是我们采用递推(找规律)的方法
来求解:
121=112;12321=1112;1234321=11112……

于是,我们归纳为 1234…n…4321=( 1 2 3 )2
111L 1
n个1

所 以 , 1234567654321 : 11111112 ; 则 ,
1234567654321×49=1111111 ×7 =7777777 .所以,题中原式乘积为 7777777 的平方.
2 2 2

111L 1
评注:以上归纳的公式 1234…n…4321=( 1 2 3 )2,只有在 n<10 时成立.
n个1

14 2L434888
9.① 444 14 2L4389 =A2,求 A 为多少?
2004 个 4 2003个8

② 求是否存在一个完全平方数,它的数字和为 2005?
【分析与解】 方法一:问题①直接求解有点难度,但是其数字有明显的规律,于是我们采用递推
(找规律)的方法来求解:

学而思奥数网 www.aoshu.cn Page 16 of 137


www.baoshi2011.com

14 2L434888
① 注意到有 444 14 2L4389 可以看成 444
14 2L434888
14 2L4389 ,其中 n=2004;
2004 个 4 2003个8 n个4 n- 1个8

寻找规律:当 n=1 时,有 49=7 ; 2

当 n=2 时,有 4489=672;


当 n=3 时,有 444889=6672;
…… ……

14 2L434888
14 2L4389 = 666
2
于是,类推有 444 14 2L436 7
2004 个 4 2003个8 2003个 6

方法二:下面给出严格计算:

14 2L434888
444 14 2L434 000
14 2L4389 = 444 14 2L430 + 888
14 2L438 +1;
2004 个 4 2003个8 2004个4 2004个0 2004 个8

444L34 000
则 14 2 4 14 2L430 + 888 1 2L31 ×(4× 1000
14 2L438 +1= 111 14 2 L430 +8)+1
2004个4 2004个0 2004 个8 2004个1 2004个0

1 2L31 ×[4×( 999


= 111 14 2L439 +1)+8]+1
2004个1 2004个9

1 2L31 ×[4×( 999


= 111 14 2L439 )+12]+1
2004个1 2004个9

1 2L31 )2×36+12× 111


=( 111 1 2L31 +1
2004个1 2004个1

1 2L31 )2×62+2×(6× 111


=( 111 1 2L31 )+1
2004个1 2004个1

14 2L 43
=( 666 67 )2
2003个6

14 2L434888
14 2L4389 = 666
2
② 由①知 444 14 2L 43
67 ,于是数字和为(4n+8n 一 8+9)=12n+1=2005;
n个4 n- 1个8 n- 1个6

14 2L434888
14 2L4389 = 666 14 2L434888
14 2L4389 .
2
于是,n=167,所以 444 14 2 67 ,所以存在,并且为 444
L 43
167个4 166个8 166个6 167个4 166个8

10.计算 14 2 4
666L 6
3 ×9× 14 2 43 的乘积是多少? 333L 3
2008个6 2008个3

【分析与解】采用递推的方法 6×9×3=162;
66×9×33=19602;
666×9×333=1996002;
…… ……
666L36 ×9× 333
于是,猜想 14 2 4 14 2L433 = 1999
14 2L43 14 2L430 2
96 000
n个6 n个3 n−1个 9 n- 1个0

14 2L436 ×9× 333


666 14 2L433 = 1999
14 2L43 14 2L430 2
96 000
2008个6 2008个3 2007个9 2007个0

评注:我们与题 l 对比,发现题 1 为 14 2 4
666L 6
3 ×9×3× 14 2 43 使用递推的方法就有障碍, 14 2 43 333L 3 999L 9
2008个6 2004个3 k个9

=10 —l 这种方法适用面要广泛一点.
k

练习 1.设 N= 14 2 4
666L 6
3 ×9× 14 2 43 ,则 N 的各位数字之和为多少? 777 L 7
2000个6 2007个7

练习 2.乘积 14 2 4
999L 9 999L 9
3 × 14 2 43 的积是多少?各位数字之和又是多少?
1999个9 1999个9

学而思奥数网 www.aoshu.cn Page 17 of 137


www.baoshi2011.com

111L 1 111L 1
练习 3.试求 1 2 3 × 1 2 3 的各位数字之和是多少?
2008个1 2008个1

第 4 讲 比例和百分数

成本、利润、价格等基本经济术语,以及它们之间的关系.各种已知数据或所求结果中包含比例与
百分数的应用题,有时恰当选取较小的量作为一个单位,司以实现整数化计算.

1.迎春农机厂计划生产一批插秧机,现已完成计划的 56%,如果再生产 5040 台,总产量就超过计


划产量的 16%.那么,原计划生产插秧机多少台?
【分析与解】 : 5040÷(1+16%-56%)=8400(台).

2.圆珠笔和铅笔的价格比是 4:3,20 支圆珠笔和 21 支铅笔共用 71.5 元.问圆珠笔的单价是每支


多少元?
【分析与解】 :设圆珠笔的价格为 4,那么铅笔的价格为 3,则 20 支圆珠笔和 21 支铅笔的价格为
20×4+21×3=143,则单位“1”的价格为 71.5÷143:0.5 元.
所以圆珠笔的单价是 O.5×4=2(元).

1
3.李大娘把养的鸡分别关在东、西两个院内.已知东院养鸡 40 只;现在把西院养鸡总数的 卖
4
1
给商店, 卖给加工厂,再把剩下的鸡与东院全部的鸡相加,其和恰好等于原来东、西两院养鸡总数的
3
50%.原来东、西两院一共养鸡多少只?

【分析与解】:方法一:设原来东西两院一共养鸡 x 只,那么西院养鸡 ( x − 40 ) 只.

 1 1 1
依题意:. ( x − 40 ) × 1 − −  + 40 = x ,解出 x = 280 .
 4 3 2

学而思奥数网 www.aoshu.cn Page 18 of 137


www.baoshi2011.com

即原来东、西两院一共养鸡 280 只.
1 1 1 1
方法二:50%即 ,东、西两院剩下的鸡等于东院的 加上西院的 ,即 20+ 西院原养鸡数.
2 2 2 2
1 1 5
有东院剩下 40 只鸡,西院剩下原 1 − − = 的鸡.
4 3 12
1 5 
所以有西院原养鸡(40—20)÷  −  =240 只,即原来东、西两院一共养鸡 40+240=280 只.
 2 12 

4.用一批纸装订一种练习本.如果已装订 120 本,剩下的纸是这批纸的 40%;如果装订了 185


本,则还剩下 1350 张纸.这批纸一共有多少张?
【分析与解】 方法一:装订 120 本,剩下 40%的纸,即用了 60%的纸.
那么装订 185 本,需用 185×(60%÷120)=92.5%的纸,即剩下 1-92.5%=7.5%的纸,为
1350 张.
所以这批纸共有 1350÷7.5%=18000 张.
方法二:120 本对应(1-40%=)60%的总量,那么总量为 120÷60%=200 本.
当装订了 185 本时,还剩下 200-185:15 本未装订,对应为 1350 张,所以每本需纸张:
1350÷15=90 张,那么 200 本需 200×90=18000 张.
即这批纸共有 18000 张.

5.有男女同学 325 人,新学年男生增加 25 人,女生减少 5%,总人数增加 16 人.那么现有男同


学多少人?
【分析与解】男生增加 25 人,女生减少 5%,而总人数增加了 16 人,说明女生减少了 25-16=9
人,那么女生原来有 9÷5%=180 人,则男生有 325-180=145 人.
增加 25 人后为 145+25=170 人,所以现有男同学 170 人.

6.有一堆糖果,其中奶糖占 45%,再放人 16 块水果糖后,奶糖就只占 25%那么,这堆糖果中有


奶糖多少块?
45 9 25 1
【分析与解】方法一:原来奶糖占 = ,后来占 = ,因此后来的糖果数是奶糖的 4 倍,
100 20 100 4
9
也比原来糖果多 16 粒,从而原来的糖果是 16+( 4 × − 1)=20 块.
20
9
其中奶糖有 20× =9 块.
20
方法二:原来奶糖与其他糖(包含水果糖)之比是 45%:(1-45%)=9:11,
设奶糖有 9 份,其他糖(包含水果糖)有 11 份.
现在奶糖与其他糖之比是 25%:(1-25%)=1:3=9:27,
奶糖的份数不变,其他糖的份数增加了 27-11=16 份,而其他糖也恰好增加了 16 块,所以,l 份
即 1 块.奶糖占 9 份,就是 9 块奶糖.

学而思奥数网 www.aoshu.cn Page 19 of 137


www.baoshi2011.com

7.甲乙两包糖的重量比是 4:l,如果从甲包取出 10 克放入乙包后,甲乙两包糖的重量比变为 7:


5.那么两包糖重量的总和是多少克?
【分析与解】两包糖数量的总数是

 4 7  13 2
10 ÷  −  = 10 ÷ = 46 克.
 4 +1 7 + 5  60 13

8.有若干堆围棋子,每堆棋子数一样多,且每堆中自子都占 28%.小明从某一堆中拿走一半棋
子,而且拿走的都是黑子,现在,在所有的棋子中,白子将占 32%.那么,共有棋子多少堆?
【分析与解】 方法一:设有 x 堆棋子,每堆有棋子“1”.根据拿走黑子白子总数不变.

 1 1
列方程得 x × 28 oo =  x −  ×32%,化简得 28 x =32( x - ),两边同除以 4,
 2 2
1
得 7 x =8( x - ),解得 x =4.
2
即共有棋子 4 堆.
方法二: 注意到所有棋子中的白子个数前后不变,所以设白子数为“1”.
那么有: .

18 17 25 1 25
黑子变化了 − = ,对应为 堆;所以 对应 l 堆.
7 8 56 2 28
18 25 25 25
而开始共有棋子 l+ = ,所以共有 ÷ = 4 堆.
7 7 7 28

9.幼儿园大班和中班共有 32 名男生,18 名女生.已知大班中男生数与女生数的比为 5:3,中


班中男生数与女生数的比为 2:1,那么大班有女生多少名?
【分析与解】设大班女生有 x 名,则中班女生有(18- x )名.根据男生数可列出
5 2
方程: x × +(18- x )× =32,解得 x =12.
3 1
所以大班有女生 12 名.

学而思奥数网 www.aoshu.cn Page 20 of 137


www.baoshi2011.com

10.某校四年级原有 2 个班,现在要重新编为 3 个班,将原一班的号与原二班的丢组成新一班,将


原一班的{与原二班的吉组成新二班,余下的 30 人组成新三班.如果新一班的人数比新二班的人数多
10%,那么原一班有多少人?
【分析与解】

7 5
有新三班的为原一、二班总人数的 1- = ,为 30 人.
12 12
5
所以原来两班总人数是:30÷ =72(人).
12
则新一班与新二班人数总和是 72-30=42(人).
现在再把新二班人数算作 1 份.

1 + 10 oo
新一班人数=42 × =22(人),新二班人数=42-22=20(人).
1 + 10 oo + 1

1 1
(原一班人数)-(原二班人数)=(22-20)÷  −  =2×12=24(人).
3 4
原一班人数=(72+24)÷2=48(人).

11.有两包糖,每包糖内装有奶糖、水果糖和巧克力糖.已知:①第一包糖的粒数是第二包糖的
2
;②在第一包糖中,奶糖占 25%,在第二包糖中,水果糖占 50%;③巧克力糖在第一包糖中所占
3
的百分比是在第二包糖中所占的百分比的两倍.当两包糖合在一起时,巧克力糖占 28%,那么水果糖
所占百分比等于多少?
【分析与解】表述 1:设第一包有 2 a 粒糖,则第二包有 3 a 粒糖,设第二包有 3 b 粒巧克力糖,
则第一包有 4 b 粒巧克力糖.
4b + 3b b 5
= 28%,所以 = ×28%=20%.
2a + 3a a 7
4b
于是第一包中,巧克力糖占 =40%,水果糖占 1-40%-25%=35%.
2a
2a × 35 oo + 3a × 50 oo
在两包糖总粒数中,水果糖占 = 44%.
2a + 3a
表述 2: 设第一包糖总数为“2”,那么第二包糖总数为“ 3”,并设第一包糖含有巧克力糖 2c,第二包
糖含有巧克力糖 c.

学而思奥数网 www.aoshu.cn Page 21 of 137


www.baoshi2011.com

那么有 2×2c+3×c=28%×(2+3),有 7c=140%,所以 c=20%,那么有如下所示的每种糖所


占的百分数.

所以水果糖占总数的(35%×2+50%×3)÷(2+3)=44%.

12.某次数学竞赛设一、二、三等奖.已知:①甲、乙两校获一等奖的人数相等:⑦甲校获一等奖的
人数占该校获奖总人数的百分数与乙校相应的百分数的比为 5:6;③甲、乙两校获二等奖的人数总和占
两校获奖人数总和的 20%;④甲校获三等奖的人数占该校获奖人数的 50%;⑤甲校获二等奖的人数
是乙校获二等奖人数的 4.5 倍.
那么,乙校获一等奖的人数占该校获奖总人数的百分数等于多少?
【分析与解】 表述 1:不妨设甲校有 60 人获奖,由①、②,乙校有 50 人获奖.
由③知两校获二等奖的共有(60+50)×20%=22 人;
由⑤知甲校获二等奖的有 22÷(4.5+1)×4.5=18 人;
由④知甲校获一等奖的有 60-60×50%-18=12 人,
从而所求百分数等于 12÷50×100%=24%.
表述 2:

(这有一个“5”)

1.2÷5×100%=24%,即乙校获一等奖的人数占该校获奖总人数的 24%.

13.①某校毕业生共有 9 个班,每班人数相等.②已知一班的男生人数比二、三班两个班的女生
总数多 1;③四、五、六班三个班的女生总数比七、八、九班三个班的男生总数多 1.那么该校毕业生中
男、女生人数比是多少?
【分析与解】表述 1: 由②知,一、二、三班的男生总数比二、三班总人数多 1.
③ 知,四至九班的男生总数比七、八、九班总人数少 1.
因此,一至九班的男生总数是二、三、七、八、九共五个班的人数,则女生总数

学而思奥数网 www.aoshu.cn Page 22 of 137


www.baoshi2011.com

等于四个班的人数.
所以,男、女生之比是 5:4.
表述 2: .

有“一、二、三班男生”加上“四、五、六、七、八、九班男生”即为一至九班全体男生数,恰为“二、三班总
人数”加上“四、五、六班总人数”,即为五个班总人数,则女生总数等于四个班的人数.
所以,男、女生之比是 5:4.

14.某商品按原定价出售,每件利润为成本的 25%;后来按原定价的 90%出售,结果每天售出


的件数比降价前增加了 1.5 倍.问后来每天经营这种商品的总利润比降价前增加了百分之几?
【分析与解】设这种商品的成本为“1”,共卖出商品“1”,则利润为 25%,总利润为 0.25,定价为
1.25.
那么按原定价的 90%出售,即以 1.25× 90%=1.125 的价格出售,现在销售的件数比原来增加了
1.5 倍 , 利 润 为 0.125×(1.5+1)=O.3125 , 而 原 来 的 总 利 润 为 O.25 , 现 在 增 加 了 0.3125 一
O.25=0.0625,0.0625÷0.25:25%.
所以,后来每天经营这种商品的总利润比降价前增加了 25%.


卖出价买入价
15.赢利百分数= ×100 oo
买入价
某电子产品去年按定价的 80%出售,能获得 20%的赢利;由于今年买入价降低,按同样定价的

今年买入价
75%出售,却能获得 25%的赢利.那么 是多少?
去年买入价
【分析与解】 根据题中给出的公式知:
赢利百分数×买入价=卖出价一买入价
则买入价×(赢利百分数+1)=卖出价,

卖出价
那么买入价=
赢利百分数+1

÷ (25
今年买入价 今年卖出价1+ o
o) 定价 × 75 oo ÷ 125 oo 9
= = =
÷ (25
去年买入价 去年卖入价1+ o
o) 定价 × 80 oo ÷ 120 oo 10

学而思奥数网 www.aoshu.cn Page 23 of 137


www.baoshi2011.com

第 5 讲 比和比例
两个数相除又叫做两个数的比.
一、比和比例的性质
性质 1:若 a: b=c:d,则(a + c):(b + d)= a:b=c:d;
性质 2:若 a: b=c:d,则(a - c):(b - d)= a:b=c:d;
性质 3:若 a: b=c:d,则(a +x c):(b +x d)=a:b=c:d;(x 为常数)
性质 4:若 a: b=c:d,则 a×d = b×c;(即外项积等于内项积)
正比例:如果 a÷b=k(k 为常数),则称 a、b 成正比;
反比例:如果 a×b=k(k 为常数),则称 a、b 成反比.
二、比和比例在行程问题中的体现

路程
在行程问题中,因为有速度= ,所以:
时间
当一组物体行走速度相等,那么行走的路程比等于对应时间的反比;
当一组物体行走路程相等,那么行走的速度比等于对应时间的反比;
当一组物体行走时间相等,那么行走的速度比等于对应路程的正比.

1.A 和 B 两个数的比是 8:5,每一数都减少 34 后,A 是 B 的 2 倍,试求这两个数.


【分析与解】
方 法 一 : 设 A 为 8x,则 B 为 5x,于是有(8x-34):(5x-34)=2:1,x=17,所以 A 为 136,B
为 85.
方法二: 因为减少的数相同,所以前后 A 、B 的差不变,开始时差占 3 份,后来差占 1 份且与 B
一 样多 ,也 就是 说减 少的 34, 占开 始的 3-1=2 份 ,所 以开 始的 1 份 为 34÷2=17, 所以 A 为
17×8=136,B 为 17×5=85.

5
2.近年来火车大提速,1427 次火车自北京西站开往安庆西站,行驶至全程的 再向前 56 千米
11
处所用时间比提速前减少了 60 分钟,而到达安庆西站比提速前早了 2 小时.问北京西站、安庆西

学而思奥数网 www.aoshu.cn Page 24 of 137


www.baoshi2011.com

站两地相距多少千米?
【分析与解】设北京西站、安庆西站相距多少千米?
5 5 10
( x+56):x=60:120,即( x+56):x=1:2,即 x= x+112,解得 x=1232.
11 11 11
即北京西站、安庆西站两地相距 1232 千米,

3.两座房屋 A 和 B 各被分成两个单元.若干只猫和狗住在其中.已知:A 房第一单元内猫的比率(即


住在该单元内猫的数目与住在该单元内猫狗总数之比)大于 B 房第一单元内猫的比率;并且 A 房第二单
元内猫的比率也大于 B 房第二单元内猫的比率.试问是否整座房屋 A 内猫的比率必定大于整座房屋 B
内猫的比率?
【分析与解】 如下表给出的反例指出:对所提出问题的回答应该是否定的.表中具体写出了各个单
元及整座房屋中的宠物情况和猫占宠物总数的比率.

4.家禽场里鸡、鸭、鹅三种家禽中公篱与母篱数量之比是 2:3,已知鸡、鸭、鹅数量之比是 8:7:5,


公鸡、母鸡数量之比是 1:3,公鸭、母鸭数量之比是 3:4.试求公鹅、母鹅的数量比.
【分析与解】 公鸡占家禽场家禽总数的
2 1 1 2 46 8 1 1
= 15 : (3 ×
× 5 + 4 × × 4) = 45 : 46 : (3 × × 5 + 4 × × 4) = 46 : 47. × = ,母鸡
3 3 3 3 45 8 + 7 + 5 1 + 3 10
3
占总数的 ;
10
8 3 3 4
公鸭占总数的 × = ,母鸭占总数的 ;
8 + 7 + 5 3 + 4 20 20
2 1 3 3 2 3 4 2
公鹅占总数的 −() + = ,母鹅占总数的 −() + = ,公鹅、母鹅数量之比
3 + 2 10 20 20 3 + 2 10 20 20
3 2
为 : :3:2.
20 20

5.在古巴比伦的金字塔旁,其朝西下降的阶梯旁 6m 的地方树立有 1 根走子,其影子的前端正好到


达阶梯的第 3 阶(箭头).另外,此时树立 l 根长 70cm 自杆子,其影子的长度为 175cm,设阶梯各阶
的高度与深度都是 50cm,求柱子的高度为多少?

【分析与解】70cm 的杆子产生影子的长度为 175cm;


所以影子的长度与杆子的长度比为:175:70=2.5 倍.

学而思奥数网 www.aoshu.cn Page 25 of 137


www.baoshi2011.com

于是,影子的长度为 6+1.5+1.5×2.5=11.25,所以杆子的长度为 11.25÷2.5=4.5m.

6.已知三种混合物由三种成分 A、B、C 组成,第一种仅含成分 A 和 B,重量比为 3:5;第二种只含成


分 B 和 C,重量比为 I:2;第三种只含成分 A 和 C,重量之比为 2:3.以什么比例取这些混合物,
才能使所得的混合物中 A,B 和 C,这三种成分的重量比为 3:5:2 ?
【分析与解】注意到第一种混合物种 A、B 重量比与最终混合物的 A、B 重量比相同,均为 3:5.所以,
先将第二种、第三种混合物的 A、B 重量比调整到 3:5,再将第二种、第三种混合物中 A、B 与第一种混
合物中 A、B 视为单一物质.
第二种混合物不含 A,第三种混合物不含 B,所以 1.5 倍第三种混合物含 A 为 3,5 倍第二种混合
物含 B 为 5,即第二种、第三种混合物的重量比为 5:1.5.
于是此时含有 C 为 5×2+1.5×3=14.5,在最终混合物中 C 的含量为 3A/5B 含量的 2 倍.有
14.5÷2-1=6.25,所以含有第一种混合物 6.25.
即第一、二、三这三种混合物的比例为 6.25:5:1.5=25:20:6.

7.现有男、女职工共 1100 人,其中全体男工和全体女工可用同样天数完成同样的工作;若将男工人


数和女工人数对调一下,则全体男 25 天完成的工作,全体女工需 36 天才能完成,问:男、女工各多
少人?
【分析与解】 直接设出男、女工人数,然后在通过方程求解,过程会比较繁琐.
设开始男工为“1”,此时女工为“k”,有 1 名男工相当 k 名女工.男工、女工人数对调以后,则男
工为“k”,相当于女工“k2”,女工为“I”.
6
有 k2:1=36:25,所以 k= .
5
1
于是,开始有男工数为 ×1100=500 人,女工 600 人.
1+ k

8.有甲乙两个钟,甲每天比标准时间慢 5 分钟,而乙每天比标准时间快 5 分钟,在 3 月 15 日的零点


零分的时候两钟正好对准.若已知在某一时刻,乙钟和甲钟时针与分针都分别重合,且在从 3 月 15
日开始到这个时候,乙钟时针与分针重合的次数比甲钟多 10 次,那么这个时候的标准时间是多少?
5
【分析与解】 标准的时钟每隔 65 分钟重合一次.
11
假设经历了 x 分钟.
5 24 × 60 24 × 60 − 5
于是,甲钟每隔 65 × 分钟重合一次,甲钟重合了 ×x 次;
11 24 × 60 − 5 24 × 60
24 × 60 + 5
同理,乙钟重合了 ×x 次; 于是,需要乙钟比甲钟多重合
24 × 60
24 × 60 + 5 24 × 60 − 5 10
×x- ×x= ×x=10;
24 × 60 24 × 60 24 × 60

学而思奥数网 www.aoshu.cn Page 26 of 137


www.baoshi2011.com

所以,x=24×60;
5
5 24 × 60 × 65
所以要经历 24×60×65 分钟,则为 11 = 65 5 天.
11
24 × 60 11
5 10 10 6
于是为 65 天 ( × 24 =)10( ) 小时 ( × 60 =)54 分钟.
11 11 11 11

9.一队和二队两个施工队的人数之比为 3:4,每人工作效率之比为 5:4,两队同时分别接受两项


2 1
工作量与条件完全相同的工程,结果二队比一队早完工 9 天.后来,由一队工人 与二队工人 组成
3 3
新一队,其余的工人组成新二队.两支新队又同时分别接受两项工作量与条件完全相同的工程,结果
新二队比新一队早完工 6 天.试求前后两次工程的工作量之比?
【分析与解】 一队与二队的工作效率之比为:(3×5):(4×4)=15:16.
1
一队干前一个工程需 9÷ =144 天.
16
新一队与新二队的工作效率之比为:
2 1 1 2
(3 × × 5 + 4 × × 4) : (3 × × 5 + 4 × × 4) = 46 : 47.
3 3 3 3
1
新一队干后一个工程需 6÷ =282 天.
47
一队与新一队的工作效率之比为
2 1
15 : (3 × × 5 + 4 × × 4) = 45 : 46
3 3
46
所以一队干后一个工程需 282× 天.
45
46
前后两次工程的工作量之比是 144:(282× )=(144×45):(282×46)=540:1081.
45

第 6 讲 工程问题

多人完成工作、水管的进水与排水等类型的应用题.解题时要经常进行工作时间与工作效率之间的转
化.

学而思奥数网 www.aoshu.cn Page 27 of 137


www.baoshi2011.com

1.甲、乙两人共同加工一批零件,8 小时司以完成任务.如果甲单独加工,便需要 12 小时完成.


2
现在甲、乙两人共同生产了 2 小时后,甲被调出做其他工作,由乙继续生产了 420 个零件才完成任
5
务.问乙一共加工零件多少个?

1 1 1
【分析与解】乙单独加工,每小时加工 - = .
8 12 24
2 1 1 84 84
甲调出后,剩下工作乙需做(8—2 )×( ÷ )= (小时),所以乙每小时加工零件 420÷
5 8 24 5 5
2 2
=25 个,则 2 小时加工 2 ×25=60(个),因此乙一共加工零件 60+420=480(个).
5 5

2.某工程先由甲单独做 63 天,再由乙单独做 28 天即可完成.如果由甲、乙两人合作,需 48 天完


成.现在甲先单独做 42 天,然后再由乙来单独完成,那么还需做多少天?
【分析与解】 由右表知,甲单独工作 15 天相当于乙单独工作 20
天,也就是甲单独工作 3 天相当于乙单独工作 4 天.
所以,甲单独工作 63 天,相当于乙单独工作 63÷3×4=84 天,
即乙单独工作 84+28=112 天即可完成这项工程.
现在甲先单独做 42 天,相当于乙单独工作 42÷3×4=56 天,即乙还需单独工作 112—56=56 天
即可完成这项工程.

3.有一条公路,甲队独修需 10 天,乙队独修需 12 天,丙队独修需 15 天.现在让 3 个队合修,


但中间甲队撤出去到另外工地,结果用了 6 天才把这条公路修完.当甲队撤出后,乙、丙两队又共同合
修了多少天才完成?

1 1 1 1 1
【分析与解】 甲、乙、丙三个队合修的工作效率为 + + = ,那么它们 6 天完成的工程量为
10 12 15 4 4
3
×6= ,而实际上因为中途撤出甲队 6 天完成了的工程量为 1.
2
3 1 1 1 1
所以 -1= 是因为甲队的中途撤出造成的,甲队需 ÷ =5(天)才能完成 的工程量,所以
2 2 2 10 2
甲队在 6 天内撤出了 5 天.
所以,当甲队撤出后,乙、丙两队又共同合修了 5 天才完成.

4.一件工程,甲队独做 12 天可以完成,甲队做 3 天后乙队做 2 天恰好完成一半.现在甲、乙两队


合做若干天后,由乙队单独完成,做完后发现两段所用时间相等,则共用了多少天?

【分析与解】 甲队做 6 天完成一半,甲队做 3 天乙队做 2 天也完成一半。所以甲队做 3 天相当于乙队

学而思奥数网 www.aoshu.cn Page 28 of 137


www.baoshi2011.com

做 2 天.
2 2
即甲的工作效率是乙的 ,从而乙单独做 12× =8(天)完成,所以两段所用时间相等,每段时间
3 3
应是:
2
8÷(1+l+ )=3(天),因此共用 3×2=6(天).
3

5.抄一份书稿,甲每天的工作效率等于乙、丙二人每天的工作效率的和;丙的工作效率相当甲、乙
1
每天工作效率和的 .如果 3 人合抄只需 8 天就完成了,那么乙一人单独抄需要多少天才能完成?
5

1
【分析与解】已知甲、乙、丙合抄一天完成书稿的 ,又已知甲每天抄写量等于乙、丙两人每天抄写量之
8
1 1
和,因此甲两天抄写书稿的 ,即甲每天抄写书稿的 ;
8 16
1 1
由于丙抄写 5 天相当于甲乙合抄一天,从而丙 6 天抄写书稿的 ,即丙每天抄写书稿的 ;于是
8 48
1 1 1 1
可知乙每天抄写书稿的 - - = .
8 16 48 24
1
所以乙一人单独抄写需要 1÷ =24 天才能完成.
24

6.游泳池有甲、乙、丙三个注水管.如果单开甲管需要 20 小时注满水池;甲、乙两管合开需要 8 小
时注满水池;乙、丙两管合开需要 6 小时注满水池.那么,单开丙管需要多少小时注满水池?

1 1 3
【分析与解】 乙管每小时注满水池的 - = ,
8 20 40
1 3 11
丙管每小时注满水池的 - = .
6 40 120
11 120 10
因此,单开丙管需要 1÷ = =10 (小时).
120 11 11

7.一件工程,甲、乙两人合作 8 天可以完成,乙、丙两人合作 6 天可以完成,丙、丁两人合作 12 天


可以完成.那么甲、丁两人合作多少天可以完成?

1 1 1
【分析与解】 甲、乙,乙、丙,丙、丁合作的工作效率依次是 、 、 .
8 6 12
对于工作效率有(甲,乙)+(丙,丁)-(乙,丙)=(甲,丁).
1 1 1 1 1
即 + - = ,所以甲、丁合作的工作效率为 .
8 12 6 24 24
所以,甲、丁两人合作 24 天可以完成这件工程.

学而思奥数网 www.aoshu.cn Page 29 of 137


www.baoshi2011.com

8.一项工作,甲、乙两人合做 8 天完成,乙、丙两人合做 9 天完成,丙、甲两人合做 18 天完成.那


么丙一个人来做,完成这项工作需要多少天?

【分析与解】 方法一:对于工作效率有:
1 1 1 13
(甲,乙)+(乙,丙)-(丙,甲)=2 乙,即 + - = 为两倍乙的工作效率,所以乙的工作效
8 9 18 72
21
率为 .
144
1 13 1
而对于工作效率有,(乙,丙)-乙=丙,那么丙的工作效率为 - =
9 144 48
1
那么丙一个人来做,完成这项工作需 1÷ =48 天.
48
1 1 1 21
方法二:2(甲,乙,丙)=(甲+乙)+(乙、丙)+(甲、丙)= + + = ,所以(甲,乙,丙)=
8 9 18 72
21 21 21
÷2= ,即甲、乙、丙 3 人合作的工作效率为 .
72 144 144
21 1 1
那么丙单独工作的工作效率为 - = ,那么丙一个人来做,完成这项工作需 48 天.
144 8 48

9.某工程如果由第 1、2、3 小队合干需要 12 天才能完成;如果由第 1、3、5 小队合干需要 7 天才能


完成;如果由第 2、4、5 小队合干需要 8 天才能完成;如果由第 1、3、4 小队合干需要 42 天才能完成.
那么这 5 个小队一起合干需要多少天才能完成这项工程?

【分析与解】 由已知条件可得,

对于工作效率有:
(1、2、3)+(1、3、5)+2(2、4、5)+(1、3、4)=3(1、2、3、4、5).
所以 5 个小队一起合作时的工作效率为:
1 1 1 1 1
( + +2× + )÷3=
12 7 8 42 6
所以 5 个小队合作需要 6 天完成这项工程.
评注:这类需综合和差倍等知识的问题在工程问题中还是很常见的.

10.一个水箱,用甲、乙、丙三个水管往里注水.若只开甲、丙两管,甲管注入 18 吨水时,水箱已
满;若只开乙、丙两管,乙管注入 27 吨水时,水箱才满.又知,乙管每分钟注水量是甲管每分钟注水

学而思奥数网 www.aoshu.cn Page 30 of 137


www.baoshi2011.com

量的 2 倍.则该水箱最多可容纳多少吨水?

【分析与解】 设甲管注入 18 吨水所需的时间为“1”,而乙管每分钟注水量是甲管每分钟注水量的 2 倍,


那么乙管注入 18 吨的水所需时间为“O.5”,所以乙管注入 27 吨水所需的时间为 27÷18×0.5=0.75.
以下采用两种方法:
方法一:设丙在单位时间内注入的水为“1”,那么有:

因此 18+“1”=27+“O.75”,则“0.25”=9 吨,所以“1”
=36 吨,即丙在单位时间内灌入 36 吨的水.
所以水箱最多可容纳 18+36=54 吨的水.
方法二:也就是说甲、丙合用的工作效率是乙、丙合用工作
3
效率的 .
4
3
再设甲单独灌水的工作效率为“1”,那么乙单独灌水的工作效率为“2”,有 1+丙= (2+丙);所以
4
丙的工作效率为“2”,即丙的工作效率等于乙的工作效率,那么在乙、丙合灌时,丙也灌了 27 吨,那
么水箱最多可容纳 27+27=54 吨水.

11.某水池的容积是 100 立方米,它有甲、乙两个进水管和一个排水管.甲、乙两管单独灌满水池分


别需要 10 小时和 15 小时.水池中原有一些水,如果甲、乙两管同时进水而排水管放水,需要 6 小时
将水池中的水放完;如果甲管进水而排水管放水,需要 2 小时将水池中的水放完.问水池中原有水多
少立方米?

【分析与解】 甲每小时注水 100÷10=10(立方米),


20
乙每小时注水 100÷15= (立方米),
3
设排水管每小时排水量为“排”,
20 50
则(“排”-10- )×3=(“排”-10),整理得 3“排”-3× =“排”-10,2“排”=40,则“排”
3 3
=20.
所以水池中原有水(20—10)×2=20(立方米).

12.一个水池,底部安有一个常开的排水管,上部安有若干个同样粗细的进水管.当打开 4 个进
水管时,需要 5 小时才能注满水池;当打开 2 个进水管时,需要 15 小时才能注满水池.现在需要在
2 小时内将水池注满,那么最少要打开多少个进水管?

【分析与解】 记水池的容积为“1”,设每个进水管的工作效率为“进”,排水管的工作效率为“排”,那
么有:
1 1
4“进”-“排”= , 2“进”-“排”= .
5 15
1 1 2 1 1
所以有,2“进”=( - )= ,那么“进”= ,则“排”= .
5 15 15 15 15

学而思奥数网 www.aoshu.cn Page 31 of 137


www.baoshi2011.com

题中需同时打开 x 个进水管 2 小时才能注满,有:


1 1 1 1
x“进”-“排”= ,即 x- = ,解得 x=8.5
2 15 15 2
所以至少需打开 9 个进水管,才能在 2 小时内将水池注满.

13.蓄水池有甲、丙两条进水管和乙、丁两条排水管.要灌满一池水,单开甲管需要 3 小时,单开
1
丙管需要 5 小时.要排光一池水,单开乙管需要 4 小时,单开丁管需要 6 小时.现在池内有 池水.
6
如果按甲、乙、丙、丁的顺序循环开各水管,每次每管开 1 小时,问经过多少时间后水开始溢出水池?

l 小时,共开 4 小时,池内灌进的水是全池
【分析与解】 方法一:甲、乙、丙、丁四个水管,按顺序各开
1 1 1 1 70
的 - + - = .
3 4 5 6 6
最优情况为:在完整周期后的 1 小时内灌满一池水.因为此时为甲管进水时间,且甲的效率是四条
管子中最大的.
1 1 1
那么在最优情况下:完整周期只需注入 1- - = 池水.
6 3 2
1 70 30 2
所需周期数为 ÷ = =4
2 6 7 7
1 7 1 7 3
那么,至少需要 5 个完整周期,而 5 个完整周期后,水池内有水 + ×5= + =
6 60 6 12 4
3 1 1 1 3
剩下 l- = 池水未灌满,而完整周期后 l 小时内为甲注水时间,有 ÷ = (小时).
4 4 4 3 4
3 3
所以,需 5 个完整周期即 20 小时,再加上 小时,即 20 小时后水开始溢出.
4 4
1
方法二:甲、乙、丙、丁四个水管,按顺序各开 1 小时,共开 4 小时,池内灌进的水是全池的 -
3
1 1 1 7
+ - = .
4 5 6 60
1 7 17
加上池内原有的水,池内有水: + = .
6 60 60
17 7 45
再过四个 4 小时,也就是 20 小时后,池内有水: + ×4= ,在 20 小时后,只需要再灌
60 60 60
45 1
水 1- = ,水就开始溢出.
60 4
1 1 3 3 3 3
÷ = (小时),即再开甲管 小时,水开始溢出,所以 20+ =20 (小时)后,水开始溢出
4 3 4 4 4 4
水池.
1
方法三:甲、乙、丙、丁四个水管,按顺序各开 1 小时,共开 4 小时,池内灌进的水是全池的 -
3
1 1 1 7
+ - = .
4 5 6 60
1 7 17 43
一个周期后,池内有水: + = , 有待注入;
6 60 60 60

学而思奥数网 www.aoshu.cn Page 32 of 137


www.baoshi2011.com

17 7 24 36 3
二个周期后,池内有水: + = , 即 有先待注入;
60 60 60 60 5
24 7 31 29
三个周期后,池内有水: + = , 有待注入;
60 60 60 60
31 7 38 22 11
四个周期后,池内有水: + = , 即 有待注入;
60 60 60 60 30
38 7 45 15 1
五个周期后,池内有水: + = , 即 有待注入.
60 60 60 60 4
1 1 1 3
而此时,只需注入 的水即可,小于甲管 1 小时注入的水量,所以有 ÷ = (小时),即再开甲
4 4 3 4
3 3 3
管 小时,水开始溢出,所以 20+ =20 (小时)后,水开始溢出水池.
4 4 4
评注:这道题中要求的是第一次溢出,因为在一个周期内不是均匀增加或减少,而是有时增加有时
又减少,所以不能简单的运用周期性来求解,这样往往会导致错误的解答,至于为什么?我们给出一个
简单的问题,大家在解完这道题就会知晓.
有一口井,深 20 米,井底有一只蜗牛,蜗牛白天爬 6 米,晚上掉 4 米,问蜗牛爬出井需多少时间?

14.一个水池,地下水从四壁渗入,每小时渗入该水池的水是固定的.当这个水池水满时,打开 A
管,8 小时可将水池排空;打开 B 管,10 小时可将水池排空;打开 C 管,12 小时可将水池排空.如
果打开 A,B 两管,4 小时可将水池排空,那么打开 B,C 两管,将水池排空需要多少时间?
【分析与解】 设这个水池的容量是“1”
1
A 管每小时排水量是: +每小时渗入水量;
8
1
B 管每小时排水量是: +每小时渗入水量;
10
1
C 管每小时排水量是: +每小时渗入水量;
12
1
A、B 两管每小时排水量是: +每小时渗入水量.
4
1 1 1
因为 +每小时渗入水量+ +每小时渗入水量= +每小时渗入水量,因 此,每小时渗入水量
8 10 4
1 1 1 1
是: -( + )= .
4 8 10 40
那么有 A、B、C 管每小时的排水量如下表所示:

于是打开 B、C 两管,将水池排空需要


1 13 1 5
1÷( + - )=1÷ =4.8(小时).
8 120 40 24

学而思奥数网 www.aoshu.cn Page 33 of 137


www.baoshi2011.com

第 7 讲 牛吃草问题

牛吃草问题在普通工程问题的基础上,工作总量随工作时间均匀的变化,这样就增加了难度.
牛吃草问题的关键是求出工作总量的变化率.
下面给出几例牛吃草及其相关问题.

1. 草场有一片均匀生长的草地,可供 27 头牛吃 6 周,或供 23 头牛吃 9 周,那么它可供 21 头牛吃


几周?(这类问题由牛顿最先提出,所以又叫“牛顿问题”.)

【分析与解】 27 头牛吃 6 周相当于 27×6=162 头牛吃 1 周时间,吃了原有的草加上 6 周新长的草;


23 头牛吃 9 周相当于 23×9=207 头牛吃 1 周时间,吃了原有的草加上 9 周新长的草;于是,多
出了 207-162=45 头牛,多吃了 9-6=3 周新长的草.所以 45÷3=15 头牛 1 周可以吃 1 周新长出的
草.即相当于给出 15 头牛专门吃新长出的草.于是 27-15=12 头牛 6 周吃完原有的草,现在有 21
头牛,减去 15 头吃长出的草,于是 21-15=6 头牛来吃原来的草;
所以需要 12×6÷6=12(周),于是 2l 头牛需吃 12 周.
评注 :我们求出单位“1”面积的草需要多少头年来吃,这样就把问题化归为一般工程问题了.
一般方法:
先求出变化的草相当于多少头牛来吃:(甲牛头数×时间甲-乙牛头数×时间乙)÷(时间甲-时间乙);
再进行如下运算:(甲牛头数-变化草相当头数)×时问甲÷(丙牛头数-变化草相当头数)=时间丙.
或者:(甲牛头数-变化草相当头数)×时间甲÷时间丙+变化草相当头数丙所需的头数.

2.有三块草地,面积分别是 4 公顷、8 公顷和 10 公顷.草地上的草一样厚而且长得一样快.第一


块草地可供 24 头牛吃 6 周,第二块草地可供 36 头牛吃 12 周.问:第三块草地可供 50 头牛吃几周?
【分析与解】 我们知道 24×6=144 头牛吃一周吃 2 个(2 公顷+2 公顷周长的草).36×12=432 头
牛吃一周吃 4 个(2 公顷+2 公顷 12 周长的草).于是 144÷2=72 头牛吃一周吃 2 公顷+2 公顷 6 周

学而思奥数网 www.aoshu.cn Page 34 of 137


www.baoshi2011.com

长的草.432÷4=108 头牛吃一周吃 2 公顷+2 公顷 12 周长的草.所以 108-72=36 头牛一周吃 2


公顷 12—6=6 周长的草.即 36÷6=d 头牛 1 周吃 2 公顷 1 周长的草.
对每 2 公顷配 6 头牛专吃新长的草,则正好.于是 4 公顷,配 4÷2×6=12 头牛专吃新长的草,即
24-12=12 头牛吃 6 周吃完 4 公顷,所以 1 头牛吃 6×1÷(4÷2)=36 周吃完 2 公顷.
所以 10 公顷,需要 10÷2×6=30 头牛专吃新长的草,剩下 50-30=20 头牛来吃 10 公顷草,要
36 ×(10÷2)÷20=9 周.
于是 50 头牛需要 9 周吃 10 公顷的草.

3.如图,一块正方形的草地被分成完全相等的四块和中间的阴影部分,已 知草在各处都是同样速度
均匀生长.牧民带着一群牛先在①号草地上吃草,两天之后把①号草地的草吃光. (在这 2 天内其他草
地的草正常生长)之后他让一半牛在②号草地吃草,一半牛在③号草地吃草,6 天后又将两个草地的草
1
吃光.然后牧民把 的牛放在阴影部分的草地中吃草,另外号的牛放在④号草地吃草,结果发
3
现它们同时把草场上的草吃完.那么如果一开始就让这群牛在整块草地上吃草,吃完这些草需
要多少时间?
【 分 析 与 解 】 一群牛,2 天,吃了 1 块+1 块 2 天新长的;一群牛,6 天,吃了 2 块+2 块
1
2+6=8 天新长的;即 3 天,吃了 1 块+1 块 8 天新长的.即 群牛,1 天,吃了 1 块 1 天新长的.
6
1 2
又因为, 的牛放在阴影部分的草地中吃草,另外 的牛放在④号草地吃草,它们同时吃完.所以,
3 3
1 9 1 9 3
③=2 × 阴 影 部 分 面 积 . 于 是 , 整 个 为 4 + = 块 地 . 那 么 需 要 × = 群 牛 吃 新 长 的 草 , 于 是
2 2 6 2 4
1 9 3 1 9 3
1−
() × 2 × =现在 ()
× 1 − .所以需要吃:()()= 1− ×2 × ÷ 1 − 30 天.
6 2 4 6 2 4
所以,一开始将一群牛放到整个草地,则需吃 30 天.

4.现在有牛、羊、马吃一块草地的草,牛、马吃需要 45 天吃完,于是马、羊吃需要 60 天吃完,于是


牛、羊吃需要 90 天吃完,牛、羊一起吃草的速度为马吃草的速度,求马、牛、羊一起吃,需多少时间?

【分析与解】 我们注意到:
牛、马 45 天吃了 原有+45 天新长的草① → 牛、马 90 天吃了
2 原有+90 天新长的草⑤
马、羊 60 天吃了 原有+60 天新长的草②
牛、羊 90 天吃了 原有+90 天新长的草③

↓ ↓ ↓
马 90 天吃了 原有+90 天新长的草④
所以,由④、⑤知,牛吃了 90 天,吃了原有的草;再结合③知,羊吃了 90 天,吃了 90 天新长的草,
所以,可以将羊视为专门吃新长的草.
所以,②知马 60 天吃完原有的草,③知牛 90 天吃完原有的草.
现在将牛、马、羊放在一起吃;还是让羊吃新长的草,牛、马一起吃原有的草.
1 1
所需时间为 l÷ ( + ) =36 天.
90 60
所以,牛、羊、马一起吃,需 36 天.

学而思奥数网 www.aoshu.cn Page 35 of 137


www.baoshi2011.com

1
5. 有三片牧场,场上草长得一样密,而且长得一样快.它们的面积分别是 3 公顷、10 公顷和 24
3
公顷.已知 12 头牛 4 星期吃完第一片牧场的草,21 头牛 9 星期吃完第二片牧场的草,那么多少头牛
18 星期才能吃完第三片牧场的草?

【分析与解】 由于三片牧场的公顷数不一致,给计算带来困难,如果将其均转化为 1 公顷时的情形.

所以表 1 中,3.6-0.9=2.7 头牛吃 4 星期吃完 l 公顷原有的草,那么 18 星期吃完 1 公顷原有的草


需要 2.7÷(18÷4)=0.6 头牛,加上专门吃新长草的 O.9 头牛,共需 0.6+0.9=1.5 头牛,18 星期
才能吃完 1 公顷牧场的草.
所以需 1.5×24=36 头牛 18 星期才能吃完第三片牧场的草.

第 8 讲 不定方程与整数分拆

求二元一次方程与多元一次方程组的自然数解的方法,与此相关或涉及整数分拆的数论问题.
补充说明:对于不定方程的解法,本讲主要利用同余的性质来求解,对于同余性质读者可参考《思维
导引详解》五年级[第 15 讲 余数问题].
解不定方程的 4 个步骤:①判断是否有解;②化简方程;③求特解;④求通解.

学而思奥数网 www.aoshu.cn Page 36 of 137


www.baoshi2011.com

本讲讲解顺序:③ ⇒ 包括 1、2、3 题 ⇒ ④ ⇒ ② ⇒ ①包括 4、5 题 ⇒ ③ ⇒ 包括 6、7 题,其中③④步骤


中加入百鸡问题.
复杂不定方程:⑧、⑨、⑩依次为三元不定方程、较复杂不定方程、复杂不定方程.
整数分拆问题:11、12、13、14、15.

1.在两位数中,能被其各位数字之和整除,而且除得的商恰好是 4 的数有多少个?

【分析与解】 设这个两位数为 ab ,则数字和为 a + b ,这个数可以表达为

10a + b ,有 ( 10a + b ) ÷ ( a + b ) = 4
即 10a + b = 4a + 4b ,亦即 b = 2a .
注意到 a 和 b 都是 0 到 9 的整数,且 a 不能为 0,因此 a 只能为 1、2、3 或 4,相应地 b 的取值为
2、4、6、8.
综上分析,满足题目条件的两位数共有 4 个,它们是 12、24、36 和 48.

A B 17
2.设 A 和 B 都是自然数,并且满足 + = ,那么 A+B 等于多少?
11 3 33

【分析与解】 将等式两边通分,有 3A+llB=17,显然有 B=l,A=2 时满足,此时 A+B=2+1=3.

3.甲级铅笔 7 分钱一支,乙级铅笔 3 分钱一支.张明用 5 角钱恰好可以买这两种不同的铅笔共多


少支?

【分析与解】设购买甲级铅笔 x 支,乙级铅笔 y 支.
有 7 x +3 y =50,这个不定方程的解法有多种,在这里我们推荐下面这种利用余数的性质来求解
的方法:
将系数与常数对 3 取模(系数 7,3 中,3 最小):
得 x =2(mod 3),所以 x 可以取 2,此时 y 取 12; x 还可以取 2+3=5,此时 y 取 5;

 x = 2 x = 5
即 、 ,对应 x + y 为 14、10
 y = 12  y = 5
所以张明用 5 角钱恰好可以买这两种不同的铅笔共 14 支或 10 支.

4.有纸币 60 张,其中 1 分、l 角、1 元和 10 元各有若干张.问这些纸币的总面值是否能够恰好是


100 元?

【分析与解】 设 1 分、1 角、1 元和 10 元纸币分别有 a 张、b 张、c 张和 d 张,


列方程如下:

学而思奥数网 www.aoshu.cn Page 37 of 137


www.baoshi2011.com

 a + b + c + d = 60 ( 1)
由
 a + 10b + 100c + 1000d = 10000 ( 2 )
(2)(1)得 9b + 99c + 999 d = 9940 L L ③
注意到③式左边是 9 的倍数,而右边不是 9 的倍数,因此无整数解,即这些纸币的总面值不能恰好为
100 元.

5.将一根长为 374 厘米的合金铝管截成若干根 36 厘米和 24 厘米两种型号的短管,加工损耗忽略不


计.问:剩余部分的管子最少是多少厘米?

【分析与解】 24 厘米与 36 厘米都是 12 的倍数,所以截成若干根这两种型号的短管,截去的总长度


必是 12 的倍数,但 374 被 12 除余 2,所以截完以后必有剩余.剩余管料长不小于 2 厘米.
另一方面,374=27×12+4×12+2,而 36÷12=3,24÷12=2,有 3×9+2×2=31.即可截成
9 根 36 厘米的短管与 2 根 24 厘米的短管,剩余 2 厘米.
因此剩余部分的管子最少是 2 厘米.

6.某单位的职工到郊外植树,其中有男职工,也有女职工,并且有寺的职工各带一个孩子参加.男
职工每人种 13 棵树,女职工每人种 10 棵树,每个孩子种 6 棵树,他们一共种了 216 棵树.那么其
中有多少名男职工?

【分析与解】设男职工 x 人,孩子 y 人,则女职工 3 y - x 人(注意,为何设孩子数为 y 人,而不是设女


职工为 y 人),

那么有 13 x + 10 ( 3 y − x ) + 6 y =216,化简为 3 x + 36 y =216,即 x + 12 y =72.

 x = 12  x = 24  x = 36  x = 48  x = 60
有     .
 y = 5  y = 4  y = 3  y = 2  y = 1

 x = 12
但是,女职工人数为 3y − x 必须是自然数,所以只有  时, 3 y − x = 3 满足.
 y =5
那么男职工数只能为 12 名

7.一居民要装修房屋,买来长 0.7 米和 O.8 米的两种木条各若干根.如果从这些木条中取出一些接


起来,可以得到许多种长度的木条,例如: O.7+O.7=1.4 米,0.7+0.8=1.5 米.那么在 3.6 米 、
3.8 米、3.4 米、3.9 米、3.7 米这 5 种长度中,哪种是不可能通过这些木条的恰当拼接而实现的?

【分析与解】设 0.7 米,0.8 米两种木条分别 x , y 根,则 0.7 x +0.8 y =3.4


3.6,…
即 7 x +8 y =34,36,37,38,39
将系数,常数对 7 取模,有 y ≡6,l,2,3,4(mod 7),于是 y 最小分别取 6,1,

学而思奥数网 www.aoshu.cn Page 38 of 137


www.baoshi2011.com

2,3,4.
但是当 y 取 6 时,8×6=48 超过 34, x 无法取值.
所以 3.4 米是不可能通过这些木条的恰当拼接而实现的.

8.小萌在邮局寄了 3 种信,平信每封 8 分,航空信每封 1 角,挂号信每封角,她共用了 1 元 2 角 2 分.


那么小萌寄的这 3 种信的总和最少是多少封?

【分析与解】显然,为了使 3 种信的总和最少,那么小萌应该尽量寄最贵的挂号信,然后是航空信,最
后才是平信.但是挂号信、航空信的邮费都是整数角不会产生几分.
所以,2 分,10 n +2 分应该为平信的邮费, n 最小取 3,才是 8 的倍数,所以平信至少要寄 4 封,
此时剩下的邮费为 122-32=90,所以再寄 4 封挂号信,航空信 1 封即可.
于是,小萌寄的这 3 种信的总和最少是 4+1+4=9 封.

9.有三堆砝码,第一堆中每个砝码重 3 克,第二堆中每个砝码重 5 克,第三堆中每个砝码重 7 克.


现在要取出最少个数的砝码,使它们的总重量为 130 克.那么共需要多少个砝码?其中 3 克、5 克和 7
克的砝码各有几个?

【分析与解】 为了使选取的砝码最少,应尽可能的取 7 克的砝码.130÷7:18


……4,所以 3 克、5 克的砝码应组合为 4 克,或 4+7 k 克重.

设 3 克的砝码 x 个,5 克的砝码 y 个,则 3 x + 5 y = 4 + 7 k .

当 k =0 时,有 3 x + 5 y = 4 ,无自然数解;

当 k =1 时,有 3 x + 5 y = 11 ,有 x =2, y =1,此时 7 克的砝码取 17 个,所以共

需 2+1+17=21 个砝码,有 3 克、5 克和 7 克的砝码各 2、1、17 个.


当 k >1 时,7 克的砝码取得较少,而 3、5 克的砝码却取得较多,不是最少的取
砝码情形.
所以共需 2+1+17=20 个砝码,有 3 克、5 克和 7 克的砝码各 2、1、17 个.

10.5 种商品的价格如表 8—1,其中的单位是元.现用 60 元钱恰好买了 10 件商品,那么有多少种


不同的选购方式?

【分析与解】 设 B、C、D、E、A 商品依次买了 b、c、d、e、(10-b-c-d-e)


件,则有

学而思奥数网 www.aoshu.cn Page 39 of 137


www.baoshi2011.com

2.9 ( 10 − b − c − d − e ) + 4.7b + 7.2c + 10.6d + 14.9e =60.

18b + 43c + 77 d + 120e =310,显然 e 只能取 0,1,2.


Ⅰ 有 18b + 43c + 77 d =310,其中 d 可取 0,1,2,3,4.

(1)当 d=0 时,有 18b + 43c =310,将系数,常数对 6 取模得:


c ≡4(mod 6),于是 c 最小取 4,那么有 18b=310-43×4=138,b 不为自然
数.所以 d=0 时。不满足;

(2) 有 18b + 43c =233,将系数,常数对 6 取模得:

c ≡5(mod 6),于是 最小 ,那么有 18b=233-43×5=18,

(3) 有 18b + 43c =156,将系数,常数对 6 取模得:

c ≡O(mod 6),于是 c 最小取 0,那么有 18b=156,b 不为自然数,所以 d=2


时,不满足;

(4) 有 18b + 43c =79,将系数、常数对 6 取模得:

c ≡1(mod 6),于是 最小 那么有 18b=79—43=36.

(5)当 d=4 时,有 18b + 43c =2,显然不满足.

Ⅱ 有 18b + 43c + 77 d =190,其中 d 可以取 0、1、2.

(1) 有 18b + 43c =190,将系数、常数对 6 取模有:

c ≡4(mod 6),于是 最小 那么有 18b=190-43×4=18 ,

(2)当 d=1 时,有 18b + 43c =113,将系数、常数对 6 取模有:


c ≡5(mod 6),于是 c 最小取 5,即 18 b +215=113,显然 d=1 时,不满足;

(3)

有 18b + 43c =36,显然有 时

有 18b + 43c + 77 d =70, d 只能取 0,


有 18b + 43c =70,将系数、常数对 6 取模有:
c ≡4(rood 6),于是 c 最小取 4,那么有 18 b +172=70,显然不满足
最后可得到如下表的满足情况:

学而思奥数网 www.aoshu.cn Page 40 of 137


www.baoshi2011.com

共有 4 种不同的选购方法.

11.有 43 位同学,他们身上带的钱从 8 分到 5 角,钱数都各不相同.每个同学都把身上带的全部钱


各自买了画片.画片只有两种:3 分一张和 5 分一张.每 11 人都尽量多买 5 分一张的画片.问他们
所买的 3 分画片的总数是多少张?

【分析与解】 钱数除以 5 余 0,1,2,3,4 的人,分别买 0,2,4,1,3 张 3 分的画片.因此,可


将钱数 8 分至 5 角 2 分这 45 种分为 9 组,每连续 5 个在一组,每组买 3 分画片 0+2+4+1+3=10
张,9 组共买 10×9=90 张,去掉 5 角 1 分钱中买的 2 张 3 分画片,5 角 2 分中买的 4 张 3 分画片,
43 个人买的 3 分画片的总数是 90-2-4=84 张.

12.哥德巴赫猜想是说:“每个大于 2 的偶数都可以表示成两个质数之和.”试将 168 表示成两个两


位质数的和,并且其中的一个数的个位数字是 1.

【分析与解】 个位数字是 1 的两位质数有 11,31,41,61,71.


其中 168-11=157,168-31=137,168-41=127,168-61=107,都不是两位数,只有
168-71=97 是两位数,而且是质数,所以 168=71+97 是惟一解.

13.(1)将 50 分拆成 10 个质数之和,要求其中最大的质数尽可能大,那么这个最大质数是多少?

(2)将 60 分拆成 10 个质数之和,要求其中最大的质数尽可能小,那么这个最大的质数是多少?

【分析与解】 (1)首先确定这 10 个质数或其中的几个质数可以相等,不然 10 个互不相等的质数和最


小为 2+3+5+7+11+13+17+19+23+29,显然大于 50.
所以,其中一定可以有某几个质数相等.
欲使最大的质数尽可能大,那么应使最小的质数尽可能小,最小的质数为 2,且最多可有 9 个 2,那
么最大质数不超过 50—2×9=32,而不超过 32 的最大质数为 31.

又有
50 = 21+42 4
+22 +4L43
+ 2 + 3 + 31 ,所以满足条件的最大质数为 31.
8个 2

(2)最大的质数必大于 5,否则 10 个质数的之和将不大于 50.

学而思奥数网 www.aoshu.cn Page 41 of 137


www.baoshi2011.com

所以最大的质数最小为 7,为使和为 60,所以尽可能的含有多个 7.

60÷7=8……4,
60= 7+7+7+ L 4+7
1 44 2 4 3 +4 ,而 4=2+2,恰好有 60= 7+7+7+ L 4+7
1 44 2 4 3 +2+2 .即 8 个 7
8个 7 8个7

与 2 个 2 的和为 60,显然其中最大的质数最小为 7.

14.有 30 个贰分硬币和 8 个伍分硬币,用这些硬币不能构成的 1 分到 1 元之间的币值有多少种?

【分析与解】 注意到所有 38 枚硬币的总币值恰好是 100 分(即 1 元),于是除了 50 分和 100 分外,


其他 98 种币值就可以两两配对了,即
(1,99);(2,98);(3,97);(4,96);…;(49,51);
每一对币值中有一个可用若干个贰分和伍分硬币构成,则另一个也一定可以,显然 50 分和 100 分
的币值是可以组成的,因此只需要讨论币值为 1 分,2 分,3 分,…,48 分和 49 分这 49 种情况.
1 分和 3 分的币值显然不能构成.
2 分,4 分,6 分,…,46 分,48 分等 2;4 种偶数币值的都可以用若干个贰分硬币构成.
5 分,7 分,9 分,…,47 分,49 分等 23 种奇数币值的只须分别在 4 分,6 分,8 分,…46 分、48
分的构成方法上,用一枚伍分硬币去换两枚贰分硬币即可,譬如,37 分币值的,由于 36 分币值可用
18 枚贰分硬币构成,用一枚伍分硬币换下两枚贰分硬币,剩下的币值即为 37 分.
综合以上分析,不能用 30 个贰分和 8 个伍分硬币构成的 1 分到 1 元之间的币值只有四种,即 1 分,
3 分,97 分,99 分.

15.小明买红、蓝两支笔,共用了 17 元.两种笔的单价都是整数元,并且红笔比蓝笔贵.小强打算
用 35 元来买这两种笔(也允许只买其中一种),可是他无论怎么买,都不能把 35 元恰好用完.那么红
笔的单价是多少元?
【分析与解】如下表

先枚举出所有可能的单价如表 1.
再依次考虑:
首先,不能出现 35 的约数.否则只买这种笔就可以刚好用完 35 元,所以含有 7,5,1 的组合不可
能.
然后,也不能出现 35—17=18 的约数.否则先各买一支需 17 元,那么再买这种笔就可以花去 18 元,

学而思奥数网 www.aoshu.cn Page 42 of 137


www.baoshi2011.com

一共花 35 元.所以含有 9,6,3,2 的组合也不可能.


所以,只有 13+4 的组合可能,经检验 13x+4y=35 这个不定方程确实无自然数解.所以红笔的单
价为 13 元.

1.庙里有若干个大和尚和若干个小和尚,已知每 7 个大和尚每天共吃 41 个馒头,每 29 个小和尚每


天共吃 11 个馒头.平均每个和尚每天恰好吃 1 个馒头,问:庙里至少有多少个和尚.
2.小花狗和波斯猫是一对好朋友,它们在早晚见面时总要叫上几声表示问候.早晨见面,小花狗叫
两声,波斯猫叫一声;晚上见面,小花狗叫两声,波斯猫叫三声.细心的小娟对它们叫声统计了 15
天,它们并不是,每天早晚都见面,在这 15 天内它们共叫 61 声.问:波斯猫至少叫了多少声?
3.《张邱建算经》百鸡问题:今有百钱,鸡翁直钱五,鸡母直钱三,鸡雏三直一,百钱买百鸡,问鸡翁、
母、雏各几何?

第 9 讲 整数分拆

1.一般的有,把一个整数表示成两个数相加,当两个数相近或相等的时候,乘积最大.也就是把
整数分拆成两个相等或者相差 1 的两个整数.
2.一般的有,把自然数 m 分成 n 个自然数的和,使其乘积最大,则先把 m 进行对 n 的带余除法,
表示成 m=np+r,则分成 r 个(p+1),(n-r)个 P.
3.把自然数 S (S>1)分拆为若干个自然数的和(没有给定是几个),则分开的数当中最多有两个
2,其他的都是 3,这样它们的乘积最大.
4.把自然数分成若干个互不相等的整数,则先把它表示成 2+3+4+5+…+n 形式,当和等于原
数则可以,若不然,比原数大多少除去等于它们差的那个自然数.
如果仅大于 1,则除去 2,再把最大的那个数加 1.
5.若自然数 N 有 k 个大于 1 的奇约数,则 N 共有 k 种表示为两个或两个以上连续自然数之和的方
法.
即当有 m 个奇约数表示的乘积,则有奇约数 2 m -1 个奇约数.
6.共轭分拆.我们通过下面一个例子来说明共轭分拆:

如:10=4+2+2+1+1,我们画出示意图 ,我们将其翻转(将图左上到右下的对角线翻转

即得到): ,可以对应的写成 5+3+l+1,也是等于 10,即是 10 的另一种分拆方式.

我们把这两种有关联的分拆方式称为互为共轭分拆.

学而思奥数网 www.aoshu.cn Page 43 of 137


www.baoshi2011.com

1.写出 13=1+3+4+5 的共轭分拆.

【分 析与解】 画出示意图 ,翻转得到 ,对应写为 4+3+3+2+1=13,即为

13=1+3+4+5 的共轭分拆.

2.电视台要播出一部 30 集电视连续剧,若要每天安排播出的集数互不相等.则该电视连续剧最多
可以播出几天?

【分析与解】 由于希望播出的天数尽可能地多,若要满足每天播出的集数互不相等的条件下,每天播
出的集数应尽可能地少.
选择从 1 开始若干连续整数的和与 30 最接近(小于 30)的情况为 1+2+3+4+5+6+7=28,现在就
可以播出 7 天,还剩下 2 集,由于已经有 2 集这种情况,就是把 2 集分配到 7 天当中又没有引起与其
他的几天里播出的集数相同.于是只能选择从后加.即把 30 表示成:
30=1+2+3+4+5+6+9 或 30=1+2+3+4+5+7+8
即最多可以播出 7 天.

3.若干只同样的盒子排成一列,小聪把 42 个同样的小球放在这些盒子里然后外出,小明从每支
盒子里取出一个小球,然后把这些小球再放到小球数最少的盒子里去。再把盒子重排了一下.小聪回来,
仔细查看,没有发现有人动过小球和盒子.问:一共有多少只盒子?

【分析与解】 设原来小球数最少的盒子里装有 a 只小球,现在增加了 b 只,由于小聪没有发现有人动


过小球和盒子,这说明现在又有了一只装有 a 个小球的盒子,而这只盒子里原来装有(a+1)个小球.
同样,现在另有一个盒子装有(a+1)个小球,这只盒子里原来装有(a+2)个小球.
类推,原来还有一只盒子装有(a+3)个小球,(a+4)个小球等等,故原来那些盒子中装有的小球数
是一些连续整数.
现在变成:将 42 分拆成若干个连续整数的和,一共有多少种分法,每一种分法有多少个加数?
因 为 42=6×7 , 故 可 以 看 成 7 个 6 的 和 , 又 (7+5)+(8+4)+(9+3) 是 6 个 6 , 从 而
42=3+4+5+6+7+8+9,一共有 7 个加数;
又因为 42=14×3,故可将 42:13+14+15,一共有 3 个加数;
又因为 42=21×2,故可将 42=9+10+11+12,一共有 4 个加数.
所以原问题有三个解:一共有 7 只盒子、4 只盒子或 3 只盒子

4.机器人从自然数 1 开始由小到大按如下规则进行染色:
凡能表示为两个不同合数之和的自然数都染成红色,不符合上述要求的自然数染成黄色(比如 23 可
表示成两个不同合数 15 和 8 之和,23 要染红色;1 不能表示为两个不同合数之和,1 染黄色).问:
要染成红色的数由小到大数下去,第 2000 个数是多少?请说明理由.

【分析与解】 显然 1 要染黄色,2=1+1 也要染黄色,


3=1+2,4=1+3=2+2,5=1+4=2+3,6=1+5=2+4=3+3,7=1+6=2+5=3+4,8=1
+7=2+6=3+5=4+4,9=1+8=2+7=3+6=4+5,10=1+9=2+8=3+7=4+6=5+5,11=1
+10=2+9=3+8=4+7=5+6.
可见,1,2,3,4,5,6,7,8,9,11 均应染成黄色.

学而思奥数网 www.aoshu.cn Page 44 of 137


www.baoshi2011.com

下面统一观察其他自然数,说明其他自然数均要染成红色.
1)当 n 为大于等于 10 的偶数时,n=2k=4+2(k-2).
由于 n≥10,所以 k≥15,k-2≥3,2(k-2)与 4 均为合数,且不相等.于是,大于等于 10 的偶
数都可以表示两个不同的合数之和,应染成红色.
2)当 n 为大于等于 13 的奇数时,n=2k+1=9+2(k-4).
由于 n≥13,所以 k≥6,k-4≥2,2(k-2)≥4 与 9 均是合数,且不相等.也就是说,大于等于
13 的奇数均能表示为两个不同的合数之和,应染红色.
所以,除了 1,2,3,4,5,6,7,8,9,11 这 10 个数染黄色外,其余自然数均染红色,第 k
个染为红色的数是第(k+10)个自然数(k≥2).
所以第 2000 个染红色的数是 2000+10=2010.

5. 在 整 数 中 , 有 用 2 个 以 上 的 连 续 自 然 数 的 和 来 表 达 一 个 整 数 的 方 法 . 例 如 9 :
9=4+5,9=2+3+4,9 有两个用 2 个以上连续自然数的和来表达它的方法.
(1)请写出只有 3 种这样的表示方法的最小自然数.
(2)请写出只有 6 种这样的表示方法的最小自然数.

【分析与解】 关于某整数,它的 “奇数的约数的个数减 1”,就是用连续的整数的和的形式来表



种数.
根据(1)知道,有 3 种表达方法,于是奇约数的个数为 3+1=4,对 4 分解质因数 4=2×2,最
小的 15(1、3、5、15);
有连续的 2、3、5 个数相加;7+8;4+5+6;1+2+3+4+5;
根 据 (2) 知 道 , 有 6 种 表 示 方 法 , 于 是 奇 数 约 数 的 个 数 为 6+1=7 , 最 小 为
729(1、3、9、27、81、243、729),有连续的 2,3、6、9、10、27 个数相加:
364+365 ; 242+243+244 ; 119+120+…+124 ; 77+78+79+…+85 ; 36+37+…
+45;14+15+…+40.

6.从整数 1 开始不改变顺序的相加,中途分为两组,使每组的和相等.如从 1 到 3 的话,1+2=3;


从 1 到 20 的话:1+2+3+…+14=15+16+17+…+20.
请问:除上述两例外,能够列出这样的最短的整数算式是从 1 到几?

【分析与解】 我们用这种阶梯图来表示连续的数相加,假设情况见下图,

我们通过图得知,c 是公共部分,而 b+c 为原等式的右边,a +c 为原等式的左边,所以有


A × ( A + 1)
a=b,a 部分面积为 (可以看成从 1 一直加到 A),b 部分面积为 B×B(可以看作从 1 一直
2
加到 B 再又加到 1);

学而思奥数网 www.aoshu.cn Page 45 of 137


www.baoshi2011.com

A × ( A + 1)
有 =B×B.
2
可以表示为奇数×相邻的偶数÷2=B×B;
其中 A 是连续两个数中较小的一个,B 的平方等于连续两个数的乘积除以 2.
因为相邻的两个数互质,所以,偶数÷2 后与原相邻奇数也互质;
所以,奇数必定为完全平方数;偶数÷2 也为完全平方数,这样:
① 奇数为 1,则偶数为 2,除以 2,为 1,均为完全平方数. A=l, B 2 =1×2÷2=1,于是为
A+B=2,A+2B=3;所以为 l+2=3;
② 奇数为 9,则偶数为 8,除以 2,为 4,均为完全平方数.A=8, B 2 =8×9÷2=36,于是为
A+B=8+6=14,A+2B=8+2×6=20;所以为 1+2+3+…+14=15+16+17+…+20;
还可以偶数为 10,除以 2,为 5,不是完全平方数,不满足.
③ 奇数为 25,则偶数为 24,除以 2,为 12,不是完全平方数,不满足;
还可以偶数为 26,除以 2,为 13,不是完全平方数,不满足.
④ 奇数为 49,则偶数为 48,除以 2,为 24,不是完全平方数,不满足;
还可以偶数为 50,除以 2,为 25,是完全平方数.A=49, B 2 =49×50÷2=1225,于是为
A+B=49+35=84 , A+2B=49+2×35=119 . 所 以 等 式 为 l+2+3+…+84=85+86+87+…
+119(=3570).
所以所求的式子为 1+2+3+…+84=85+86+87+…+119(=3570).

7.把一个整数写成非零自然数的和的形式.如果所用的几个自然数相同,只是写的顺序不同,也
只算做一种方法.另外,只使用一个自然数,也算做一种方法.
(1)比如,把 6 用三个以内的自然数的和来表示的方法有如下七种:
6,5+1,4+2,3+3,4+l+1,3+2+1,2+2+2.请问:把 50 用三个以内的自然数的和来
表示的方法有几种?
(2)比如,把 7 用 3 以下的自然数的和来表示的方法有如下八种:
3+3+1,3+2+2,3+2+1+1,2+2+2+l,3+1+1+1+1,2+2+l+1+1,2+1+1+1+1
+1,1+l+1+1+1+1+1.请问:把 50 用 3 以下的自然数的和来表示的方法有几种?

【分析与解】 (1)我们注意到设 x+y+z=50,求 x、y、z 有多少组可能的值,并且 x、y、z 代表的数字调


换顺序只算一种.
为了方便计算,不妨设 x≤y≤z.
当 x=0 时,y+z=50,y 可以取 0~25,z 对应取值,于是有 26 组解;
当 x=1 时,y+z=49,y 可以取 1~24,z 对应取值,于是有 24 组解;
当 x=2 时,y+z=48,y 可以取 2~24,z 对应取值,于是有 23 组解;
当 x=3 时,y+z=47,y 可以取 3~23,z 对应取值,于是有 21 组解;
当 x=4 时,y+z=46, y 可以取 4~23,z 对应取值,于是有 20 组解;
…… …… …… …… …… …… ……
当 x=15 时,y+z=35,y 可以取 15~17,z 对应取值,于是有 3 组解;
当 x=16 时,y+z=34,y 可以取 16~17,z 对应取值,于是有 2 组解.
所以,共有 26+24+23+21+20+…+3+2 组可能的值;
我们知道有 17 个数的和,我们注意到这些数的规律,每个数是上一个数-2,-1,-2,-1,
…,
-2,-1;
47 + 47 + L + 5
所 以 , 我 们 这 样 计 算 26+(24+23)+(21+20)+…+(3+2)=26+ 1 4 4 2 4 43
8项

学而思奥数网 www.aoshu.cn Page 46 of 137


www.baoshi2011.com

=26+(47+5)×8÷2=26+52×
4=234
所以有 234 种不同的表示方法.
(2)我们注意一下,把 6 也分成三个以内的数的和,如:
6=1+1+4.
我们注意到从左往右看可以得到下面的数:1+1+4=6,
而从上往下看得到右边的数 3+1+1+1=6,每个数都是 3 或 3 以下.

并且不光是 6 满足,其他的也满足,当把它从左到右排列成三个数以内的和,则从上到下一定是 3
以内的数的和.
也就说是一一对应的,于是(1)的种数就是(2)所对应的种数.即 234 种.

8.洗衣服要打好肥皂,揉搓得很充分,再拧一下,当然不可能全拧干.假设使劲拧紧后,衣服上
还留有 1 千克带污物的水.现在有清水 18 千克,假设每次用来漂洗的水都是整数千克,试问留下的
污物最少是洗涤前的几分之几?

【分析与解】 我们假设分成 n 次分别为 x,y,z,……,


则每次漂洗的时候,总是加上上次剩下的 l 千克污水,则每次实际水量分别为:
x+1,y+l,z+1,…,

1 1 1 1
则最后剩下了 × × × ×L , ,要使最后残留的最少,只要分母最大即可.
x +1 y +1 z +1 L +1
注意到当 18 全部分成 2 的时候,2+1 即是 3,也就是满足我们【内容概述】第 3 条了,此时分了
9
1
18÷2=9 次,于是为   =
1
.
 3 19683
但是我们还应注意到,当分的次数越多,分母的和越大.如:当分成 10 次时,经过的水量变成
8 2

18+10=28,则此时可以是 8 个 3 千克,2 个 2 千克,此时为   × 


1 1 1
 = .
3  2  26244
于是考虑最极端的情况,我们把清水分成 18 次,此时经过的水量变成 18+18=36,为 18 个 2 千
18
1
克,此时对应   =
1
.因为每次必须是整数千克的水,所以不能再分.
2 262144
1
于是,当分成 18 次,每次 1 千克,此时剩下的污物残留量最少,为洗涤前的 .
262144

学而思奥数网 www.aoshu.cn Page 47 of 137


www.baoshi2011.com

第 10 讲 数论综合(一)

涉及知识点多、解题过程比较复杂的整数综合题,以及基本依靠数论手段求解的其他类型问题.

1.如果把任意 n 个连续自然数相乘,其积的个位数字只有两种可能,那么 n 是多少?

【分析与解】 我们知道如果有 5 个连
续的自然数,因为其内必有 2 的倍数,也有 5 的倍数,则它们乘积的个位数字只能是 0。
所以 n 小于 5.

:当 n 为 4 时,如果其内含有 5 的倍数(个位数字为 O 或 5),显然其内含有 2 的倍数,

那么它们乘积的个位数字为 0;
如果不含有 5 的倍数,则这 4 个连续的个位数字只能是 1,2,3,4 或 6,7,8,9;它们的积的
个位数字都是 4;
所以,当 n 为 4 时,任意 4 个连续自然数相乘,其积的个位数字只有两科可能.

:当 n 为 3 时,有 1×2×3 的个位数字为 6,2×3×4 的个位数字为 4,3×4×5 的

个位数字为 0,……,不满足.

:当 n 为 2 时,有 1×2,2×3,3×4,4×5 的个位数字分别为 2,6,4,0,显然

不满足.
至于 n 取 1 显然不满足了.
所以满足条件的 n 是 4.

2.如果四个两位质数 a,b,c,d 两两不同,并且满足,等式 a+b=c+d.那么,


(1)a+b 的最小可能值是多少?
(2)a+b 的最大可能值是多少?

【分析与解】两位的质数有 11,13,17,19,23,29,3l,37,41,43,47,53,59,6l,
67,71,73,79,83,89,97.
可得出,最小为 11+19=13+17=30,最大为 97+71=89+79=168.
所以满足条件的 a+b 最小可能值为 30,最大可能值为 168.

学而思奥数网 www.aoshu.cn Page 48 of 137


www.baoshi2011.com

3.如果某整数同时具备如下 3 条性质:
① 这个数与 1 的差是质数;
② 这个数除以 2 所得的商也是质数;
③ 这个数除以 9 所得的余数是 5.
那么我们称这个整数为幸运数.求出所有的两位幸运数.

【分析与解】 条件①也就是这个数与 1 的差是 2 或奇数,这个数只能是 3 或者偶数,再根据条件③,


除以 9 余 5,在两位的偶数中只有 14,32,50,68,86 这 5 个数满足条件.
其中 86 与 50 不符合①,32 与 68 不符合②,三个条件都符合的只有 14.
所以两位幸运数只有 14.

4.在 555555 的约数中,最大的三位数是多少?


【分析与解】555555=5×111×1001
=3×5×7×11×13×37
显然其最大的三位数约数为 777.

5.从一张长 2002 毫米,宽 847 毫米的长方形纸片上,剪下一个边长尽可能大的正方形,如果剩下


的部分不是正方形,那么在剩下的纸片上再剪下一个边长尽可能大的正方形.按照上面的过程不断地
重复,最后剪得正方形的边长是多少毫米?

【分析与解】 从长 2002 毫米、宽 847 毫米的长方形纸板上首先可剪下边长为 847 毫米的正方形,这


样的正方形的个数恰好是 2002 除以 847 所得的商.而余数恰好是剩下的长方形的宽,于是有:
2002÷847=2……308,847÷308=2……231,308÷231=1……77.231÷77=3.
不难得知,最后剪去的正方形边长为 77 毫米.

6.已知存在三个小于 20 的自然数,它们的最大公约数是 1,且两两均不互质.请写出所有可能的


答案.

【分析与解】 设这三个数为 a、b、c,且 a<b<c,因为两两不互质,所以它们均是合数.


小于 20 的合数有 4,6,8,9,10,12,14,15,16,18.其中只含 1 种因数的合数不满足,
所以只剩下 6,10,12,14,15,18 这 6 个数,但是 14=2×7,其中质因数 7 只有 14 含有,无法
找到两个不与 14 互质的数.
所以只剩下 6,10,12,15,18 这 5 个数存在可能的排列.

所以,所有可能的答案为(6,10,15);(10,12,15);(10,15,18).

学而思奥数网 www.aoshu.cn Page 49 of 137


www.baoshi2011.com

7.把 26,33,34,35,63,85,91,143 分成若干组,要求每一组中任意两个数的最大公约


数是 1.那么最少要分成多少组?

【 分 析 与 解 】 26=2×13 , 33=3×11 , 34=2×17 , 35=5×7 , 63= 32

×7,85=5×17,91=7×13,143=11×13.
由于质因数 13 出现在 26、91、143 三个数中,故至少要分成三组,可以分成如下 3 组:
将 26、33、35 分为一组,91、34、33 分为一组,而 143、63、85 分为一组.
所以,至少要分成 3 组.

8. 图 10-1 中两个圆只有一个公共点 A,大圆直径 48 厘米,小圆直径 30 厘米.两只甲虫同时从


A 出发,按箭头所指的方向以相同的速度分别爬了几圈时,两只甲虫首次相距最远?

【分析与解】 圆内的任意两点,以直径两端点得距离最远.如果沿小圆爬行的甲虫爬到 A 点,沿大


圆爬行的甲虫恰好爬到 B 点,两甲虫的距离便最远.
小圆周长为 π ×30=307r,大圆周长为 48 π ,一半便是 24 π ,30 与 24 的最小公倍数时 120.
120÷30=4.120÷24=5.
1
所以小圆上甲虫爬了 4 圈时,大圆上甲虫爬了 5 个 圆周长,即爬到了过 A 的直径另一点 B.这
2
时两只甲虫相距最远.

9.设 a 与 b 是两个不相等的非零自然数.
(1)如果它们的最小公倍数是 72,那么这两个自然数的和有多少种可能的数值?
(2)如果它们的最小公倍数是 60,那么这两个自然数的差有多少种可能的数值?
【 分 析 与解】 (1)a 与 b 的 最 小 公 倍 数 72=2×2×2×3×3 , 有 12 个 约 数 :
1,2,3,4,6,8,9,12,18,24,36,72.不妨设 a>b.

:当 a=72 时,b 可取小于 72 的 11 种约数,a+b≥72+1=73;

:当 a=36 时,b 必须取 8 或 24,a+b 的值为 44 或 60,均不同第一种情况中的值;

:当 a=24 时,b 必须取 9 或 18,a+b 的值为 33 或 42,均不同第一、二种情况中的

值;

学而思奥数网 www.aoshu.cn Page 50 of 137


www.baoshi2011.com

当 a=18 时,b 必须取 8,a+b=26,不同于第一、二、三种情况的值;

:当 a=12 时,b 无解;

:当 a=9 时,b 必须取 8,a+b=17,不同于第一、二、三、四情况中的值.

总之,a+b 可以有 ll+2+2+1+1=17 种不同的值.


(2)60=2×2×3×5,有 12 个约数:1,2,3,4,5,6,10,12,15,20,30,60.a、b 为
60 的约数,不妨设 a>b.

:当 a=60 时,b 可取 60 外的任何一个数,即可取 11 个值,于是 a-b 可取 11 种不

同的值:59,58,57,56,55,54,50,48,45,40,30;

.当 a=30 时,b 可取 4,12,20,于是 a-b 可取 26,18,10;

:当 a=20 时,b 可取 3,6,12,15,所以 a-b 可取 17,14,8,5;

当 a=15 时,b 可取 4,12,所以 a-b 可取 11,3;

: 当 a=12 时,b 可取 5,10,所以 a-b 可取 7,2.

总之,a-b 可以有 11+3+4+2+2=22 种不同的值.

1 3
10.狐狸和黄鼠狼进行跳跃比赛,狐狸每次跳 4 米,黄鼠狼每次跳 2 米,它们每秒钟都只跳一
2 4
3
次.比赛途中,从起点开始每隔 12 米设有一个陷阱,当它们之中有一个掉进陷阱时,另一个跳了多
8
少米?
3 1 11 3 3 9
【分析与解】 由于 12 ÷ 4 = , 12 ÷ 2 = .
8 2 4 8 4 2
3 3
所以狐狸跳 4 个 12 米的距离时将掉进陷阱,黄鼠狼跳 2 个 12 米的距离时,将掉进陷阱.
8 8
又由于它们都是一秒钟跳一次,因此当狐狸掉进陷阱时跳了 11 秒,黄鼠狼掉进陷阱时跳了 9 秒,
因此黄鼠狼先掉进陷阱,此时狐狸跳了 9 秒.
1
距离为 9× 4 =40.5(米).
2

11.在小于 1000 的自然数中,分别除以 18 及 33 所得余数相同的数有多少个?(余数可以为 0)

【分析与解】 我们知道 18,33 的最小公倍数为[18,33]=198,所以每 198 个数一次.


1~198 之间只有 1,2,3 , … ,17,198(余 O)这 18 个数除以 18 及 33 所得的余数相同,而
999÷198=5……9,所以共有 5×18+9=99 个这样的数.

学而思奥数网 www.aoshu.cn Page 51 of 137


www.baoshi2011.com

12.甲、乙、丙三数分别为 603,939,393.某数 A 除甲数所得余数是 A 除乙数所得余数的 2 倍,


A 除乙数所得余数是 A 除丙数所得余数的 2 倍.求 A 等于多少?

【分析与解】 由题意知 4 倍 393 除以 A 的余数,等于 2 倍 939 除以 A 的余数,等于甲 603 除以 A 的


余数.
即 603÷A=a……k;(2×939)÷A=b……k;(4×393)÷A=c……k.
于是有(1878-603)÷A=b-a;(1878-1572)÷A=b-c;(1572-603)÷A=c-a.
所以 A 为 1275,306,969 的约数,(1275,306,969)=17×3=51.
于是,A 可能是 51,17(不可能是 3,因为不满足余数是另一余数的 4 倍).
当 A 为 51 时,有 603÷51=11……42;939÷51=18……21;393÷51=7……36.不满足;
当 A 为 17 时,有 603÷17=35……8;939÷17=55……4;393÷17=23……2;满足.
所以,除数 4 为 17.

13.证明:形如 11,111,1111,11111,…的数中没有完全平方数.

【分析与解】 我们知道奇数的完全平方数是奇数,偶数的完全平方数为偶数,而奇数的完全平方数除
以 4 余 1,偶数的完全平方数能被 4 整除.
现在这些数都是奇数,它们除以 4 的余数都是 3,所以不可能为完全平方数.

评注:设奇数为 2n+1,则它的平方为 4n 2 +4n+1,显然除以 4 余 1.

14.有 8 个盒子,各盒内分别装有奶糖 9,17,24,28,30,31,33,44 块.甲先取走一盒,


其余各盒被乙、丙、丁 3 人所取走.已知乙、丙取到的糖的块数相同且为丁的 2 倍.问:甲取走的一盒
中有多少块奶糖?

【分析与解】 我们知道乙、丙、丁三人取走的七盒中,糖的块数是丁所取糖块数的 5 倍.
八盒糖总块数为 9+17+24+28+30+31+33+44=216.
从 216 减去 5 的倍数,所得差的个位数字只能是 1 或 6.
观察各盒糖的块数发现,没有个位数字是 6 的,只有一个个位数字是 1 的数 31.
因此甲取走的一盒中有 3l 块奶糖.

15. 在一根长木棍上,有三种刻度线.第一种刻度线将木棍分成 10 等份;第二种将木棍分成 12


等份;第三种将木棍分成 15 等份.如果沿每条刻度线将木棍锯断,那么木棍总共被锯成多少段?

1
【分析与解】 10,12,15 的最小公倍数[10,12,15]=60,把这根木棍的 作为一个长度单位,
60
这样,木棍 10 等份的每一等份长 6 个单位;12 等份的每等份长 5 个单位;15 等份的每等份长 4 单
位.

学而思奥数网 www.aoshu.cn Page 52 of 137


www.baoshi2011.com

不计木棍的两个端点,木棍的内部等分点数分别是 9,11,14(相应于 10,12,15 等份),共计


34 个.
由于 5,6 的最小公倍数为 30,所以 10 与 12 等份的等分点在 30 单位处相重,必须从 34 中减
1.
又由于 4,5 的最小公倍数为 20,所以 12 与 15 等份的等分点在 20 单位和 40 单位两处相重,必
须再减去 2.
同样,6,4 的最小公倍数为 12,所以 15 与 10 等份的等分点在 12,24,36,48 单位处相重,
必须再减去 4.
由于这些相重点各不相同,所以从 34 个内分点中减去 1,再减去 2,再减去 4,得 27 个刻度点.
沿这些刻度点把木棍锯成 28 段.

第 11 讲 立体图形

各种涉及长方体、立方体、圆柱、圆锥等立体图形表面积与体积的计算问题,解题时考虑沿某个方向的
投影常能发挥明显的作用.较为复杂的是与剪切、拼接、染色等相关联的立体几何问题.

第六届:“华罗庚金杯”少年数学邀请赛初赛第 12 题(略有改动)
1.用棱长是 1 厘米的立方块拼成如图 11-1 所示的立体图形,问该图形的表面积是多少平方厘米?

【分析与解】显然,图 11-1 的图形朝上的面与朝下的面的面积相等,都等于 3×3=9 个小正方形的面

学而思奥数网 www.aoshu.cn Page 53 of 137


www.baoshi2011.com

积,朝左的面和朝右的面的面积也相等,等于 7 个小正方形的面积;朝前的面和朝后的面的面积也相
等,都等于 7 个小正方形的面积,因此,该图形的表面积等于(9+7+7)×2=46 个小正方形的面积,
而每个小正方形面积为 l 平方厘米,所以该图形表面积是 46 平方厘米.

2.如图 11-2,有一个边长是 5 的立方体,如果它的左上方截去一个边分别是 5,3,2 的长方体,


那么它的表面积减少了百分之几?

【分析与解】 原来正方体的表面积为 5 ×5×6=150.


现 在 立 体 图 形 的 表 面 积 截 了 两 个 面 向 我 们 的 侧 面 , 它 们 的 面 积 为
(3×2)×2=12,12÷150=0.08=8%.
即表面积减少了百分之八.

3.如图 11-3,一个正方体形状的木块,棱长 l 米,沿水平方向将它锯成 3 片,每片又锯成 4 长条,


每条又锯成 5 小块,共得到大大小小的长方体 60 块.那么,这 60 块长方体表面积的和是多少平方米?

【分析与解】 我们知道每切一刀,多出的表面积恰好是原正方体的 2 个面的面积.


现在一共切了(3-1)+(4-1)+(5-1)=9 刀,而原正方体一个面的面积 1×l=1(平方米),所以表面积
增加了 9×2×1=18(平方米).
原来正方体的表面积为 6×1=6(平方米),所以现在的这些小长方体的表积之和为 6+18=24(平
方米).

4.图 11-4 中是一个边长为 4 厘米的正方体,分别在前后、左右、上下各面的中心位置挖去一个边


长 l 厘米的正方体,做成一种玩具.它的表面积是多少平方厘米?

【分析与解】原正方体的表面积是 4×4×6=96(平方厘米).
每一个面被挖去一个边长是 1 厘米的正方形,同时又增加了 5 个边长是 1 厘米的正方体作为玩具

学而思奥数网 www.aoshu.cn Page 54 of 137


www.baoshi2011.com

的表面积的组成部分.总的来看,每一个面都增加了 4 个边长是 1 厘米的正方形.


从而,它的表面积是 96+4×6=120 平方厘米.

5.图 11-5 是一个边长为 2 厘米的正方体.在正方体的上面的正中向下挖一个边长为 1 厘米的正


1
方体小间;接着在小洞的底面正中再向下挖一个边长为 厘米的小洞;第三个小洞的挖法与前两个相
2
1
同,边长为 厘米.那么最后得到的立体图形的表面积是多少平方厘米?
4

【分析与解】 因为每挖一次,都在原来的基础上,少了 1 个面,多出了 5 个面,即增加了 4 个面.


所以,最后得到的立体图形的表面积是:
1 1 1 1
2×2×6+1×l×4+× × ×4+ × ×4=29.25(平方厘米).
2 2 4 4

6.有大、中、小 3 个正方形水池,它们的内边长分别是 6 米、3 米、2 米.把两堆碎石分别沉没在中、


小水池的水里,两个水池的水面分别升高了 6 厘米和 4 米.如果将这两堆碎石都沉没在大水池的水里,
大水池的水面升高了多少厘米·
【分析与解】 放在中水池里的碎石的体积为 3×3×0.06:0.54 立方米;
放在小水池里的碎石的体积为 2×2×0.04=0.16 立方米;
则两堆碎石的体积和为 0.54+0.16=0.7 立方米,现在放到底面积为 6×6=36 平方米的大水池中,
7 700 17
则使大水池的水面升高 0.7÷36= 米= 厘米= 1 厘米
360 360 18

7.如图 11-6,从长为 13 厘米,宽为 9 厘米的长方形硬纸板的四角去掉边长 2 米的正方形,然后,


沿虚线折叠成长方体容器.这个容器的体积是多少立方厘米?
【分析与解】 容器的底面积是(13-4)×(9-4)=45(平方厘米),高为 2 厘米,所以容器得体积为:

45×2=90(立方厘米).

学而思奥数网 www.aoshu.cn Page 55 of 137


www.baoshi2011.com

8.今有一个长、宽、高分别为 21 厘米、15 厘米、12 厘米的长方体.现从它的上面尽可能大的切下一


个正方体,然后从剩余的部分再尽可能大的切下一个正方体,最后再从第二次剩余的部分尽可能大的
切下一个正方体.问剩下的体积是多少立方厘米?
【分析与解】 本题首先要确定三次切下的正方体的棱长,因为 21:15:12=7:5:4,为了叙述方
便,我们先考虑长、宽、高分别为 7 厘米、5 厘米、4 厘米的长方体.
易知第一次切下的正方体的棱长应为 4 厘米,第二次切下的正方体棱长为 3 厘米时符合要求,第
三次切下的正方体的棱长为 2 厘米时符合要求.
于是,在长、宽、高分别为 21 厘米、15 厘米、12 厘米的长方体中,第一、二、三次切下的正方体的
棱长为 12 厘米、9 厘米、6 厘米.
所以剩下的体积应为:

21×15×12-( 123 + 93 + 63 )=1107(立方厘米).

9.如图 11-7,有一个圆柱和一个圆锥,它们的高和底面直径都标在图上,单位是厘米.那么,圆
锥体积与圆柱体积的比是多少?

1 2 16
【 分 析 与 解 】 圆锥的体积是 × 2 × 4×π = π , , 圆 柱 的 体 积
3 3
是 42 × 8 × π = 128π .

16
所以,圆锥体积与圆柱体积的比是 π :128π = 1: 24 .
3

10.张大爷去年用长 2 米、宽 1 米的长方形苇席围成容积最大的圆柱形粮囤.今年改用长 3 米宽 2


米的长方形苇席围成容积最大的圆柱形的粮囤.问:今年粮囤的容积是去年粮囤容积的多少倍?

3 3 2 32 32
【分析与解】底面周长是 3,半径是 ,π × ( ) = 所以今年粮囤底面积是 ,高是 2.
2π 2π 4π 4π

22
同理,去年粮囤底面积是 ,高是 1.

32 22
( × 2) ÷ ( ×1) = 4.5.
4π 4π
因此,今年粮囤容积是去年粮囤容积的 4.5 倍.

学而思奥数网 www.aoshu.cn Page 56 of 137


www.baoshi2011.com

11.一个盛有水的圆柱形容器底面内半径为 5 厘米,深 20 厘米,水深 15 厘米.今将一个底面半


径为 2 厘米,高为 18 厘米的铁圆柱垂直放人容器中.求这时容器的水深是多少厘米?
【分析与解】若铁圆柱体能完全浸入水中,则水深与容积底面积的乘积应等于原有水的体积与圆柱体在
水中体积之和,因而水深为:

52 × π ×15 + 22 × π ×18
= 17.72 (厘米);
52 × π

它比铁圆柱体的高度要小,那么铁圆柱体没有完全浸入水中.此时容器与铁圆柱组成一个类似于
下图的立体图形.

底面积为 52 π − 22 π = 21π ,水的体积保持不变为 52 π × 15 = 315π .

315π 6
所以有水深为 = 17 (厘米),小于容器的高度 20 厘米,显然水没有溢出
21π 7
6
于是 17 厘米即为所求的水深.
7

12.如图 ll-8,用高都是 1 米,底面半径分别为 1.5 米、1 米和 0.5 米的 3 个圆柱组成一个物体.


问这个物体的表面积是多少平方米?( π 取 3.14)

【分析与解】 物体的表面积恰好等于一个大圆柱的表面积加上中、小圆柱的侧面积,即

2 × π ×1.52 + 2 × π × 1.5 ×1 + 2 × π ×1×1 + 2 × π × 0.5 ×1

= 4.5π + 3π + 2π + π
= 10.5π
≈ 32.97(平方米)
即这个物体的表面积是 32.97 平方米.

学而思奥数网 www.aoshu.cn Page 57 of 137


www.baoshi2011.com

13.某工人用薄木板钉成一个长方体的邮件包装箱,并用尼龙编织条如图 11-9 所示在三个方向上


加固.所用尼龙编织条的长分别为 365 厘米、405 厘米、485 厘米.若每个尼龙条加固时接头处都重叠
5 厘米,则这个长方体包装箱的体积是多少立方米?

【分析与解】 长方体中,高+宽=+(365-5)=180,……………………①
1
高+长= (405-5)=200,…………………………………………………②
2
1
长+宽= (485-5)=240,…………………………………………………③
2
②-① 得 长-宽=20,……………………………………………………④
④+③ 得 长=130,则宽=110,代入①得高=70,所以长方体得体积为:
70×110×30=1001000(立方厘米)=1.001(立方米).

1
14.有甲、乙、丙 3 种大小不同的正方体木块,其中甲的棱长是乙的棱长的 ,乙的棱长是丙的棱长的
2
2
.如果用甲、乙、丙 3 种木块拼成一个体积尽可能小的大正体,每种至少用一块,那么最少需要这 3
3
种木块一共多少块?
【分析与解】设甲的棱长为 1,则乙的棱长为 2,丙的棱长为 3.显然,大正方体棱长不可能是 4,否
则无法放下乙和丙各一个.
于是,大正方体的棱长至少是 5.事实上,用甲、乙、丙三种木块可以拼成棱长为 5 的大正方体,其
中丙种木块只能用 1 块;乙种木块至多用 7 块(使总的块数尽可能少 );甲种木块需用:5×5×5-
1×3×3×3-7×2×2×2=42(块).
因此,用甲、乙、丙三种木块拼成体积最小的大正方体,至少需要这三种木块一共
1+7+42=50(块).

15.有 6 个相同的棱长分别是 3 厘米、4 厘米、5 厘米的长方体,把它们的某划面染上红色,使得


有的长方体只有 1 个面是红色的,有的长方体恰有 2 个面是红色的,有的长方体恰有 3 个面是红色的,
有的长方体恰有 4 个面是红色的,有的长方体恰有 5 个面是红色的,还有一个长方体 6 个面都是红色
的,染色后把所有长;方体分割成棱长为 1 厘米的小正方体.分割完毕后,恰有一面是红色的小正方
体;最多有多少个?

学而思奥数网 www.aoshu.cn Page 58 of 137


www.baoshi2011.com

【分析与解】一面染红的长方体,显然应将 4×5 的长方体染红,这时产生 20 个一面染成红色的小正


方体,个数最多.
二面染红的长方体,显然应将两个 4×5 的长方体染红,这时产生 40 个一面染成红色的小正方体,
个数最多.
三面染红的长方体,显然应将 4×5,4×5,4×3 的面染红,于是产生 4×(5+5+3-4)=36 个一面
染成红色的小正方体,其他方法得出的一面染成红色的正方体均少于 36 个.
四 面 染 红 的 长 方 体 , 显 然 应 将 4×5 , 4×5 , 4×3 , 4×3 的 面 染 红 , 产 生 4×(5+5+3+3-
2×4)=32 个一面染成红色的正方体,其他方法得到的一面染成红色的小正方体均少于 32 个.
五面染红的长方体,应只留一个 3×5 的面不染,这时就产生(3-2)×(5-2)+(4-1)×(5+5+3+3-
2×4)=27 个一面染成红色的小正方体,其他染法得到的一面染成红色的小正方体均少于 27.
六面染红的长方体,产生 2×[(3-2)×(5-2)+(5-2)×(4-2)+(4-2)×(3-2)]=22 个一面染成红色的
小正方体.
于是最多得到:22+27+32+36+40+20=177 个一面染成红色的小正方体.

第 12 讲 几何综合(一)

几何图形的设计与构造.涉及比例与整数分解,需要添加辅助线、寻找规律或利用对称性解的较为
复杂的直线形和圆的周长与面积计算问题.

1.今有 9 盆花要在平地上摆成 9 行,其中每盆花都有 3 行通过,而且每行都通过 3 盆花.请你给出


一种设计方案,画图时用点表示花,用直线表示行.
【分析与解】 如下图所示,我们给出四种不同的排法.

学而思奥数网 www.aoshu.cn Page 59 of 137


www.baoshi2011.com

2.已知如图 12-1,一个六边形的 6 个内角都是 120°,其连续四边的长依次是 1、9、9、5 厘米.求


这个六边形的周长.

【分析与解】 如下图所示,将六边形的六条边分别延长,相交至三点,并将其标上字母,
因为∠BAF=120°,而么∠IAF=180°-∠BAF=60°.
又∠EFA=120°,而∠IFA=180°-∠EFA:60°,则
△IAF 为等边三角形.
同理△BCG、△EHD、△IGH 均为等边三角形.
在△IAF 中,有 IA=IF=AF=9(厘米),
在△BGC 中,有 BG=GC=BC=1(厘米),
有 IA+AB+BG=IG=9+9+1=19,即为大正三角形的边长, 所 以 有
IG=IH=GH=19(厘米).
则 EH=IH-IF-FE=19-9-5=5( 厘 米 ) , 在 △ EDH 中 , DH=EH=5( 厘 米 ) , 所 以 CD=GH-GC-
DH=19-1-5=13(厘米).
于是,原图中六边形的周长为 1+9+9+5+5+13=42(厘米).

3.图 12-2 中共有 16 条线段,每两条相邻的线段都是互相垂直的.为了计算出这个图形的周长,


最少要量出多少条线段的长度?

【分析与解】 如下图所示,我们想像某只昆虫绕图形爬行一周,回到原出发点,那么往右的路程等于
往左的路程,往上的路程等于往下的路程.于是只用量出往右的路程,往下的路程,再将它们的和乘
以 2 即为所求的周长.所以,最少的量出下列 6 段即可.

学而思奥数网 www.aoshu.cn Page 60 of 137


www.baoshi2011.com

4.将图 12-3 中的三角形纸片沿虚线折叠得到图 12-4,其中的粗实线图形面积与原三角形面积之比



2:3.已知图 12-4 中 3 个画阴影的三角形面积之和为 1,那么重叠部分的面积为多少?

【分析与解】设重叠部分的面积为 x,则原三角形面积为 1+2x,粗实线的面棚为 1+x.因此


(1+2x):(1+x)=3:2,解得 x=1,即重叠部分面积为 1.

5.如图 12-5,涂阴影部分的小正六角星形面积是 16 平方厘米.问:大正六角星形的面积是多少平


方厘米?

【分析与解】 如下图所示,在正六边形 ABCDEF 中, 与 面积相等,12 个 组成小正六

角星形,那么由 6 个 及 12 个 组成的正六边形的面积为 16÷12×(12+6)=24(平方厘米).

而通过下图,我们知道,正六边形 ABCDEF 可以分成 6 个小正三角形,并且它们面积相等,且与六个


的面积相等,所以大正六角星形的

积为 24÷6×12=48(平方厘米).

学而思奥数网 www.aoshu.cn Page 61 of 137


www.baoshi2011.com

6.如图 12-6 所示,在三角形 ABC 中,DC=3BD,DE=EA.若三角形 ABC 的面积是 1.则阴影


部分的面积是多少?

【 分 析 与 解 】 △ ABC 、 △ ADC 同 高 , 所 以 底 的 比 等 于 面 积 比 , 那 么 有
DC 3 3
S ∆ADC = × S∆ABC = × S∆ABC = .
BC 4 4
1 3
而 E 为 AD 中点,所以 S ∆DEC = S∆ADC = .
2 8
1 1
连接 FD,△DFE、△FAE 面积相等,设 S ∆FEA = x, 则. S ∆FDE 的面积也为 x, S ∆ABD = S∆ABC = .
4 4

1 3
S ∆BDF = S∆ABD − S∆FEA − S∆FDE = − 2 x, 而 S ∆FDC = S∆FDE + S∆DEC = x + .
4 8
1 3 3
S ∆BDF : S∆FDC = ( − 2 x);( x + ) = 1: 3 ,解得 x = .
4 8 56
3 3 3
所以,阴影部分面积为 S ∆DEC + S∆FEA = + = .
8 56 7

7.如图 12-7,P 是三角形 ABC 内一点,DE 平行于 AB,FG 平行于 BC,HI 平行于 CA,四边形
AIPD 的面积是 12,四边形 PGCH 的面积是 15,四边形 BEPF 的面积是 20.那么三角形 ABC 的面积
是多少?

学而思奥数网 www.aoshu.cn Page 62 of 137


www.baoshi2011.com

【分析与解】 有平行四边形 AIPD 与平行四边形 PGCH 的面积比为 IP 与 PH 的比,即为 12:15=4:


5.
同理有 FP:PG=20:15=4:3, DP:PE=12:20=3:5.

15
如图 12-7(a),连接 PC、HD,有△PHC 的面积为 △DPH 与△PHC 同底 PH,同高,所以面积
2
15
相 等 , 即 S ∆DPH = , 而 △ DPH 与 △ EPH 的 高 相 等 , 所 以 底 的 比 即 为 面 积 的 比 , 有
2
5 5 15 25
S ∆DPH : S∆EPH = DP : PE = 3 : 5 ,所以 S ∆EPH = × S∆DPH = × × .
3 3 2 2
IP IP 4
如图 12-7(b)所示,连接 FH、BP, S ∆IFP = S∆EPH = S∆FBP = × 10 = 8;
PH PH 5
PG PG 3 9
如图 12-7(c)所示,连接 FD、AP, S ∆DPG = S∆DFP = S∆APD = × 6 = .
FP FP 4 2

9 25
有 S ∆ABC = SY AIPD + SY BEPF + SY CGPH + S∆IFP + S∆DGP + S∆EHP = 12 + 20 + 15 + 8 + + = 72.
2 2

8.如图 12-8,长方形的面积是小于 100 的整数,它的内部有三个边长是整数的正方形,①号正


5 1
方形的边长是长方形长的 ,②号正方形的边长是长方形宽的 .那么,图中阴影部分的面积是多少?
12 8

学而思奥数网 www.aoshu.cn Page 63 of 137


www.baoshi2011.com

5 7
【分析与解】 有①号正方形的边长为长方形长的 ,则图中未标号的正方形的边长为长方形长的 .
12 12
1 7
而②号正方形的边长为宽的 ,所以未标号的正方形的边长为长方形宽的 .
8 8
7 7
所以在长方形中有: 长= 宽,则长:宽=12:8,不妨设长的为 12k,宽为 8k,则①号正方
12 8
形的边长为 5k,又是整数,所以 k 为整数,有长方形的面积为 96 k 2 ,不大于 100.所以 k 只能为 1,

即长方形的长为 12,宽为 8.
于是,图中①号正方形的边长为 5,②号正方形的边长为 1,则未标号的正方形的边长为 7,所以剩
余的阴影部分的面积为:

12 × 8 − 52 − 12 − 72 = 21.

9.如图 12-9,三个一样大小的正方形放在一个长方形的盒内,A 和 B 是两个正方形重叠部分,


C,D,E 是空出的部分,这些部分都是长方形,它们的面积比是 A:B:C:D:E=1:2:3:4:
5.那么这个长方形的长与宽之比是多少?

【分析与解】 以下用 E横 表示 E 部分横向的长度, E 坚 竖表示 E 部分竖向的长度,其他下标意义类似.

有 E横 : D横 =5:4, A 横 : B横 =l:2.

而 E横 + A 横 = D横 + B横 ,所以有 E横 : D横 : A 横 : B横 =5:4:1:2.

而 A 横 + B横 + C横 = E横 + A 横 对应为 5+1=6,那么 C横 对应为 3.

而 A 面积:B 面积:C 面积=1:2:3,所以 A 坚 = B坚 = C坚 .

有 A 坚 + C坚 竖对应为 6,所以 A 坚 = C坚 对应为 3.

那么长方形的竖边为 6+ C坚 对应为 9,长方形横边为

E横 +6+ D横 对应为 5+6+4=15.

所以长方形的长与宽的比为 15:9=5:3.

学而思奥数网 www.aoshu.cn Page 64 of 137


www.baoshi2011.com

10.如图 12-10,红、黄、绿三块大小一样的正方形纸片,放在一个正方形盒内,它们之间互相叠合.
已知露在外面的部分中,红色的面积是 20,黄色的面积是
14,绿色的面积是 lO.那么,正方形盒子的底面积是多少?

【分析与解】 如下图所示,我们将黄色的正方形纸片向左推向纸盒的过缘,有露在外面的部分,黄色
减少的面积等于绿色增加的面积,也就是说黄色、绿色部分露在外面部分的面积和不变.

并且有变化后,黄色露出面积+红色部分面积,绿色露出面积+红色部分面积,都是小正方形纸片
边长乘以大正方形盒子边长的积.
所以,黄色露出面积+红色部分面积=绿色露出面积+红色部分面积,于是.黄色露出面积=绿色
露出面积,而它们的和为 14+10=24,即黄色露出面积=绿色露出面积=12.
有黄:空白=红:绿,12:空白=20:12,解得空白=7.2,所以整个正方形纸盒的底面积为
12+7.2+20+12=51.2.

11.如图 12-11,在长 260 厘米,宽 150 厘米的台球桌上,有 6 个球袋 A,B,C,D,E,F,其


中 AB=EF=130 厘米.现在从 4 处沿 45°方向打出一球,碰到桌边后又沿 45°方向弹出,当再碰到桌
边时,仍沿 45°方向弹出,如此继续下去.假如球可以一直运动,直至落入某个球袋中为止,那么它
将落人哪个袋中?

【分析与解】 将每个点的位置用一组数来表示,前一个数是这个点到 FA 的距离,后一个数是点到 FD


的距离,于是 A 的位置为(0,150),球经过的路线为:
(0,150)→(150,0) →(260,110) →(220 , 150) →(70 , 0) →(0 , 70) →(80 , 150) →(230,0)
→(260 , 30) →(140 , 150) →(0 , 10) →(10 , 0) →(160 , 150) →(260 , 50) →(210,0)
→(60,150) →(0,90) →(90,0) →(240,150) →(260,130) →(130,0).
因此,该球最后落入 E 袋.

12.长方形 ABCD 是一个弹子盘,四角有洞.弹子从 A 出发,路线与边成 45 度角,撞到边界即反

学而思奥数网 www.aoshu.cn Page 65 of 137


www.baoshi2011.com

弹,并一直按此规律运动,直到落人一个洞内为止.如图 12-12.当 AB=4,AD=3 时,弹子最后落


入 B 洞.问:若 AB=1995,AD=1994 时,弹子最后落入哪个洞?在落入洞之前,撞击 BC 边多少次?

【分析与解】撞击 AD 边的点,每次由 A 向 D 移动 2;撞击 BC 边的点,每次由 C 向 B 移动 2.


因为第一次撞击 BC 边的点距 C 点 1,第一次撞击 AB 边的点距 A 点为 2,1994÷2=997.
所以最后落人 D 洞,在此之前撞击 BC 边 997 次.

13.10 个一样大的圆摆成如图 12-13 所示的形状.过图中所示两个圆心

A,B 作直线,那么直线右上方圆内图形面积总和与直线左下圆内图形面积总和的比是多少?

【分析与解】 直线 AB 的右上方的有 2 个完整的圆,2 个半圆,1 个 1个 而1个 1个

正好组成一个完整的圆,即共有 4 个完整的圆.

那么直线 AB 的左下方有 10-4=6 个完整的圆,每个圆的面积相等,所以直线右上方圆内图形面


积总和与直线左下圆内图形面积总和的比是 4:6=2:3.

14.在图 12-14 中,一个圆的圆心是 0,半径 r=9 厘米,∠1=∠2=15°.那么阴影部分的面积是


多少平方厘米?( π 取 3.14)

【分析与解】有 AO=OB,所以△AOB 为等腰三角形,AO=OC,所以△AOC 为等腰三角形.


∠ABO=∠1=15°,∠AOB=180°-∠1-∠ABO=150°.
∠ACO=∠2=15°,∠AOC=180°-∠2-∠ACO=150°.

学而思奥数网 www.aoshu.cn Page 66 of 137


www.baoshi2011.com

60
所以 ∠BOC=360°-∠AOB-∠AOC=60°,所以扇形 BOC 的面积为 × 92 × π ≈ 42.39 (平方厘米).
360

15.图 12-15 是由正方形和半圆形组成的图形.其中 P 点为半圆周的中点,Q 点为正方形一边的


中点.已知正方形的边长为 10,那么阴影部分的面积是多少?( π 取 3.14)

【分析与解】 过 P 做 AD 平行线,交 AB 于 O 点,P 为半圆周的中点,所以 0 为 AB 中点.

10 1
有 SABCD = 10 × 10 = 100,()
S半圆DPC = 2
×π × = 12.5π .
2 2
10 1  10   1
S∆AOP = 5 ×(),
10+ × = 37.5 S梯形OPQB =  10 +  + 5 × 5 × = 50.
2 2  2  2

阴影部分面积为 SABCD + S半圆梯形


DPC - S∆AOP − S OPQB = 100 + 12.5π − 37.5 − 50 = 12.5 + 12.5π ≈ 51.75.

第 13 讲 植树问题
内容概述
几何图形的设计与构造,本讲讲解一些有关的植树问题.
典型问题

1.今有 10 盆花要在平地上摆成 5 行,每行都通过 4 盆花.请你给出一种设计方


案,画图时用点表示花,用直线表示行.
【分析与解】 如下图所示:

2.今有 9 盆花要在平地上摆成 10 行,每行都通过 3 盆花.请你给出一


种设计方案,画图时用点表示花,用直线表示行.
【分析与解】 如下图所示:

3.今有 10 盆花要在平地上摆成 10 行,每行都通过 3 盆花.请你给出一


种设计方案,画图时用点表示花,用直线表示行·
【分析与解】 如下图所示:

4.今有 20 盆花要在平地上摆成 18 行,每行都通过 4 盆花.请你给出一种设


计方案,画图时用点表示花,用直线表示行.

学而思奥数网 www.aoshu.cn Page 67 of 137


www.baoshi2011.com

【分析与解】 如下图所示:

5.今有 20 盆花要在平地上摆成 20 行,每行都通过 4 盆花.请你给出一种设计方案,画图时用点


表示花,用直线表示行.
【分析与解】 如下图所示:

第 14 讲 数字谜综合
内容概述
各种具有相当难度、求解需要综合应用多方面知识的竖式、横式、数字及数阵图等类型的数字谜问题.

典型问题
1.ABCD 表示一个四位数, EFG 表示一个三位数, A,B,C,D,E,F,G 代表 1 至 9 中
的 不 同 的 数 字 . 已 知 ABCD+EFG=1993, 问 : 乘 积 ABCD×EFG 的 最 大 值 与 最 小 值 相 差 多
少?

【分析与解】 因为两个数的和一定时,两个数越紧接,乘积越大;两个数的差越大,乘积越小.
A 显然只能为 1,则 BCD+EFG=993,
当 ABCD 与 EFG 的积最大时,ABCD、EFG 最接近,则 BCD 尽可能小,EFG 尽可能大,有 BCD 最
小为 234,对应 EFG 为 759,所以有 1234×759 是满足条件的最大乘积;
当 ABCD 与 EFG 的积最小时,ABCD、EFG 差最大,则 BCD 尽可能大,EFG 尽可能小,有 EFG 最
小为 234,对应 BCD 为 759,所以有 1759×234 是满足条件的最小乘积;
它们的差为 1234×759—1759×234=(1000+234)×759 一(1000+759)×234=1000×(759
—234)=525000.
1 1 1 1 1
2.有 9 个分数的和为 1,它们的分子都是 1.其中的 5 个是 , , , , 另外 4 个数的
3 7 9 11 33
分母个位数字都是 5.请写出这 4 个分数.
1 1 1 1 1 2 ×101 10 ×10
【分析与解】 l 一( + + + + )= =
3 7 9 11 33 3 × 3 × 7 × 11 3 × 3 × 5 × 7 × 11
需要将 1010 拆成 4 个数的和,这 4 个数都不是 5 的倍数,而且都是 3×3×7×1l 的约数.因此,
它们可能是 3,7,9,11,21,33,77,63,99,231,693.
经试验得 693+231+77+9=1010.
1 1 1 1
所以,其余的 4 个分数是: , , , .
5 15 45 385
3.

请在上面算式的每个方格内填入一个数字,使其成为正确的等式.
1 1 1 1 1
【分析与解】 1988=2×2×7×7l=4×497, + = ,在等式两边同时乘上 ,就得
12 4 3 497 5964
1 1
+ = .显然满足题意.
1988 1491

学而思奥数网 www.aoshu.cn Page 68 of 137


www.baoshi2011.com

1 1 1 1 1 1 1
又 + = ,两边同乘以 ,就得 + = .显然也满足.
35 14 10 142 4970 1988 1420
1 1 1 1 1 1
+ = , + = 均满足.
3053 1988 1204 8094 1988 1596
4.小明按照下列算式: 乙组的数口甲组的数 ○1=
对甲、乙两组数逐个进行计算,其中方框是乘号或除号,圆圈是加号或减号他将计算结果填
入表 14—1 的表中.有人发现表中 14 个数中有两个数是错的请你改正.问改正后的两个数的
和是多少?

2 9 17
【 分 析 与 解 】 甲组的前三个数 0.625, , 都是小于 1 的数,2 与这三个数运算后,得
3 14 32
51 5 17 17
5.05,4 ,4 ;不论减 1 还是加 l 后,这三个数都比 2 大,而这是 2 与小于 1 的数运算的
64 16 32 32
结果,因此可以猜想方框内是除号.
现在验算一下:
17 81 8 81
2 ÷0.625= × = =4.05;
32 32 5 20
17 2 81 3 15
2 ÷ = × =3 ;
32 3 32 2 64
17 9 81 14 63 15
2 ÷ = × = =3 ;
32 14 32 9 16 16
17 27
2 ÷3= .
32 32
5
从上面四个算式来看,圆圈内填加号,这样有三个结果是对的,而 4 是错的.
16
按照算式
乙组的数÷甲组的数+1…………………………*
2
2÷3+1=1 ,显然不为 1.5,上面已认定 3 是正确的,因此,只有把 2 改为 1.5,才有
3
1 2
1.5÷3+1=1 ,而 1.5÷0.625+l=3.4,1.5÷ +1=3.25.
2 3
由此可见,确定的算式*是正确的.
5 15
表中有两个错误,4 应改为 4 ,2 应改为 1.5,
16 16
15 1 15 + 8 7
4 +1 =5+ =6 .
16 2 16 16
7
改正后的两个数的和是 6 .
16
5.图 14—3 中有大、中、小 3 个正方形,组成了 8 个三角形.现在先把 1,2,3,4 分别填
在 大 正 方 形 的 4 个 顶 点 上 , 再 把 1, 2, 3, 4 分 别 填 在 中 正 方 形 的 4 个 顶 点 上 , 最 后 把

学而思奥数网 www.aoshu.cn Page 69 of 137


www.baoshi2011.com

1,2,3,4 分别填在小正方形的 4 个项点上.


(1)能否使 8 个三角形顶点上数字之和都相等?如果能,请给出填数方法:如果不能,请说明理由.

(2)能否使 8 个三角形顶点上数字之和各不相同?如果能,请给出填数方法;如果不能,请说明理由.

【分析与解】 (1)无论怎样填法,都不可以使八个三角形顶点上数字之和相等.
事实上,假设存在某种填法使得八个三角形顶点上数字之和都相等,不妨设每个三角形顶点上数字
之和为 k.
在计算八个三角形顶点上数字之和时,大正方形四个顶点上每个数字恰好使用过一次;中正方形四
个顶点上每个数字各使用过三次;小正方形四个顶点上每个数字各使用过二次.
因此,这八个三角形顶点上数字之和的总和为:
8k=(1+2+3+4)+3×(1+2+3+4)+2×(1+2+3+4),即 8k=60,k 不为整数,矛盾,所以假设
是错误的.
(2)易知:不可能做到三角形的三个顶点上数字完全相同,所以三角形顶点上数字之和最小为 1
+1+2=4,最大为 3+4+4=11.
而 4~11 共 8 个数,于是有可能使得 8 个三角形顶点上数字之和各不相同,可如下构造,且填法
不惟一.图(a)和图(b)是两种填法.

6.图 14—5 中 有 11 条 直 线. 请 将 1 至 11 这 11 个 数 分别 填 在 11 个 圆 圈里 , 使 每一 条 直
线上所有数的和相等.求这个相等的和以及标有*的圆圈中所填的数.

【分析与解】 表述 1:设每行的和为 S,在左下图中,除了 a 出现 2 次,其他数字均只出现了 1


次,并且每个数字都出现了,于是有 4S=(1+2+3+…+11)+a=66+a;

学而思奥数网 www.aoshu.cn Page 70 of 137


www.baoshi2011.com

在右上图中除了 a 出现 5 次,其他数字均只出现了 1 次,并且每个数字都出现了,于是有


5S=(1+2+3+…11)+4a=66+4a.

 4S = 66 + a(1)
综合以上两式  ,
5S = 66 + 4a (2)
①×5-②×4 得 66-11a=0,所以 a=6,则 S=18.
考虑到含有*的五条线,有 4*+(1+2+3+4+…+11)-t=5S=90.即 4*-t=24,由 t 是 1~11 间
的数且 t≠*,可知*=7,而每行相等的和 S 为 18.

表述 2:如下图所示,在每个圆圈内标上字母,带有*的圆圈标为 x,

首先考虑以下四条直线:(h、f、a),(i、g、a),(x、d、b),(j、e、c),除了标有 a 的圆圈外,其余每个
圆圈都出现了一次,而标有 a 的圆圈出现了两次,设每条直线上数字之和为 S,则有:
(1+11)×11÷2+a=4S,即 66+a=4S.
再考虑以下五条直线:(h、f、a),(i、g、a),(j、x、a),(e、d、a),(c、b、a),同理我们可得到
66+4a=5S.

66 + a = 4 S
综合两个等式  ,可得 a 为 6,每条直线上和 S 为 18.
66 + 4a = 5S
最后考虑含 x 的五条直线:(x、h),(x、g、f),(j、x、a),(x、d、b),(i、x、c).其中除了 x 出现了 5 次,
e 没有出现,其他数字均只出现了一次,于是可以得到:
66+4x-e=5S=90,即 4x-e=24,由 e 是 1—11 间的数且 e≠x 可知 x=7.
即每行相等的和 S 为 18,*所填的数为 7.
7.一 个六位数 ,把个位 数字移到 最前面便 得到一个 新的六位 数,再将 这个六位 数的个位 数
字移到最前面又得到一个新的六位数,如此共进行 5 次所得的新数连同原来的六位数共 6 个数
称为一组循环数.已知一个六位数所生成的一组循环数恰巧分别为此数的 l 倍,2 倍,3 倍,4
倍,5 倍,6 倍,求这个六位数.

学而思奥数网 www.aoshu.cn Page 71 of 137


www.baoshi2011.com

1 . . 2 . . 3 . . 4 . . 5
【分析与解】方法一 : = 0.14285 7 , = 0.285714 , = 0.4 28571 , = 0.571428 , =
7 7 7 7 7
. . 6 . .
0.714285 , 7 = 0.85714 2 。
对应有 142857,285714,428571,571428,714285,857142,它们依次是 142857 的
1、2、3、4、5、6 倍.
且只用了 1、4、2、8、5、7 这 6 个数字,满足题意.
所以这个六位数为 142857.
方法 二 :首先可以确定最小的六位数的首位为 1,不然 2*****的 6 倍就不是六位数,于是不妨设

这个六位数为 1abcde ,那么 6 个六位数中必定存在一个数为 abcde1 .

而个位数字 1,只能由 1×1,3×7 或 9×9 得到.但是 abcde1 只能对应为 1abcde ×(2—6),所以

只能是 1abcde ×3 得到.即 abcde1 = 1abcde ×3.

于是,我们不难递推出 d 为 5,c 为 8,b 为 2,a 为 4,所以这个六位数为 142857.

方法三:部分同方法二, abcde1 = 1abcde ×3.

那么有 abcde ×10+l=(100000+ abcde )×3,解得 abcde =42857.

所以这个六位数为 142857.

15 讲 计数综合 1
内容概述
将关键的已知数据看作变量,得到一类结构相同的计数问题,通过建立这些问题的结果所构成数列
的递推关系,逐步地求得原问题的答案.与分数、几何等相关联的计数综合题.
典型问题
1.一个长方形把平面分成两部分,那么 3 个长方形最多把平面分成多少部分?
【分析与解】 一个长方形把平面分成两部分.第二个长方形的每一条边至多把第一个长方形的内
部分成 2 部分,这样第一个长方形的内部至多被第二个长方形分成五部分.
同理,第二个长方形的内部至少被第一个长方形分成五部分.这两个长方形有公共部分 (如下图,
标有数字 9 的部分).还有一个区域位于两个长方形外面,所以两个长方形至多把平面分成 10 部分.
第三个长方形的每一条边至多与前两个长方形中的每一个的两条边相交,故第一条边被隔成五条小

学而思奥数网 www.aoshu.cn Page 72 of 137


www.baoshi2011.com

线段,其中间的三条小线段中的每一条线段都把前两个长方形内部的某一部分一分为二,所以至多增
加 3×4=12 个部分.而第三个长方形的 4 个顶点都在前两个长方形的外面,至多能增加 4 个部分.
所以三个长方形最多能将平面分成 10+12+4=26.

2.一个楼 梯共有 10 级台阶, 规定每步 可以迈 1 级台阶或 2 级台 阶,最多 可以迈 3 级台阶 .


从地面到最上面 1 级台阶,一共可以有多少种不同的走法?

【分析与解】 我们知道最后一步可以迈 1 级台阶、2 级台阶或 3 级台阶,也就是说可以从倒数第 1、2


或 3 级台阶直接迈入最后一级台阶.
即最后一级台阶的走法等于倒数第 1、2 和 3 级台阶的走法和.而倒数第 l 级台阶的走法等于倒数第
2、3 和 4 级台阶的走法和,……
如果将 1、2、3……级台阶的走法依次排成一个数列,那么从第 4 项开始,每一项等于前 3 项的和.
有 1,2,3 级台阶的走法有 1,2,4 种走法,所以 4,5,6,7,8,9,10 级台阶的走法有
7,13,24,44,81,149,274 种走法.
3.一个圆上有 12 个点 A1,A2,A3, …,A11,A12.以它们为顶点连三角形,使每个点恰
好是一个三角形的顶点,且各个三角形的边都不相交.问共有多少种不同的连法?

【分析与解】我们采用递推的方法.
I 如果圆上只有 3 个点,那么只有一种连法.

Ⅱ 如果圆上有 6 个点,除 A1 点所在三角形的三顶点外,剩下的三个点一定只能在 A1 所在三角形的


一条边所对应的圆弧上,表 1 给出这时有可能的连法.
Ⅲ 如果圆上有 9 个点,考虑 A1 所在的三角形.此时,其余的 6 个点可能分布在:
①A1 所在三角形的一个边所对的弧上;
② 也可能三个点在一个边所对应的弧上,另三个点在另一边所对的弧上.
在表 2 中用“+”号表示它们分布在不同的边所对的弧.
如果是情形①,则由Ⅱ,这六个点有三种连法;
如果是情形②,则由①,每三个点都只能有一种连法.

学而思奥数网 www.aoshu.cn Page 73 of 137


www.baoshi2011.com

共有 12 种连法.
Ⅳ 最后考虑圆周上有 12 个点.同样考虑 A1 所在三角形,剩下 9 个点的分布有三种可能:
①9 个点都在同一段弧上:
② 有 6 个点是在一段弧上,另三点在另一段弧上;
③ 每三个点在 A1 所在三角形的一条边对应的弧上.得到表 3.

共有 12×3+3×6+1=55 种.
所以当圆周上有 12 个点时,满足题意的连法有 55 种.
4.现在流行的变速自行车,在主动轴和后轴分别安装了几个齿数不同的齿轮.用链条连接不同搭配
的齿轮,通过不同的传动比获得若干挡不同的车速.“希望牌”变速自行车主动轴上有 3 个齿轮,齿数
分别是 48,36,24;后轴上有 4 个齿轮,齿数分别是 36,24,16,12.问:这种变速车一共有多
少挡不同的车速?

【分析与解】算出全部的传动比,并列成表:

3
这里有 4 对传动比是相同的:1, ,2,3,将重复的传动比去掉,剩下 8 个不同的比,所
2
以共有 8 挡不同的车速.

学而思奥数网 www.aoshu.cn Page 74 of 137


www.baoshi2011.com

5.分子小于 6,分母小于 60 的不可约真分数有多少个?


【分析与解】 分子的取值范围是从 1 到 5.
当分子为 1 时,分母可从 2 到 59,共有 58 个真分数,它们当然都是不可约分数.
由于 2,3,5 都是质数,因此当分子分别为 2,3,5 时,分母必须而且只需适合下列两个条件:
① 分母大于分子且小于 60.
⑦ 分母不是分子的倍数.
易知:当分子为 2 时,适合条件的分母有 29 个;
当分子为 3 时,适合条件的分母有 38 个:
当分子为 5 时,适合条件的分母有 44 个;
最后来看分子为 4 的情形,与分子为 2 基本相同,分母不能为偶数,此外分母不能为 3.所以共有
28(=29—1)个.
总之,符合要求的分数共有 58+29+38+44+28=197 个.
6.一个正方形的内部有 1996 个点,以正方形的 4 个顶点和内部的 1996 个点为顶点,将
它剪 成一些三 角形.问 :一共可 以剪成多 少个三角 形 ?如果 沿上述这 些点中某 两点之间 所连的
线段剪开算作一刀,那么共需剪多少刀?

【分析与解】方法一:如下图,采用归纳法,列出 1 个点、2 个点、3 个点…时可剪出的三角形个数,需


剪的刀数.

不难看出,当正方形内部有 n 个点时,可以剪成 2n+2 个三角形,需剪 3n+l 刀,现在内部有


1996 个点,所以可以剪成 2×1996+2=3994 个三角形,需剪 3×1996+1=5989 刀.
方法二:我们知道内部一个点贡献 360 度角,原正方形的四个顶点共贡献了 360 度角,所以当内
部 有 n 个 点 时 , 共 有 360n+360 度 角 , 而 每 个 三 角 形 的 内 角 和 为 180 度 角 , 所 以 可 剪 成
(360n+360)÷180=2n+2 个三角形.
2n+2 个三角形共有 3×(2n+2)=6n+6 条边,但是其中有 4 条是原有的正方形的边,所以正方形
内部的三角形边有 6n+6—4=6n+2 条边,又知道每条边被 2 个三角形共用,即每 2 条边是重合的,
所以只用剪(6n+2)÷2=3n+1 刀.
本题中 n=1996,所以可剪成 3994 个三角形,需剪 5989 刀.
7. 如 图 15—3, 某 城市 的 街 道由 5 条 东 西与 7 条南 北 向 马路 组 成 .现 在 要 从西 南 角 的 A
处沿最短路线走到东北角的 B 处,由于修路十字路口 C 不能通过,那么共有多少种不同走法?

【分析与解】 因为每个路口(点)只能由西边相邻点、南边相邻点走过来,所以达到每个点的走法为西边

学而思奥数网 www.aoshu.cn Page 75 of 137


www.baoshi2011.com

相邻点、南边相邻点的走法之和,并且最南方一排、最西方一排的所有点均只有 1 种走法.
因为 C 点不能通过,所以 C 处所标的数字为 0.如下图所示:

所以,从 A 到 B 满足条件的走法共有 120 种


8.经理将 要打印的 信件交给 秘书,每 次给一封 ,且放在 信封的最 上面,秘 书一有空 就从
最上面拿一封信来打.有一天共有 9 封信打,经理按第 1 封,第 2 封, …,第 9 封的顺序交给
秘书.午饭时,秘书告诉同事,已把第 8 封信打印好了,但未透露上午工作的其他情况,这个
同事很想知道是按什么顺序来打印.根据以上信息,下午打印的信的顺序有多少种可能?(没有
要打的信也是一种可能)

【分析与解】 我们根据最后一封信来计数:
(1)第 9 封信在上午送给秘书;
于是,T={1,2,3,4,5,6,7,9}
则下午打印的每种可能都是 T 的一个子集,因为秘书可以把不在子集中的信件上午一送来就打完了,
而未打别的信.集 T 有 8 个元素,故有 28=256 个不同子集(包括空集).
(2)第 9 封信在午后才送给秘书.令
S={1,2,3,4,5,6,7},
则上午未打印的信的号码是 S 的一个子集.若将 9 排在子集之后,则与⑴中的情形相同,故只有子
集中至少有一封信已把号码 9 放在该子集的非最后的位置上.对于有 k 个元素的子集,号码 9 有 k 个
位置可放,即可放在第 i 一 1 个元素之后和 i 个元素之前,i=1,2,…,k.于是不同的顺序总数为:
0×C07+1×C17+2×C27+…+7×C77=7×27÷2=7×26=448
即下午有 448 种可能的打印顺序.
所以,下午共有 256+448=704 种打印的方法.

第 16 讲 逻辑推理
内容概述
体育比赛形式的逻辑推理问题,其中存在的呼应——“一队的胜、负、平分对应着另一队的负、平、
胜”对解题有重要作用,有时宜将比赛情况用点以及连这些点的线来表示.需要从整体考虑,涉及数量
比较、整数分解等具有一定综性的逻辑推理问题.
典型问题
1.共有 4 人进行跳远、百米、铅球、跳高 4 项比赛,规定每个单项中,第一名记 5 分,第二名记 3 分,
第三名记 2 分,第四名记 1 分.已知在每一单项比赛中都没有并列名次,并且总分第一名共获 17 分,
其中跳高得分低于其他项得分;总分第三名共获 11 分,其中跳高得分高于其他项得分.问总分第二
名在铅球项目中的得分是多少?
【分析与解】 每个单项的 4 人共得分 5+3+2+1=11 分,所以 4 个单项的总分为 11×4=44 分,而

第一,三名得分为 17、11 分,所以第二、四名得分之和为 44 − (17 + 11) = 16 分 其中第四名得分最少

为 4 分,此时第二名得分最高,为 16-4=12 分;又因为第三名为 11 分,那么第二名最低为 12 分;


那么第二名只能为 12 分,此时第四名 4 分.

学而思奥数网 www.aoshu.cn Page 76 of 137


www.baoshi2011.com

于是,第一、二、三、四名的得分依次为 17、12、1l、4 分,而 17 只能是


5+5+5+2,4 只能是 1+1+1+1.
不难得到下表:

由表知总分第二名在铅球项目中的得分是 3 分.
2.4 支足球队进行单循环比赛,即每两队之间都比赛一场.每场比赛胜者得 3 分,负者得
0 分,平局各得 1 分.比赛结果,各队的总得分恰好是 4 个连续的自然数.问:输给第一名的
队的总分是多少?
4×3
【分析与解】 四个队共赛了 C4 = = 6 场,6 场总分 m 在 12(=6×2)与 18(=6×3)之间.
2

2
由于 m 是 4 个连续自然数的和,所以 m =2+3+4=5=14 或 m =3+4+5=18.
如果 m =18,那么每场都产生 3 分,没有平局,但 5=3+1+1 表明两场踢平,矛盾.
所以 m =14,14=3×2+2×4 表明 6 场中只有 2 场分出胜负.此时第一、二、三、四名得分依次为
5、4、3、2.
则第三名与所有人打平,那么第二名没有了平局,只能是第一名与第四名打平,这样第一名还有 1
局胜,第二名还有 1 局负,所以第一名胜第二名.
即输给第一名的队得 4 分.

如下图所示,在两队之间连一条线表示两队踢平,画一条 A → B, ,表示 A 胜 B, 各队用它们的得

分来表示.

评注:常见的体育比赛模式
N 个队进行淘汰赛,至少要打 N − 1 场比赛:每场比赛淘汰一名选手;
N ( N − 1)
N 个队进行循环赛,一共要打 C 2 N = 场比赛:每个队要打 N − 1 场比赛.
2
循环赛中常见的积分方式:
① 两分制:胜一场得 2 分,平一场得 1 分,负一场得 0 分;
核心关系:总积分=2×比赛场次;
② 三分制:胜一场得 3 分,平一场得 1 分,负一场得 O 分;
核心关系:总计分=3×比赛场次-1×赛平场次.
3. 6 支足球队 进行单循 环比赛, 即每两队 之间都比 赛一场. 每场比赛 胜者得 3 分,负者 得
0 分,平局各得 1 分.现在比赛已进行了 4 轮,即每队都已与 4 个队比赛过,各队已赛 4 场的
得分之和互不相同.已知总得分居第三位的队共得 7 分,并且有 4 场球踢成平局,那么总得分
居第五位的队最多可得多少分?最少可得多少分?
【分析与解】 每轮赛 3 场,最多产生 3 × 3 = 9 分,四轮最多 4 × 9 = 36 分.现在有 4 场踢成平局,每
平一场少 1 分,所以总分为 36 − 4 ×1 = 32 .
前三名得分的和至少为 7 + 8 + 9 = 24.
所以后三名的得分的和至多为 32 − 24 = 8.

学而思奥数网 www.aoshu.cn Page 77 of 137


www.baoshi2011.com

第 5 名如果得 4 分,则后三名的得分的和至少为 4 + 5 = 9, 这不可能,所以第 5 名最多得 3 分,图( a

)为取 3 分时的一种可能的赛况图.
显然第 5 名最少得 1 分,图(b)为取 1 分时的一种可能的赛况图.

评注:以下由第 5 名得分情况给出详细赛
况:

4. 某 商 品 的 编 号 是 一 个 三 位 数 . 现 有 5 个 三 位 数 : 874, 765, 123, 364, 925, 其


中每一个数与商品编号,恰好在同一位上有一个相同的数字.那么这个三位数是多少?
【分析与解】 方法一:每一个与商品编号,恰好在同一位上有一个相同的数字.五个数,就要有五
次相同,列出这五个数:874,765, 123,364,925 百位上五个数各不相同,十位上有两个 6 和
两个 2,个位上有两个 4 和两个 5.
因此,商品编号的个位数字一定和给定 5 个数中的两个个位数字相同,商品编号的十位数字一定和
给定 5 个数中的两个十位数字相同,商品编号的百位数字只能跟 5 个数中的一个百位数字相同.
若商品编号的个位数字是 5,我们就把第二个和第五个数拿走,剩下的三个数的十位数字各不相同,
无法满足题目的要求(事实上,十位数字只能取 7,而十位上只有一个 7).
若商品编号的个位数字是 4,拿走第一和第四个数后,十位上仍有两个 2,可取十位数字为 2,再拿
走第三和第五个数,剩第二个数,它的百位是 7,所以商品的编号为 724.
如果一个数与商品编号在某一位有相同数字,那么这个数与商品编号不会再有另外相同数字.因此
解的过程中用“拿走”这一说法是恰当的.
方法二:商品编号的个位数字只可能是 3、4、5.
如果是 3,那么 874,765,364,925 这 4 个数中至多有三个数与商品编号有相同数字(百位有一
个相同,十位有两个相同),还有一个数与商品编号无相同数字,矛盾.
如果是 5,那么 765,925 的个位数字是 5,从而商品号码的十位数字不是 6、2,因此必须是 7.这
时 123、364 中至少有一个与商品号码无相同数字,矛盾.

学而思奥数网 www.aoshu.cn Page 78 of 137


www.baoshi2011.com

所以,该商品号码的个位数字只能是 4,而且这个号码应为 724.


即这个三位数为 724.
5.某 楼住着 4 个女 孩和 2 个男孩, 他们的年 龄各不相 同,最大 的 10 岁, 最小的 4 岁, 最
大的女孩比最小的男孩大 4 岁,最大的男孩比最小的女孩大 4 岁.求最大的男孩的岁数.
【分析与解】 本题中最大的孩子,可能是男孩,可能是女孩.
当最大的孩子为女孩时,即最大的女孩为 10 岁,那么最小的男孩为 10-= 4 6 岁,则 4 岁定是最小
的女孩,那么最大的男孩是 4+4:8 岁,满足题意;
当最大的孩子为男孩时,即最大的男孩为 10 岁,那么最小的女孩为 10—4=6 岁.则 4 岁一定时最
小的男孩,那么最大的女孩为 4+4=8 岁,也就是说 4 个年龄不同的女孩的年龄在 6—8 之间,显然
得不到满足.
于是,最大的男孩为 8 岁. .
6.某次考试满分是 100 分,A,B,C,D,E 这 5 个人参加了这次考试.
A 说: “我得了 94 分. ”
B 说: “我在 5 个人中得分最高. ”
C 说: “我的得分是 A 和 D 的平均分,且为整数. ”
D 说: “我的得分恰好是 5 个人的平均分. ”
E 说: “我比 C 多得了 2 分,并且在 5 个人中居第二. ”
问这 5 个人各得了多少分?
【分析与解】 B、E 分别为第一、二名,C 介于 A、D 之间,则当 A 为第三时,C 为第四,D 为第五,得
5 人平均分的人为最后一名,显然不满足.
于是 D、C、A 只能依次为第三、四、五名,有 B、E、D、C、A 依次为第一、二、三、四、五名,A 为 94 分,C
为 D、A 得平均分,且为整数,所以 D 的得分为偶数,只可能为 98 或 96(如果为 100,则 B、E 无法取
值),D、C、A 得分依次为 98、96、94 或 96、95、94,有 E 比 C 高 2 分,则 E、D、C、A 得分依次为
98、98、96、94 或 97、96、95、94.对应 5 个人的平均分为 98 或 96,而 B 的得分对应为 104 或 98,
显然 B 得不到 104 分.
所以 B、E、D、C、A 的得分只能依次是 98、97、96、95、94.
7.在一次射击练习中,甲、乙、丙 3 位战士各打了 4 发子弹,全部中靶.其命中情况如下:
① 每人 4 发子弹所命中的环数各不相同;
② 每人 4 发子弹所命中的总环数均为 17 环;
③ 乙有 2 发命 中 的 环数 分 别 与甲 其 中 的 2 发 一 样, 乙 另 2 发 命 中的 环 数 与丙 其 中 的 2 发 一
样:
④ 甲与丙只有 1 发环数相同;
⑤ 每人每发子弹的最好成绩不超过 7 环.
问:甲与丙命中的相同环数是几?
【分析与解】 条件较多,一次直接求出满足所有条件的情况有些困难,争把条件分类,再逐个满足
之.
第一步:使用枚举法找出符合每发最多不超过 7 环、四发子弹命中的环型不相同,和为 17 环的所有
情况;
第二步:在这些情况中去掉不符合条件③、④的,剩下的就是符合全部条利的情况,即为答案.
满足条件①、②、⑤的只有如下四种情况:

甲 A.7 + 6 + 3 + 1 = 17(杯) 
 都有和;
1 7
乙 B.7 + 5 + 4 + 1 = 17(杯) 

C.7 + 5 + 3 + 2 = 17(杯) 
 都有和
4 5
丙 D.6 + 5 + 4 + 2 = 17(杯) 

学而思奥数网 www.aoshu.cn Page 79 of 137


www.baoshi2011.com

从上述四个式子中看出式 A 与式 B 有数字 1、7 相同;式 B 与式 D 有数字 4 和 5 相同.式 B 既与式 A


有两个数字相同,又与式 D 有两个数字相同,式 B 就是乙.
式 A 与式 D 对应为甲和丙.
式 A 与式 D 相同的数字是 6,所以甲和丙相同的环数是 6.

第 17 讲 赛况分析
内容概述
赛况分析是一些学校近年考试的热点,我们再给出几例,希望大家在掌握了下面的知识点以后,多
多练习.
常见的体育比赛模式:
N 个队进行淘汰赛,至少要打 N-1 场比赛:每场比赛淘汰一名选手;
N()
N-1
N 个队进行循环赛,一共要打 C N =
2
场比赛:每个队要打 N-1 场比赛.
2
循环赛中常见的积分方式:
① 两分制:胜一场得 2 分,平一场得 1 分,负一场得 0 分;
核心关系:总积分=2×比赛场次;
② 三分制:胜一场得 3 分,平一场得 1 分。负一场得 0 分;
核心关系:总计分=3×比赛场次一 1×赛平场次.
典型问题
2.一次围棋比赛共有 10 名选手参加,他们分别来自甲、乙、丙三个队,每队不少于 2 人,每个人
都与其他的 9 人比赛,每盘胜者得 2 分,负者得 0 分,平居各得 1 分.结果乙队平均得分为 5.2 分,
丙队平均分 17 分,试求甲队的平均分.
【分析与解】 因为每队的总分均为整数,所以乙队为 5 人,那么乙队的总为 26 分.考虑丙队的情
况:选手所能得到的最高分为 18 分,而丙队中的最高不少于 17 分.
当最高分为 18 分,次高分至多为 16 分,第三名至多 14 分,……,前两名的平均分为 17 分.
当最高分为 17 分,次高分至多为 17 分,第三名至多 14 分,……,第一名/前两名的平均分为 17
分.
因为丙队不少于 2 人,所以丙队 2 人,则丙队的总分为 34 分.

所以甲队有 10-5-2=3 人,总分为 C10 × 2 − 26 − 34 = 30 分,所以平均分为 30÷3=10 分.


2

4.五支足球队进行单循环赛,每两队之间进行一场比赛.胜一场得 3 分,平一场得 1 分,负一场得


0 分.最后发现各队得分都不相同,第三名得了 7 分,并且和第一名打平,那么这五支球队的得分从
高到低依次是多少?

学而思奥数网 www.aoshu.cn Page 80 of 137


www.baoshi2011.com

【分析与解】 每个队各赛 4 场,共赛 5×4÷2:10 场.第三名得 7 分,与第一名打平,那么剩下


的 3 场,得 6 分,只能是 3+3+0,即第二名的比赛输了,所以只能是 1+0+/+3+3.
那么,第一名为/+3+1+3+3,第二名为 0+/+3+3+3,第三名为 1+0+/+3+第四名为
0+0+0+/+3,第五名为 0+0+0+0+/.
所以,这五支球队的得分从高到低依次是 10、9、7、3、0.
6. 有五支足球队进行循环赛,每两个队之间进行一场比赛,胜者得 3 分,平者各得 1 分,负者得 0
分.现在还有一些比赛没有进行,各个队目前的得分恰好是五个连续的偶数,其中甲队积 2 分,并且
负于乙队,那么乙队现在积多少分?
2
【分析与解】 最高分为 3×4=12,而赛完后 5 支队伍的最高分为 C5 ×3=30 分,因为出现 2 分,

所以 5 个连续的偶数,可能是 0、2、4、6、8;2、4、6、8、10.
但是,2+4+6+8+10=30 分,而还有些比赛没有进行,所以只能是 0、2、4、6、8.
甲:1+0+1+1+~,乙:3+1+~,丙:1+~,丁:1+~,戊:~
所以,只能是戊为 0 分.
① 当乙为 8 分时,只能是 3+/+1+1+3,因为丙、丁在我们看来完全等价,当丁为 6 分时 1+1+~
+1+3,此时丙只能是 4 分,只能是 1+1+~+1+1,而这时戊一定有得分.所以不满足.
② 当乙为 6 分时,只能是 3+/+0+0+3,当丙为 8 分时 1+3+/+1+3,此时丁只能是 4 分,但
是只能是 1+3+1+/+~,超过 4 分.所以不满足.
③ 当乙为 4 分时,只能是 3+/+l+0+~,当丁为 8 分时 l+3+1+/+3,此时丙只能为 6 分,为
1+1+/+l+3.满足.
所以,乙的得分为 4.
8.五支足球队 A、B、C、D、E 进行单循环比赛,即每两队之间都比赛一场.每场比赛胜者得 2 分,
负者得 0 分,平局各得 1 分.已知:(1)4 队获得了冠军;(2)B 队、C 队和 D 队的得分相同,且无其它
并列情况;(3)在 C 队参加的比赛中,平局只有一场,那场的对手是 B 队;(4)D 队战胜了 A 队.请你
根据上述信息,分析出每场比赛的胜、平、负情况.
【分析与解】 根据已知条件可以画出如下赛况图:

2
因为每场比赛 2 个队共得 2 分,所以 5 个队的总分为 C5 ×2=20 分.

(1)当 B、C、D 均得 2 分,而 A 最多得到 6 分,E 最少得到 20-2×3-6=8 分,超过 A,而 A 是冠军,
所以不满足;
(2)当 B、C、D 均得 3 分,此时 E 的得分最少为 20-3×3-6=5 分,所以此时只能是 A 得 6 分,
B 、C 、D 均得 3 分,E 得 5 分.于是,A 的另外三场均是 A 胜

E 于是只能一场平,另外的 2 场为胜.由条件 3 知,E 不可能与 C 平,所以只能是与 B 或 D 打平.

学而思奥数网 www.aoshu.cn Page 81 of 137


www.baoshi2011.com

① 当 E 与 D 平,有 ,有如左下图的赛况表.

② 当 E 与 B 平,有 ,有如右上图的赛况表.

(3)当 B、C、D 均得 4 分,因为 C 只能平一场得到 1 分,而其他情况,要么得到 2 分,要么不得分,


所以不可能;
(4)当 B、C、D 均得 5 分,那么只能是 A 得 5 分,E 得 0 分,不满足.
综上所述,有 2 种赛况表满足,

第 18 讲 方程与方程组 1

学而思奥数网 www.aoshu.cn Page 82 of 137


www.baoshi2011.com

内容概述
二元、三元一次方程组的代入与加减消元法.各种可通过列方程与方程组解的应用题,求解时要恰当
地选取未知数,以便于将已知条件转化为方程.
典型问题
1.一个分数,分子与分母的和是 122,如果分子、分母郡减去 19,得到的分数
1
约简后是 .那么原来的分数是多少?
5
a
【 分 析 与 解 】 方 法 一 :设这个分数为 , 则 分 子 、 分 母 都 减 去 19 为
122 − a
a −19 −
a 19 1
= = ,即 5a-95=103-a ,解得 a = 33 ,则 122-33=89.所以原来的分
(122 −)a−19 103
− a5
33
数是
89
a a + 19
方法二 :设这个分数为变化后为 ,那么原来这个分数为 ,并且有 (a + 19) + (5a + 19)
5a 5a + 19
33
=122, ,解得。=14.所以原来的分数是 .
89

2.有两堆棋子,A 堆有黑子 350 和白子 500 个,B 堆有黑子 400 个和白子 100 个.为了使 A
堆中黑子占 50%,B 堆中黑子占 75%,那么要从 B 堆中拿到 A 堆黑子多少个?白子多少个?

【分析与解】 要使 A 堆中黑、白子一样多,从 B 堆中拿到 A 堆的黑子应比白子多 150 个,设从 B 堆中


x
拿白子 个,则拿黑子( +150)个. x
400 − ( x + 15).
依题意有 =75%, 解得 x =25. 所以要拿黑子 25+150=175 个.白子 25
400 + 100 − (2 x + 150)


3.A 种酒精中纯酒精的含量为 40%,B 种酒精中纯酒精的含量为 36%,C 种酒精中纯酒精的含
量为 35%.它们混合在一起得到了纯酒精的含量为 38.5%,的酒精 11 升,其中 B 种酒精比 C 种酒
精多 3 升.那么其中的 A 种酒精有多少升?
【分 析与解】 设 c 种酒精 x 升,则 B 种酒精戈 x+3 升,A 种酒精 ll-x-(x+3) 升.有:[11-x-(x+3)]
+4%+( x +3)×36%+ x×35%=11×38.5%解得 x =0.5.
其中 A 种酒精为 11-2x-3=7(升).
4.校早晨 6:00 开校门,晚上 6:40 关校门。下午有位同学问老师现在的时间,老师说:从开校
1 1
门到现在时间的 加上现在到关校门时间的 ,就是现在的时间.那么现在的时间是下午几点?
3 4

【分析与解】 设现在为下午 x 点.那么上午 6:00 距下午 x 点为 6+ x 小时;下午 x 点距下午 6:40


2
为6 − x 小时.
3
1 1 2 
有: × (6 + x) +  6 − x  = x ,解得 x=4. 所以现在的时间为下午 4 点.
3 4 3 
5.如图 18—2 中的短除式所示,一个自然数被 8 除余 1,所得的商被 8 除余 1,再把第二次所

学而思奥数网 www.aoshu.cn Page 83 of 137


www.baoshi2011.com

得的商被 8 除后余 7,最后得到的一个商是 a .图 18-3 中的短除式表明:这个自然数被 17 除余 4,


所得的商被 17 除余 15,最后得到的一个商是 a 的 2 倍.求这个自然数.

【分析 与 解 】 由题意 知

( 8a + 7 ) × 8 + 1 × 8 + 1 = ( 17 + 2a ) + 17 + 4, 整 理 得 512a+457=578a+259 , 即

66a=198,a=3.
于是,[(80+1)×8+1]× 8+1=1993.
6.一堆彩色球,有红、黄两种颜色.首先数出的 50 个球中有 49 个红球;以后每数出的 8 个球
中都有 7 个红球.一直数到最后 8 个球,正好数完.如果在已经数出的球中红球不少于 90%,那么这
堆球的数目最多只能有多少个?
【分析与解】 方法一 :首先数出的 50 个球中,红球占 49÷50×100%=98%.以后每次数出的球
中,红球占 7÷8×100%=87.5%. 取得次数越多,红球在所取的所有球中的百分数将越低.设取
得 x 次后,红球恰占 90%.共取球 50+8z,红球为 49+7 x .
(49+7 x )÷(50+8 x )×100%=90%,解得 x =20,所以最多可取 20 次,此时这堆球的数目最多
只能有 50+8×20=210 个.
方法 二 :设,除了开始数出的 50 个球,以后数了 n 次,那么,共有红球 49+7n,共有球 50+8n,有
49 + 7 n
≥90%,即 49+7n≥45+7.2n,解得 n ≤20,所以 n 的最大值 20.
50 + 8n
则这堆球的数目最多只能有 50+8×20:210 个.
7.有甲、乙、丙、丁 4 人,每 3 个人的平均年龄加上余下一人的年龄分别为 29,23,2l 和 17.这 4
人中最大年龄与最小年龄的差是多少?

【分析与解】 设这些人中的年龄从大到小依次为 x 、 y 、 z 、 w ,

①+②+③ 十④得:2( x +y+z+ w )=90,


x+ y+z+w
则 =15…………………………………………⑤
3
2
①-⑤ 得: x = 14 , x =21;
3
2
④-⑤ 得: z = 2 , z=3;
3

学而思奥数网 www.aoshu.cn Page 84 of 137


www.baoshi2011.com

所以最大年龄与最小年龄的差为 x − w =21—3=18(岁).

第 19 讲 方程与方程组 2
内容概述
一般的,把含有未知数的等式称为方程1
将含有未知数的个数称为“元”,如: x +y=2 就是一个二元方程,而两个含有 2 个未知数的方程合

在一起,就组成了二元方程组, { x + y =2
3 x + 4 y = 6.5
就是一个二元一次方程组.

把未知数的最高次数称为“次”,如 x 2 + y 2 = 25 就是一个二元二次方程.

如果方程组的个数等于未知数的个数,我们就称这个方程为适定方程;
如果方程组的个数少于未知数的个数,我们就称这个方程为不定方程;一般的不定方程没有确定解.
方程的基本性质:
1.方程两边同时加上或减去某个数,等号仍然成立;
2.方程两边同时乘以或除以某个非零数,等号仍然成立.
在解方程中最常用的一种技巧是移项,将方程中的某些项改变符号后,从方程的一边移到另一边的
变形叫移项.如 3 x +12=18,可以将 12 移项为 3x=18-12.
通过“代人”消去一个未知数,将方程组减少一元来解的方法叫做代入消元
法,简称代人法;
通过将两个方程相加(或相减)消去一个未知数,将方程组减少一元来解的方
法叫做加减消元法,简称加减法
典型问题

105
1.若石是自然数,且满足 = x − 6 ,试求 x 的值.
4x −1
【分析与解】4 x -1 必须是 105 的约数,105=3×5×7,当 4 x -1=7 时, x =2:当 4x-1=15 时, x
=4;当 4x-1=3 时, x =1;当 4 x -1=35 时, x =9.
所以只能是 105÷(4×9-1)=9-6,即 x =9.

《九章算术》第八卷“方程”刘徽注:程,课程也.群物总杂,各列有数,总言其实,令每行为率,二物者再程,三物者三程,皆如物数
1

程之.并列为行,故谓方程.

学而思奥数网 www.aoshu.cn Page 85 of 137


www.baoshi2011.com

2.小吴和小林两人解方程组,
{ ax− 2 y =2( )1
7 x − by (=1 ) 2
由手小吴看错了方程 ①中的 a 而得到方程组的解为

{ x= 4
y=9
,小林看错了方程②中的 b 而得到的解为 { x =3
y =8
,如果按正确的 a 、b 计算,试求出原方程组的解.

【分析与解】 因为小吴同学没有看错②,所以 { x= 4
y=9
是符合②的解,有 4×7-b×9=1,解得 b=3;

因为小林同学没有看错①,所以 { x =3
y =8
是符合①的解,有 a ×3-2×8=2,解得 a =6;

即原方程组为 { 6 x− 2 y =2
7 x − 3 y =1
解得 { x =1
y =2

3.解方程组:

{ x1 − x 2 = x 3 − x 2 = x 3− x 4= L x 2003− x 2004= x 2005− x 2004= 1


x1 − x 2 + x 3 − x 4 + L − x 2002 +x 2003− x 2004+ x 2005= 2005
【分析与解】这是一个高达 2005 元的一次方程组,必须从中发现规律才求出来未知数的值.

由 x − x = x3 −x x 3 =
所以 x1 ; x3 −
x 2=x−
3
x4 所以 x = x
1 2 2 2 4

x 3 − x 4 = x 5 − x 4 ,所以 x 3 =x 5; x 5 − x 4 = x 5 − x 6 ,所以 x 4=x 6

x 2003 − x 2004 = x 2005 − x 2004 所以 x 2003 = x 2005

于是有 x =
x1 =x =L = x 2005 , x2 = x4 = x 6 = L = x 2004 令 x = A
3 5 1

x 2 = B , 那么有 { A− B =1
1003 A−1002 B = 2005
所以 { A=1003
B =1002

即 { x1 =x 3=x 5=x 7 x= L 1003


2005
x 2=x 4 x=6=x 8= =xL = 1002
2004

4. 一只小虫从 A 爬到 B 处.如果它的速度每分钟增加 1 米,可提前 15 分钟到达.如果它的速度每


分钟再增加 2 米,则又可提前 15 分钟到达.那么 A 处到 B 处之间的路程是多少米?
【分析与解】设小虫的速度为名 x 米/分钟,从 A 到 B 所需时间为 Y 分钟,那么有:

{ (1+ x )( y −15) = xy
(3+ x )( y −30) = xy , 化简为 { y −15 x =15
y −10 x = 30 , 解得 x = 3, y = 60 所以 A、B 地相距 3×60=180
米.
5.若干学生搬一堆砖,若每人搬五块,则剩下 20 块未搬走;若每人搬 9 则最后一名学生只搬 6 块,
那么学生共有多少人?

学而思奥数网 www.aoshu.cn Page 86 of 137


www.baoshi2011.com

【分析与解】设有 n 个学生.根据砖的数量可得到方程

nk + 20 = 9n − (9 − 6) 即 n (9 − 6) =23 因为 23 是质数,所以 n 与(9-K 中一个是 23,另一个是 1.所

以只能是 n=23
评注:在这道题中,K 仅是一个过渡变量,借用 9-K≤9,求得 n=23.

第 20 讲 列方程解应用题
内容概述

列方程解决问题是一种很重要的通法,以前我们往往将应用题分成:鸡兔同笼、年龄问题、还原问题
等等,再归纳出每一类问题的解法.而现在我们就可以利用方程统一来考虑这些问题.方程思想的建
立可以说是一个很大的飞跃.

下面我们就如何找好等量关系,如何建立方程给出一些示范,希望大家体会掌握以提高自己的解题
能力.

典型问题

1 1
1.有一篮子鸡蛋分给若干人,第一人拿走 1 个鸡蛋和余下的 ,第二人拿走 2 个和余下的 ,第三
9 9
1
人拿走 3 个和余下的 ,……,最后恰好分完,并且每人分到的鸡蛋数相同,问:共有多少鸡蛋 ?分
9
给几个人?

1
【分析与解】 设原有 x 个 鸡 蛋 , 那 么 第 一 人 拿 了 1 + ( x − 1) 个 鸡 蛋 , 第 二 人 拿 了
9
1 8  1 1 8 
2 + ×  ( x − 1) − 2  个鸡蛋. 1 + ( x − 1) = 2 + ×  ( x − 1) − 2 
9 9  9 9 9 

1
解得 x = 64 ,则第一人拿了 1 + × (64 − 1) = 8 个鸡蛋,所以共有 64÷8=8 人.
9

即共有 64 个鸡蛋,分给 8 个人.

2.某人每日下午 5 时下班后有一辆汽车按时接他回家.有一天,他提前 l 小时下班,因汽车未到,


遂步行返家,在途中遇到来接他的汽车,因而比平日早 16 分钟到家,问此人是步行几分钟后遇见汽
车的?

【分析与解】设此人在步行 x 分钟以后遇见汽车,汽车的速度为“1”,汽车从家到单位需要 y 分钟.

由家到单位的总路程为 y ,如果汽车在 4 时就在单位接他,他应该提前 1 小时到家,但是现在只提


前 16 分钟到家,说明相对汽车他在 x 分钟这段路程上耽搁 44 分钟,所以汽车走这段路程只需要 x -
44 分钟.

学而思奥数网 www.aoshu.cn Page 87 of 137


www.baoshi2011.com

而汽车是从 5:00- y 从家出发,在 4:00+ x 达到相遇点.所以行驶 x + y -60 分钟.

x − 44 + ( x + y − 60) = y ,有 2 x − 104 = 0, x = 52 .

所以,此人是在步行 52 分钟后遇见汽车的.

 3.一次数学竞赛中共有 A、B、C 三道题,25 名参赛者每人至少答对了一题.在所有没有答对 A 的


学生中,答对 B 的人数是答对 C 的人数的两倍,只答对问题 A 的人数比既答对 A 又至少答对其他一题
的人数多 1.又已知在所有恰好答对一题的参赛者中,有一半没有答对 A.请问有多少学生只答对 B?

 【分析与解】设不只答对 A 的为 x 人,仅答对 B 的为 y 人,没有答对 A 但答对 B 与 C 的为 z 人.

 25 − x
y =
解得:  3 ,
 z = 23 − 3 x

y ≥ z , x ≥ 6,

  x =7 时, y 、 z 都是正整数,所以 x = 7, y = 6, z = 2 。

故只答对 B 的有 6 人.

4.河水是流动的,在 Q 点处流入静止的湖中,一游泳者在河中顺流从 P 到 Q,然后穿过湖到 R,


共用 3 小时.若他由 R 到 Q 再到 P,共需6小时.如果湖水也是流动的,速度等于河水的速度,那么
5
从 P 到 Q 再到 R 需 小时.问在这样的条件下,从 R 到 Q 再到 P 需几小时?
2

【分析与解】设游泳者的速度为 1,水速为 y,PQ=a,QR=b,则有:

学而思奥数网 www.aoshu.cn Page 88 of 137


www.baoshi2011.com

且有 1+y、 1—y、y 均不为 0.

by 1 1+ y
①-② 得 = ,即 b =  ……………………………………………………………………④
1+ y 2 2y

a + 2y 3(1 − y 2 )
③-① 得 = 3 ,即 a =  ………………………………………………………………
1− y2 2y

由②、④、⑤得

5 1+ y
× (1 + y ) = a + b = × (4 − 3 y ) ,即 5 y = 4 − 3 y .
2 2y

1 5 1 15
于是, y = .由②得 a + b = × (1 + ) = .
2 2 2 4

a + b 15 1 15
= ÷ (1 − ) = 小时.
1− y 4 2 2

15
即题中所述情况下从 R 到 Q 再到 P 需 小时.
2

第 21 讲 行程与工程
内容概述
运动路线或路况复杂,与周期性或数论知识相关联,需进行优化设计等具有相当难度的行程问题.
工作效率发生改变,要完成的项目及参加工作的对象较多的工程问题.
典型问题

学而思奥数网 www.aoshu.cn Page 89 of 137


www.baoshi2011.com

1。如图 21-l,A 至 B 是下坡,B 至 C 是平路,C 至 D 是上坡.小张和小王在上坡时步行速度是每小


时 4 千米,平路时步行速度是每小时 5 千米,下坡时步行速度是每小时 6 千米.小张和小王分别从 A
1
和 D 同时出发,1 小时后两人在 E 点相遇.已知 E 在 BC 上,并且 E 至 C 的距离是 B 至 C 距离的 .
5
当小王到达 A 后 9 分钟,小张到达 D.那么 A 至 D 全程长是多少千米?

4 1
【分析与解】 BE 是 BC 的 ,CE 是 BC 的 ,说明 DC 这段下坡,比 AB 这段下坡所用的时间多,也
5 5
就是 DC 这一段,比 AB 这一段长,因此可以在 DC 上取一段 DF 和 AB 一样长,如下图:

另外,再在图上画出一点 G,使 EG 和 EC 一样长,这样就表示出,小王从 F 到 C.小张从 B 到 G.


小王走完全程比小张走完全程少用 9 分钟,这时因为小张走 C 至 F 是上坡,而小王走 F 至 C 是下坡
(他们两人的其余行程走下坡、平路、上坡各走一样多).

6 
因此,小王从 F 至 C,走下坡所用时间是 9÷  − 1 =18(分钟).
4 
因此得出小张从 B 至 G 也是用 18 分钟,走 GE 或 CE 都用 6 分钟.走 B 至 C 全程(平路)要 30 分钟.
从 A 至曰下坡所用时间是 60-18-6=36(分钟);
从 D 至 C 下坡所用时间是 60-6=54(分钟);
6 5
A 至 D 全程长是(36+54)× +30× =11.5 千米.
60 60
2.如图 2l-2,A,B 两点把一个周长为 l 米的圆周等分成两部分.蓝精灵从 B 点出发在这个圆周上
3
沿逆时针方向做跳跃运动,它每跳一步的步长是 米,如果它跳到 A 点,就会经过特别通道 AB 滑向
8
曰点,并从 B 点继续起跳,当它经过一次特别通道,圆的半径就扩大一倍.已知蓝精灵跳了 1000 次,
那么跳完后圆周长等于多少米?

3 3
【分析与解】 ×4= 即蓝精灵跳 4 次到 A 点.圆半径扩大一倍即乘以 2 后,跳 8 次到 A 点.
8 2
圆半径乘以 4 后,跳 16 次到 A 点.
依次类推,由于 4+8+16+32+64+128+256+492=1000,所以有 7 次跳至 A 点.
1000 次跳完后圆周长是 1× 27 =128 米.
3.已知猫跑 5 步的路程与狗跑 3 步的路程相同;猫跑 7 步的路程与兔跑 5 步的路程相同.而猫跑 3
步的时间与狗跑 5 步的时间相同;猫跑 5 步的时间与兔跑 7 步的时间相同,猫、狗、兔沿着周长为 300
米的圆形跑道,同时同向同地出发.问当它们出发后第一次相遇时各跑了多少路程?

【分析与解】 方法一:由题意,猫与狗的速度之比为 9:25,猫与兔的速度之比为 25:49.

学而思奥数网 www.aoshu.cn Page 90 of 137


www.baoshi2011.com

25 49
设单位时间内猫跑 1 米,则狗跑 米,兔跑 米.
9 25
25 675
狗追上猫一圈需 300÷( -1)= 单位时间,
9 4
49 625
兔追上猫一圈需 300÷( -1)= 单位时间.
25 2
675 625
猫、狗、兔再次相遇的时间,应既是 的整数倍,又是 的整数倍.
4 2
675 625
与 的最小公倍数等于两个分数中,分子的最小公倍数除以分母的最大
4 2
 675 625  675, 625] 16875
公约数,即  , = = =8437.5.
 4 2  ( 4, 2 ) 2
上式表明,经过 8437.5 个单位时间,猫、狗、兔第一次相遇.此时,猫跑了 8437.5 米,狗跑了
25 49
8437.5× =23437.5 米,兔跑了 8437.5× =16537.5 米.
9 25
方法二: 有猫跑 35 步的路程与狗跑 21 步的路程,兔跑 25 步的路程相;而猫跑 15 步的时间与狗跑
25 步的时间,兔跑 21 步的时间相同.
15 25 21
所以猫、狗、兔的速度比为 : : ,它们的最大公约数为
35 21 25
 15 25 21  ( 15, 25, 21) 1
 , , = = .
 35 21 25  [ 35, 21, 25] 3 × 5 × 5 × 7
15 1 25 1
即设猫的速度为 ÷ = 225 ,那么狗的速度为 ÷ = 625
35 3 × 5 × 5 × 7 21 3 × 5 × 5 × 7
21 1
,则兔的速度为 ÷ = 441 .
25 3 × 5 × 5 × 7
3
于是狗每跑 300÷(625-225)= 单位时追上猫;
4
25
兔每跑 300÷(441-225)= 单位时追上猫.
18
 3 25  [ 3, 25] 75 75
而 ,  = = ,所以猫、狗、兔跑了 单位时,三者相遇.
 4 18  ( 4,18 ) 2 2
75 75 75
有猫跑了 ×225=8437.5 米,狗跑了 ×625=23437.5 米,兔跑了 ×441=16537.5 米.
2 2 2
75
评注:方法一、方法二中的相遇时间一个是 8437.5 单位,一个是 单位,可是答案却是一样的,
2
为什么呢?
在方法二中,如果按下面解答会得到不同答案,又是为什么?哪个方法有问题呢?自己试着解决,
并在今后的学习中避免这种错误.
1875
于是狗每跑 300÷(625-225) ×625= 米追上猫;
4
1225
兔每跑 300÷(441-225)×441= 米追上猫;
2

学而思奥数网 www.aoshu.cn Page 91 of 137


www.baoshi2011.com

1875 1225  [ 1875,1225]


而 , = ,…
 4 2  ( 4, 2)
4.一条环形道路,周长为 2 千米.甲、乙、丙 3 人从同一点同时出发,每人环行 2 周.现有自行车 2
辆,乙和丙骑自行车出发,甲步行出发,中途乙和丙下车步行,把自行车留给其他人骑.已知甲步行
的速度是每小时 5 千米,乙和丙步行的速度是每小时 4 千米,3 人骑车的速度都是每小时 20 千米.
请你设计一种走法,使 3 个人 2 辆车同时到达终点.那么环行 2 周最少要用多少分钟?

1
“1”的路程只需时间
【分析与解】 如果甲、乙、丙均始终骑车,则甲、乙、丙同时到达,单位 ;乙、丙
20
情况类似,所以先只考虑甲、乙,现在甲、乙因为步行较骑车行走单位“1”路程,耽搁的时间比为:

1 1  1 1 
 −  : −  = 3: 4
 5 20   4 20 
而他们需同时出发,同时到达,所以耽搁的时间应相等.于是步行的距离比应为耽搁时间的倒数比,
即为 4:3;因为丙的情形与乙一样,所以甲、乙、丙三者步行距离比为 4:3:3.
因为有 3 人,2 辆自行车,所以,始终有人在步行,甲、乙、丙步行路程和等于环形道路的周长.
4
于是,甲步行的距离为 2× =0.8 千米;则骑车的距离为 2×2-0.8=3.2 千米;
4 + 3+ 3
0.8 3.2
所以甲需要时间为( + )×60=19.2 分钟
5 20
环形两周的最短时间为 19.2 分钟.
参考方案如下:甲先步行 0.8 千米,再骑车 3.2 千米;
乙先骑车 2.8 千米,再步行 0.6 千米,再骑车 0.6 千米(丙留下的自行车) ;
丙先骑车 3.4 千米,再步行 0.6 千米.

5.甲、乙两项工程分别由一、二队来完成.在晴天,一队完成甲工程需要 12 天.二队完成乙工程需
要 15 天;在雨天,一队的工作效率要下降 40%,二队的工作效率要下降 10%.结果两队同时完成
这两项工程,那么在施工的日子里,雨天有多少天?

1 1 1 1 1
【分析与解】 晴天时,一队、二队的工作效率分别为 和 ,一队比二队的工作效率高 - = ;
12 15 12 15 60
1 1 1 3
雨天时,一队、二队的工作效率分别为 ×(1-40%)= 和 ×(1-10%)= ,这时二队的工作效率
12 20 15 50
3 1 1
比一队高 - = .
50 20 100
1 1 1
由 : =5:3 知,要两个队同时完工,必须是 3 个晴天,5 个雨天,而此时完成了工程的
60 100 12
1 1
×3+ ×5= ,所以,整个施工期间共有 6 个晴天,10 个雨天.
20 2
6.画展 9 时开门,但早有人来排队等候入场.从第一个观众来到时起,每分钟来的观众人数一样多.

学而思奥数网 www.aoshu.cn Page 92 of 137


www.baoshi2011.com

如果开 3 个入场口,9 时 9 分就不再有人排队;如果开 5 个入场口,9 时 5 分就没有人排队.那么第


一个观众到达的时间是 8 时几分?

【分析与解】 由题意可得两个等式,如下:
(开门前排队人数)+(9 分钟内到的人数)=3×(每个入口每分钟进的人数)×9 ①
(开门前排队人数)+(5 分钟内到的人数)=5×(每个入口每分钟进的 1 人数)×5 ②
①-② 得:4 分钟内到的人数=2×(每个人口每分钟进的人数)……③
从而有:每个入口每分钟进的人数=2×(每分钟进的人数)……④
代入②得,开门前排队人数=25×2-5=45 分钟内到的人数.
因此第一个人是 8 点 15(=60-45)分到达的.
7.甲、乙、丙 3 名搬运工同时分别在 3 个条件和工作量完全相同的仓库工作,搬完货物甲用 10 小
时,乙用 12 小时,丙用 15 小时.第二天 3 人又到两个较大的仓库搬运货物,这两个仓库的工作量也
相同.甲在 A 仓库,乙在 B 仓库,丙先帮甲后帮乙,结果干了 16 小时后同时搬运完毕.问丙在 A 仓
库做了多长时间?
【分析与解】 设第一天的每个仓库的工作量为“1”,

1 1 1 1
那么甲、乙、丙的合作工作效率为  + +  = ,第二天,甲、乙、丙始终在同时工作,所以第
 10 12 15  4
1
二天两个仓库的工作总量为 ×16=4,即第二天的每个仓库的工作总量为 4÷2=2.
4
1 8 8 2
于是甲工作了 16 小时只完成了 16× = 的工程量,剩下的 2- = 的工程量由丙帮助完成,则
10 5 5 5
2 1
丙需工作 ÷ =6(小时).
5 15
丙在 A 仓库做了 6 小时.

第 22 讲 复杂工程问题
内容概述

本讲主要讲解需运用比和比例及分段解决的较复杂问题,还有一些需借助程来求解的问题.

经典问题
1.甲、乙两个工程队修路,最终按工作量分配 8400 元工资.按两队原计划的工作效率,乙队应获

5040 元.实际从第 5 天开始,甲队的工作效率提高了 1 倍,这样甲队最终可比原计划多获得 960 元.

那么两队原计划完成修路任务要多少天?

【分析与解】 开始时甲队拿到 8400—5040=3360 元,甲乙的工资比等于甲乙的工效比,即为

3360:5040=2:3;

学而思奥数网 www.aoshu.cn Page 93 of 137


www.baoshi2011.com

甲提高工效后,甲乙的工资及工效比为

(3360+960):(5040—960)=18:17;

设甲开始的工效为“2”,那么乙的工效为“3”,设甲在提高工效后还需 x 天完成任务.

40
有(2×4+4 x ):(3×4+3 x )=18:17,化简为 216+54 x =136+68 x ,解得 x = .
7
40
于是共有工程量为 4 × 5 + 7 × = 60,
7
所以原计划 60÷(2+3)=12 天完成.

2. 规定两人轮流做一个工程,要求第一个人先做 1 个小时,第二个人接着做一个小时,然后再由第

一个人做 1 个小时,然后又由第二个人做 1 个小时,如此反复,做完为止.如果甲、乙轮流做一个工

程需要 9.8 小时,而乙、甲轮流做同样的程只需要 9.6 小时,那乙单独做这个工程需要多少小时?

【分析与解】

即甲工作 2 小时,相当与乙 1 小时.

所以,乙单独工作需 9.8 − 5 + 5 ÷ 2 = 7.3 小时.

3.甲、乙、丙三人完成一件工作,原计划按甲、乙、丙顺序每人轮流工作一天,正好整数天完成,若

1
按乙、丙、甲的顺序每人轮流工作一天,则比原计划多用 天;若按丙、甲、乙的顺序每人轮流工作一天,
2

1
则比原计划多用 天.已知甲单独完成这件工作需 10.75 天.问:甲、乙、丙一起做这件工作,完成
3
工作要用多少天?

【分析与解】 我们以甲、乙、丙各工作一天为一个周期,即 3 天一个周期.

通过上一题的类似分析,我们知道第一种情况下一定不是完整周期内完成;

但是在这题中,就有两种可能,第一种可能是完整周期+1 天,第二种可能是完整周期+2 天.

验证第一种可能不成立(详细过程略)

再看第二种可能:

学而思奥数网 www.aoshu.cn Page 94 of 137


www.baoshi2011.com

1
即丙工作 1 天,甲只需要工作 天.代入第 3 种情况知:
2

4
即甲工作 1 天,乙需要工作 天.
3
4
因为甲单独做需 10.75 天,所以工作效率为 , 于是乙工作效率为
43
4 4 3 4 1 2
÷ = , 丙工作效率为 × = .
43 3 43 43 2 43
4 3 2 9
于是,一个周期内他们完成的工程量为 + + = .
43 43 43 43

 9 9 7
则需 1 ÷  = 4 个完整周期,剩下 1 − × 4 = 的工程量;正好甲、乙各一天完成.
 43  43 43

所以第二种可能是正确的.

4 3 2 9
于是,采用第二种可能算出的数据:一个周期内他们完成的工程量: + + = .
43 43 43 43
9 43 7
需要 1 ÷ = = 4 天.
43 9 9
而甲、乙、丙合作一天完成的工程量正好是甲、乙、丙轮流做一天一个周期内的工程量.

7
于是,甲、乙、丙合作这件工程需 4 天.
9
4.如图,有一个正方体水箱,在某一个侧面相同高度的地方有三个大小相同的出水孔.用一个进水

管给空水箱灌水,若三个出水孔全关闭,则需要用 1 个小时将水箱灌满;若打开一个出水孔,则需要

学而思奥数网 www.aoshu.cn Page 95 of 137


www.baoshi2011.com

用 1 小时 5 分钟将水箱灌满;若打开两个出水孔,则需要用 72 分钟将水箱灌满.那么,若三个出水

孔全打开,则需要用多少分钟才能将水箱灌满?

【分析与解】 方法一:设打开一个出水孔时,灌满出水孔以上的部分需要时间为 x ,则不打开出

水孔和打开两个出水孔灌满水孔以上部分所需时间为 x − 5.

有工作效率之间的关系:

2 1 1 2 2x + 2
= + , 通分为 = , 化简为 x 2 + 1 = x2 + 2 x − 35, 解得 x = 35.
x x −5 x +7 x ( x − 5)( x + 7)

所以,不打开出水孔需 x − 5 = 30 分钟灌满水孔以上的水,而灌满出水孔以下的水为 60 − 30 − 30

分钟.

1 1 1
视水孔以上的水箱水量为单位“l”,有一个出水孔的工作效率为: − = .
30 35 210
1 1 2
那么打开三个出水孔的工作效率为 − 3× = .
30 210 105
2
所以,打开三个出水孔灌满整个水箱所需的时间为 30 + 1 ÷ = 82.5 分钟
105
方法二: 在打开一个出水孔时,从小孔流出的水量相当于进水管 65 − 60 = 5 分钟的进水量;在打

开两个出水孔时,从小孔流出的水量相当于进水管 72 − 60 = 12 分钟的进水量.而且注意到,后者出

水孔出水的时间比前者多 72 − 65 = 7 分钟.

因此两个出水孔 7 分钟的排水量相当于进水管 12 − 5 × 2 = 2 分钟的进水量

因此进水管 1 分钟的进水量相当于一个出水孔 7 分钟的排水量.

那么在打开一个出水孔的时候,小孔排水 5 × 7 = 35 分钟,也就是说,进水,

进水 65 − 35 = 30 分钟后,水面达到小孔高度.

1
因此打开三个出水孔的时候,灌满水箱需要 30 + 30 ÷ (1 − × 3) = 82.5 分钟.
7

第 23 讲 运用比例求解行程问题
内容概述
本讲主要讲解如何利用比例求解行程问题,而行程问题中的三个量:速度、时间、路程在某些时候存
在比例关系.
典型问题
1.甲、乙、丙三辆汽车各以一定的速度从 4 地开往 B 地.若乙比丙晚出发 10 分钟,则乙出发后 40

学而思奥数网 www.aoshu.cn Page 96 of 137


www.baoshi2011.com

分钟追上丙;若甲比乙又晚出发 20 分钟,则甲出发后 1 小时 40 分钟追上丙;那么甲出发后追上乙所


需要的时间为多少分钟?
【分析与解】 我们知道开始时,乙走了 40 分钟与丙走了 40+10=50 分钟的路程相等,所以速度比
为乙:丙=5:4;甲走了 100 分钟,丙走了 100+20+lO=130 分钟所走的路程相等,所以速度比为:
甲:丙=13:10
于是.甲:乙:丙=26:25:20.
于是,乙比甲先走 20 分钟,路程相当于 20 × 25=500,速度差相当于 26-25=l;
于是,追击时间为 500 ÷ 1=500 分钟.
2. 客车和货车分别从甲、乙两地出发相向而行.如果两车出发的时间都是 6:00,那么它们在
11:00 相遇;如果客车和货车分别于 7:00 和 8:00 出发,那么它们在 12:40 相遇.现在,客车
和货车出发的时间分别是 10:00 和 8:00,则何时它们相遇?(本题中所述的时间均为同一天,采用
24 小时制计法.)

【分析与解】 第一次,客、货各走了5 小时;第二次,客、货各走了 5 小时 40 分,4 小时 40 分,但是


两次客、货所走的路程和不变;于是有 300 客+300 货=340 客+280 货;40 客=20 货,所以客、货
1 1 1
两车的速度比为 1:2:将全程看成“1”,则客、货车速度和为 1÷5= ;所以客车速度为 ÷ 3 = ;
5 5 15

1 2 2 4
货车的速度为 × 2= ; 货 车 先 出 发 2 小 时 , 于 是 行 走 了 × 2 = ; 于 是 剩 下 的 路 程 为
15 15 15 15
4 11
1− = ;
15 15
11 1 11
还需要的时间为 ÷ = 小时,还需要 3 小时 40 分钟,在 10:00 后计时,所以相遇时间为
15 5 3
13 点 40 分.
3.在久远的古代,有一个智者叫做芝诺,他曾经说过:兔子永远追不上 10 米外的乌龟.他这样解
释:当兔子跑到 10 米处(即乌龟原来的地方),乌龟已经往前走了一点;当兔子再次到达乌龟的位置时,
乌龟又往前走了一点,……,也就说当兔子到达乌龟以前的位置时,乌龟总是往前走了一点,所以兔
子永远追不上乌龟.你认为芝诺的说法错在哪里?

【分析与解】 因为兔子的速度比乌龟快,为了方便叙述,假设兔子的速度是乌龟的 10 倍.
那么,按芝诺的说法,这些时间,乌龟走的路程为:
100
10,1,0.1,0.01,0.001,……是无穷的,而 10+1+0.1+0.01+0.001+…= ,也就是说
9
100 100
兔子只是在乌龟行走 米之前追不上.等乌龟在 米之后,兔子就在它的前面了.
9 9
在这里,芝诺用无穷个数的和来说明它们的和一定是无穷的,这显然是谬误的.

第 24 讲 应用题综合
内容概述
较为复杂的以成本与利润、溶液的浓度等为内容的分数与百分数应用题.要利用整数知识,或进行
分类讨论的综合性和差倍分问题.
典型问题
1.某店原来将一批苹果按 100%的利润(即利润是成本的 100%)定价出售.由于定价过高,无人购
买.后来不得不按 38%的利润重新定价,这样出售了其中的 40%.此时,因害怕剩余水果腐烂变质,

学而思奥数网 www.aoshu.cn Page 97 of 137


www.baoshi2011.com

不得不再次降价,售出了剩余的全部水果.结果,实际获得的总利润是原定利润的 30.2%.那么第二
次降价后的价格是原定价的百分之多少?
【分析与解】 第二次降价的利润是:
(30.2%-40%×38%)÷(1-40%)=25%,
价格是原定价的(1+25%)÷(1+100%)=62.5%.
2.某商品 76 件,出售给 33 位顾客,每位顾客最多买三件.如果买一件按原定价,买两件降价
10%,买三件降价 20%,最后结算,平均每件恰好按原定价的 85%出售.那么买三件的顾客有多少
人?

【分析与解】 3×(1-20%)+1×100%=340%=4×85%,所以 1 个买一件的与 1 个买三件的平均,


正好每件是原定价的 85%.
由于买 2 件的,每件价格是原定价的 1-10%=90%,所以将买一件的与买三件的一一配对后,仍剩
下一些买三件的人,由于
3×(2×90%)+2×(3×80%)=12×85%.
所以剩下的买三件的人数与买两件的人数的比是 2:3.
于是 33 个人可分成两种,一种每 2 人买 4 件,一种每 5 人买 12 件.共买 76 件,所以后一种
4 12 4
(76-33×
)÷( - )=25(人).
2 5 2
3
其中买二件的有:25× =15(人).
5
前一种有 33-25=8(人),其中买一件的有 8÷2=4(人).
于是买三件的有 33-15-4=14(人).
3.甲容器中有纯酒精 11 立方分米,乙容器中有水 15 立方分米.第一次将甲容器中的一部分纯酒精
倒入乙容器,使酒精与水混合;第二次将乙容器中的一部分混合液倒人甲容器.这样甲容器中的纯酒
精含量为 62.5%,乙容器中的纯酒精含量为 25%.那么,第二次从乙容器倒入甲容器的混合液是多
少立方分米?

【分析与解】 设最后甲容器有溶液 x 立方分米,那么乙容器有溶液(11+15- x )立方分米.


有 62.5%× x +25%×(26- x )=11,解得 x =12,即最后甲容器有溶液 12 立方分米,乙容器则有
溶液 26-12=14 立方分米.
而第二次操作是将乙容器内溶液倒入甲容器中,所以乙溶液在第二次操作的前后浓度不变,那么在
第二次操作前,即第一次操作后,乙容器内含有水 15 立方分米,则乙容器内溶液 15÷(1-25%):20
立方分米.
而乙容器最后只含有 14 立方分米的溶液,较第二次操作前减少了 20-14=6 立方分米,这 6 立方分
米倒给了甲容器.
即第二次从乙容器倒入甲容器的混合液是 6 立方分米.
4.1994 年我国粮食总产量达到 4500 亿千克,年人均 375 千克.据估测,我国现有耕地 1.39 亿
公顷,其中约有一半为山地、丘陵.平原地区平均产量已超过每公顷 4000 千克,若按现有的潜力,到
2030 年使平原地区产量增产七成,并使山地、丘陵地区产量增加二成是很有把握的.同时在 20 世纪
末把我国人口总数控制在 12.7 亿以内,且在 21 世纪保持人口每年的自然增长率低于千分之九或每十
年自然增长率不超过 10%.请问:到 2030 年我国粮食产量能超过年人均 400 千克吗? 试简要说明
理由.

【分析与解】 山地、丘陵地区耕地为1.39÷2≈0.70 亿公顷,那么平原地区耕地为 1.39-0.70=0.69


亿公顷,因此平原地区耕地到 2030 年产量为:4000×0.69×1.7=4692(亿千克);
山地、丘陵地区的产量为:(4500-4000×0.69)×1.2=2088(亿千克);

学而思奥数网 www.aoshu.cn Page 98 of 137


www.baoshi2011.com

粮食总产量为 4692+2088=6780(亿千克).
而人口不超过 12.7×1.13≈16.9(亿),按年人均 400 千克计算.共需 400×16.9=6760(亿千克).
所以,完全可以自给自足.
5.要生产基种产品 100 吨,需用 A 种原料 200 吨,B 种原料 200.5 吨,或 C 种原料 195.5 吨,或
D 种原料 192 吨,或 E 种原料 180 吨.现知用 A 种原料及另外一种(指 B,C,D,E 中的一种)原料
共 19 吨生产此种产品 10 吨.试分析所用另外一种原料是哪一种,这两种原料各用了多少吨?

【分析与解】 我们知道题中情况下,生产产品 100 吨,需原料 190 吨。


生产产品 100 吨,需 A 种原料 200 吨,200 > 190,所以剩下的另一种原料应是生产 100 吨,需
原料小于 190 吨的,B、C、D、E 中只有 E 是生产 100 吨产品。只需 180 吨(180 < 190),所以另一种原
料为 E,
设 A 原料用了 x 吨,那么 E 原料用了 19- x 吨,即可生产产品 10 吨:
100 100
x× +(19- x )× =10,解得 x =10.
200 180
即 A 原料用了 10 吨,而 E 原料用了 19-10=9 吨.
6.有 4 位朋友的体重都是整千克数,他们两两合称体重,共称了 5 次,称得的千克数分别是
99,113,125,130,144.其中有两人没有一起称过,那么这两个人中体重较重的人的体重是多
少千克?

【分析与解】 在已称出的五个数中,其中有两队之和,恰好是四人体重之和是 243 千克,因此没有称


过的两人体重之和为 243-125=118(千克).
设四人的体重从小到大排列是 a 、 b 、 c 、 d ,那么一定是 a + b =99, a + c :=113.
因为有两种可能情况: a + d =118, b + c =125;
或 b + c =118. a + d =125.
因为 99 与 113 都是奇数, b =99- a , c =113- a ,所以 b 与 c 都是奇数,或者 b 与 c 都是偶数,
于是 b + c 一定是偶数,这样就确定了 b + c =118.
a 、 b 、 c 三数之和为:(99+113+118)÷2=165.
b 、 c 中较重的人体重是 c ,
c =( a + b + c )-( a + b )=165-99=66(千克).
没有一起称过的两人中,较重者的体重是 66 千克.
补充选讲问题
1、A、B、C 四个整数,满足 A+B+C=2001,而且 1<A<B<C,这四个整数两两求和得到六个数,
把这 6 个数按从小到大排列起来,恰好构成一个等差数列
请问:A、B、C 分别为多少?

【试题分析】 我们注意到:
①1+A<1+B<1+C<A+B<A+C<B+C
②1+A<1+B<A+B<1+C<A+C<B+C 这两种情况有可能成立.
先看① 1+A<l+B<l+C<A+B<A+C<B+C

(A-1):(B-1):(C-1)=2:3:4,A+B+C=2001
A-1+B-l+C-1=1998.

学而思奥数网 www.aoshu.cn Page 99 of 137


www.baoshi2011.com

2
于是 A-l=1998× =444,A=444+1=445;
2+3+ 4
3 4
B=1998× +l=667;C=1998× +l=889.
2+3+ 4 2+3+ 4
再看② l+A<l+B<A+B<1+C<A+C<B+C

(A-1):(B-1):(C-1)=1:2:4,A+B+C=2001.
A-1+B-1+C-1=1998.
1
于是 A-1=1998× ,A 不是整数,所以不满足.
1+ 2 + 4
于是 A 为 445,B 为 667,C 为 889.
7.甲、乙两人参加同一场考试,又同时在上午 10 点离开考场,同时午饭.但甲说:“我是在午饭前 2
小时与考试开始后 1.5 小时这两个时间中较早的一个时间离开考场的.”乙说:“我是在午饭前 2.5 小
时与考试后 1 小时这两个时间中较晚的一个时间离开考场的”.求考试开始和午饭开始的时间.

【分析与解】 由题中条件知,午饭前 2 小时,考试开始后 1.5 小时,早者为 10 点;于是,有两种情


况:
第一种情况:午饭开始前 2 小时较早,为 10 点,有午饭(10+2=)12 点开始,
而考试开始后 1.5 小时应超过 10 时,即考试开始的时间在 8 点 30 分以后;
那么午饭前 2.5 小时为 12-2.5 为 9 点 30 分,而考试开始后 1 小时在 9 点 30 分后,所以,晚者为
考试开始后 1 小时,为 10 点,所以 10-1=9 点开始考试的;
第二种情况:考试开始后 1.5 小时较早,为 10 点,有 10-1.5 为 8 点 30 分开始考试,午饭前 2 小
时超过 10 点,则午饭应在 12 点以后;
那么午饭前 2.5 小时应在 9 点 30 分之后,而考试后 1 小时为 9 点 30 分,有午饭前 2.5 小时为晚
者,为 10 点,所以午饭是在 10+2.5 即 12 点 30 分开始的.
综合这两种情况,有下表

第 25 讲 数论综合 2
内容概述
进位制的概念、四则运算法则及整数在不同进位制之间的转化,利用恰当的进位制解数论问题.取整
符号[]与取小数部分符号{}的定义与基本性质,包含这两种符号的算式与方程的求解.两次与分式不
定方程,不便直接转化为不定方程的数论问题.各种数论证明题.
典型问题
1.算式 1534×25=43214 是几进位制数的乘法?

【分析与解】 注意到尾数,在足够大的进位制中有乘积的个位数字为 4×5=20,但是现在为 4,说明


进走 20-4=16,所以进位制为 16 的约数:16、8、4、2.

学而思奥数网 www.aoshu.cn Page 100 of 137


www.baoshi2011.com

因为原式中有数字 5,所以不可能为 4,2 进位,而在十进制中有 1534×25=38350<43214,


所以在原式中不到 10 就有进位,即进位制小于 10,于是原式为 8 进制.
2.求方程 19[x]-96{x}=0 的解的个数.

【分析与解】 有{x}为一个数的小数部分,显然小于 1,则 96{x}小于 96,而 19[x]=96{x},所以


96
19[x]小于 96,即[x]小于,又[x]为整数,所以[x]可以取 0,1,2,3,4,5,对应有 6 组解.
19
96 19 19 19 95
进一步计算有 0,1 , 2 ,,,
3 4 5 为原方程的解.
19 48 32 24 96
3.一个自然数与自身相乘的结果称为完全平方数.已知一个完全平方数是四位数,且各位数字均
小于 7.如果把组成它的数字都加上 3,便得到另外一个完全平方数.求原来的四位数.

【分析与解】 设这个四位数为 abcd = m 2 ………………………………… ①

每位数字均加 3,并且没有进位,为

(a+3)(b+3)(c+3)(d+3) = n 2 …………………………………………………②
有②-①得:3333= n 2 − m 2 =(n-m)(n+m) ………………………………③
将 3333 分解质因数,有 3333=3×11×101,其有(1+1)(1+1)(1+1)=8 个约数,但是有 n+m
>n-m,所以只有 4 种可能满足题意,一一考察,如下表:

如上表,只有 1156,4489 满足,即原来这个四位数为 1156.


1
4.将 表示成两个自然数的倒数之和,请给出所有的答案.
6
1 1 1
【分析与解】 设有 + = ,化简有(a-6)(b-6)=6 2 =2×2×3×3,
a b 6

1 1 1
评注:形如 + = (t 为己知常数)的解法及解的个数.
A B t

学而思奥数网 www.aoshu.cn Page 101 of 137


www.baoshi2011.com

1 1 1
+ = (t 为已知常数)类问题,可以通过计算,转化为(A-t)×(B-t)= t 2 ;
A B t
我们 t 2 将分解质因数后,再令(A-t)其中一个为 t 2 的一个约数(A-t)=a,那么 A=a+t,则

t2
B= + t (t 为已知常数),
a

A = a + t

所以,一般公式为  t2 (a 为 t 的一个约数);
B = + t
 a
x+1
设 t 2 的约数有 x 个,则 A、B 有 组(调换顺序算一种).
2
 A = 2t
注意有一组解 A、B 相等,就是  .
 B = 2t
5.在给定的圆周上有 2000 个点.任取一点标上数 1;按顺时针方向从标有 1 的点往后数 2 个点,
在第 2 个点上标上数 2;从标有 2 的点再往后数 3 个点,在第 3 个点上标上数 3;……;依此类推,
直至在圆周上标出 1993.对于圆周上的这些点,有的点可能标上多个数,有的点可能没有被标数.
问标有数 1993 的那个点上标的最小数是多少?

【分析与解】 记标有 1 为第 1 号,序号顺时针的依次增大.当超过一圈时,编号仍然依次增加,如 1


号也是 2001 号,4001 号,……
则标有 2 的是 1+2 号,标有 3 的是 1+2+3 号,标有 4 的是 1+2+3+4 , … ,标有 1993 的是
1+2+3+…+1993=1987021 号.
1987021 除以 2000 的余数为 1021,即圆周上的第 1021 个点标为 1993.
k(k + 1)
那么 1021+2000n=1+2+3+…+k= ,即 2042+4000n=k(k+1).
2
当 n=0 时,k(k+1)=2042,无整数解;
当 n=1 时,k(k+1)=6042,无整数解;
当 n=2 时,k(k+1)=10042,无整数解;
当 n=3 时,k(k+1)=14042,有 118×119=14042,此时标有 118;
随着 n 的增大,k 也增大.
所以,标有 1993 的那个点上标出的最小数为 118.
1
6.有些三位数,如果它本身增加 3,那么新的三位数的各位数字的和就减少到原来三位数的 .求
3
所有这样的三位数.

【分析与解】 设这个三位数为 abc ,数字和为 a+b+c,如果没有进位,那么 abc + 3 = ab(c + 3) ,

显然数字和增加了 3,不满足,所以一定有进位,
1
则 abc +3= a(b + 1)(c + 3 − 10) , 数 字 和 为 0+(b+1)+(c+3 - 10)= (a + b + c) , 则
3
a+b+c=9,而 c+3 必须有进位,所以 c 只能为 7,8,9.
一一验,如下表:

学而思奥数网 www.aoshu.cn Page 102 of 137


www.baoshi2011.com

验证当十位进位及十位、个位均进位时不满足.
所以,原来的三位数为 207,117 或 108.
7.将某个 17 位数各位数字的排列顺序颠倒,再将得到的新数与原来的数相加.试说明,所得的
和中至少有一个数字是偶数.

【分析与解】 先假设和的各位数字全是奇数,设这个 17 位数为 abL cd ,则 a+d 为奇数,b+c 的

和小于 10,于是十位不向前进位,从而去掉前后各两个两位数字所得的 13 位数仍具有题述性质.


依次类推 6 次后,得到一位数,它与自身相加的和的个位数字必是偶数,矛盾.
即开始的假设不正确,所以和中至少有一个数字是偶数.

第 26 讲 进位制问题
内容概述
本讲不着重讨论 n 进制中运算问题,我们是关心 n 这个数字,即为几进制.对于进位制我们要注
意本质是: n 进制就是逢 n 进一.
但是,作为数论的一部分,具体到每道题则其方法还是较复杂的.
说明:在本讲中的数字,不特加说明,均为十进制.
典型问题
1.在几进制中有 4×13=100.
【分析与解】 我们利用尾数分析来求解这个问题:

不管在几进制均有(4) 10 ×(3) 10 =(12) 10 .但是,式中为 100,尾数为 0.

也就是说已经将 12 全部进到上一位.
所以说进位制 n 为 12 的约数,也就是 12,6,4,3,2.
但是出现了 4,所以不可能是 4,3,2 进制.

我们知道(4) 10 ×(13) 10 =(52) 10 ,因 52 < 100,也就是说不到 10 就已经进位,才能是 100,于

是我们知道 n <10.
所以, n 只能是 6.
2.在三进制中的数 12120120110110121121,则将其改写为九进制,其从左向右数第 l 位数字是
几?
【分析与解】 我们如果通过十进制来将三进制转化为九进制,那运算量很大.

学而思奥数网 www.aoshu.cn Page 103 of 137


www.baoshi2011.com

注意到,三进制进动两位则我们注意到进动了 3 个 3,于是为 9.所以变为遇 9 进 1.也就是九进


制.
于是,两个数一组,两个数一组,每两个数改写为九进制,如下表:
12 12 0l 20 11 01 10 12 11 21 3 进制
5 5 l 6 4 1 3 5 4 7 9 进制
所以,首位为 5.
评注:若原为 n 进制的数,转化为 n k 进制,则从右往左数每 k 个数一组化为 n k 进制.
如:2 进制转化为 8 进制,2 3 =8,则从右往左数每 3 个数一组化为 8 进制.
10 100 001 101 2 进制
2 4 1 5 8 进制

(10100001101) 2 =(2415) 8 .

3.在 6 进制中有三位数 abc ,化为 9 进制为 cba ,求这个三位数在十进制中为多少?

【分析与解】 ( abc ) 6 = a ×62+ b ×6+ c =36 a +6 b + c ;

( cba ) 9 = c ×92+ b ×9+ a =81 c +9 b + a .

所以 36 a +6 b + c =81 c +9 b + a ;于是 35 a =3b+80 c ;


因为 35 a 是 5 的倍数,80 c 也是 5 的倍数.所以 3 b 也必须是 5 的倍数,又(3,5)=1.
所以, b =0 或 5.
① 当 b =0,则 35 a =80 c ;则 7 a =16 c ;(7,16)=1,并且 a 、 c ≠0,所以 a =16, c =7:
但是在 6,9 进制,不可以有一个数字为 16.
② 当 b =5,则 35 a =3×5+80 c ;则 7 a =3+16 c ;mod 7 后,3+2 c ≡0
所以 c =2 或者 2+7 k ( k 为整数).因为有 6 进制,所以不可能有 9 或者 9 以上的数,于是 c =2.
于是,35 a =15+80×2; a =5.

于是( abc ) 6 =(552) 6 =5×62+5×6+2=212.

所以.这个三位数在十进制中为 212.

4.设 1987 可以在 b 进制中写成三位数 xyz ,且 x + y + z =1+9+8+7,试确定出所有可能的 x 、 y 、

z 及b .
( xyz )b = b 2 x + by + z = 1987 ①
【分析与解】 我们注意 
 x + y + z = 1+ 9 + 8 + 7 ②

①-② 得:( b 2 -1) x +( b -1) y =1987-25.

则( b -1)( b +1) x +( b -1) y =1962,


即( b -1)[( b +1) x + y ]=1962.
所以,1962 是( b -1)的倍数.
1962=2×9×109:
当 b -1=9 时, b =10,显然不满足;
当 b -1=18 时 , b =19 , 则 ( b -1)[( b +1) x + y ]=18×(20 x + y )=1962 ; 则 20 x + y
=109,

学而思奥数网 www.aoshu.cn Page 104 of 137


www.baoshi2011.com

 b=19
x=5
x = 5  x = 4 
所以,  , (不满足) , . . . . . .则 
 y = 9  y = 29 y=9
 z = 11
显然,当 b =109 不满足, b =2×109 不满足,当 b =9×109 也不满足.

于是为(59B) 19 =(1987) 10 ,B 代表 11.

5.下面加法算式中不同字母代表不同的数字,试判定下面算式是什么进制,A、B、C、D 的和为多少?
【分析与解】
于是,我们知道 n =4,所以为 4 进制,

则 A+B+C+D=3+1+2+0=6.
6. 一 个 非零自然数, 如果它的二进制表示中数码 l 的个数是
偶数,则称之为“坏数”.例如:18=(10010)2 是“坏数”.试求小于 1024 的所有坏数的个数.
【分析与解】 我们现把 1024 转化为二进制:

(1024) 10 =2 10 =(10000000000)2.

于是,在二进制中为 11 位数,但是我们只用看 10 位数中情况.


     
并且,我们把不足 10 位数的在前面补上 0,如  111...10000...0
1 2 3 14 2 43   111...1
1 2 3  =  0111...1
14 2 43  则,
 5个1个或以上
5 0 2  9 个1 2  9 个1 2

 
 *1*4* 4*4*2*4* 4* 4
*3*  可以含 2 个 l,4 个 1,6 个 1,8 个 l,10 个 1.
 10个位置 

于是为 C10 + C10 + C10 + C10 + C10


2 2 6 8 10

10 × 9 10 × 9 × 8 × 7 10 × 9 × 8 × 7 × 6 × 5 10 × 9 × 8 × 7 × 6 × 5 × 4 × 3
= ++++ 1
2 1× 2 × 3 × 4 1× 2 × 3 × 4 × 5 × 6 1× 2 × 3 × 4 × 5 × 6 × 7 × 8
=45+210+210+45+1=511
于是,小于 1024 的“坏数”有 511 个.

 
1×43 ×23...
7.计算:  3
 4 ×33 − 1 26 的余数.

 2003个 3 
【分析与解】

学而思奥数网 www.aoshu.cn Page 105 of 137


www.baoshi2011.com

     
 31×43 ×23... × −  1000... × 0 − 1  222...2
43 3 1  =
  14 2 43   1 2 3 
= 
 2003个3   2003个3 3  2003个2 3

   
1×43 ×23...
26=(222) 3 所以,  3
 4 ×33 − 1 ÷26=  222...2
1 2 3  ÷(222) 3

 2003个 3   2003个2 3

(222) 3 整除(222) 3 ,2003÷3:667……2,所以余(22) 3 =8.

所以余数为 8.
8.一个 10 进制的三位数,把它分别化为 9 进制和 8 进制数后,就又得到了 2 个三位数.老师发现这
3 个三位数的最高位数字恰好是 3、4、5,那这样的三位数一共有多少个?

【分析与解】 我们设(3 ab ) 10 =(4 cd ) 9 =(5 ef ) 8 ;

我们知道(4 cd ) 9 在(400) 9 ~(488) 9 之间,也就是 4×92~5×92-1,也就是 324~406;

还知道(5 ef ) 8 在(500) 8 ~(577) 8 之间,也就是 5×82~6×82-1,也就是 320~383;

又知道(3 ab ) 10 在(300) 10 ~(399) 10 之间.

所以,这样的三位数应该在 324~383 之间,于是有 383-324+1=60 个三位数满足条件.


9. 一 袋花生共有 2004 颗,一只猴子第一天拿走一颗花生,从第二天起 ,每天拿走的都是以前各天的
总和.
① 如果直到最后剩下的不足以一次拿走时却一次拿走,共需多少天?
② 如果到某天袋里的花生少于已拿走的总数时,这一天它又重新拿走一颗开始,按原规律进行新的
一轮.如此继续,那么这袋花生被猴子拿光的时候是第几天?
【分析与解】 ①我们注意到

每天 1 2 3 4 8 16 32 64 …
前若干天的和 …

2 10 <2004<2 11 前 1 天为 1,前 2 天为 21,前 3 天是 22,所以前 11 天为 2 10 ,前 12 天是 2 11 ,


也就是说不够第 11 天拿的,但是根据题中条件知.所以共需 12 天.


每天 1 1 2 4 8 16 32 64 …
前若干天的和 1 2 4 8 16 32 64 128 …
改写为 2 进制 1 10 100 100 1000 1000 10000 100000 …
0 0 00 00 00

2004=(11111010100)2,
(10+1)+(9+1)+(8+1)+(7+1)+(6+1)+(4+1)+(2+1)=11+10+9+8+7+5+3=53 天.

学而思奥数网 www.aoshu.cn Page 106 of 137


www.baoshi2011.com

第 27 讲 整取问题
内容概述
有时我们只关心某数的整数部分,于是我们就有了取整问题,如在抽屉原理里,在不定方程里等一
些数论问题中.
我们规定[ x ]表示不超过 x 的最大整数,{ x }= x -[ x ],即为 x 的小数或真分数部分.
如[3.14]=3,{3.14}=0.14,
显然有{x}<1.
O≤{x}+{y}<2( x 、y 均为整数时等号才成立).
典型问题
1981×1  1981× 2  1981× 2005  1981× 2006 
2.求  + + ... +  2006  +  2006  的和.
 2006   2006 
【分析与解】 我们知道如果直接求解是无法解出的,现在试着观察规律:

1981× 1  1981× 2005  1981× 1 1981× 1 


最 后 一 项 为 1981 不 难 得 到 , 再 看  + ; = +
 2006   2006  2006  2006 

1981×1 
 
 2006 

1981× 2005 1981× 2005  1981× 2005 


= + 
2006  2006   2006 

1981× 1 1981× 2005 1981×1  1981×1  1981× 2005 


所 以 有 + =1981=  + + +
2006 2006  2006   2006   2006 

1981× 2005 
 
 2006 

1981× 1  1981× 2005  1981×1  1981× 2005 


= + + + 
 2006   2006   2006   2006 

1981× 1  1981× 2005 


因为  + 的和为整数,
 2006   2006 

1981×1  1981× 2005 


所以  +  的和也为整数,但是我们知道 0≤{ x }+{y}<2;在此题中显
 2006   2006 

1981×1  1981× 2005 


然≠0,所以  +  =1
 2006   2006 

学而思奥数网 www.aoshu.cn Page 107 of 137


www.baoshi2011.com

1981× 1  1981× 2005 


于是  + =1981-1=1980;
 2006   2006 
这样,我们就找到了一般规律,我们知道原式除了最后一项,还有 2005 项,于是有 1002 组和

1981× 1003 
 2006  =990;

所以为 1002×1980+990+1981=1986931.
4.解方程[ x ]{ x }+ x =2{ x }+10
【分析与解】 我们注意到 x 不超过 10, x 不能小于 5;
所以当[ x ]=5,6,7,8,9,10 的时候我们分别计算小数部分{ x }
当[ x ]=5 时,有 5{ x }+5+{ x }=2{ x }+10;则 4{ x }=5,{ x }>1,不满足;
4
当[ x ]=6 时,有 6{ x }+6+{ x }=2{ x }+10;则 5{ x }=4,{ x }= ;
5
1
当[ x ]=7 时,有 7{ x }+7+{ x }=2{ x }+10;则 6{x}=3,{ x }= ;
2
2
当[ x ]=8 时,有 8{ x }+8+{ x }=2{ x }+10;则 7{ x }=2,{ x }= ;
7
1
当[ x ]=9 时,有 9{ x }+9+{ x }=2{ x }+10;则 8{ x }=1,{ x }= ;
8
当[ x ]=10 时,有 10{ x }+10+{ x }=2{ x }+10;则 9{ x }=0,{ x }=0.
4 1 2 1
所以有 x =6 ,7 ,8 ,9 ,10.
5 2 7 8
 19   20   21   91 
6. r 满足  r + + r + + r + + ... + +
 100  =546.求[100 r ]的值?
r
 100   100   100 
19
【分析与解】 显然等式的左边有 91-19+1=73 项,每项值为[ r ]或[ r +1],这是因为: 、
100
20 91
、…、 均小于 l,
100 100
又由于 73×7< 546 <73×8,为使和数为 546,则[ r ]=7,
x
则设有 t 个[ r + ]值为 7,于是,7× t +8×(73- t )=546,
100
解得 t =38.
所以有 38 项整数部分为 7.
19 + 38 − 1 56
即: r + <8,即 r + <8.
100 100
19 + 38 57
r+ ≥8,即 r + ≥8
100 100
56
于是,100[ r + ]<8×100.
100
100 r +56<800,100 r <744;100 r +57≥800,100 r ≥743.
于是,[100 r ]=743

学而思奥数网 www.aoshu.cn Page 108 of 137


www.baoshi2011.com

第 28 讲 数论综合 3
内容概述
具有相当难度,需要灵活运用各种整数知识,或与其他方面内容相综合的数论同题.
典型问题
2. 有 3 个自然数,其中每一个数都不能被另外两个数整除,而其中任意两个数的乘积却能被第三个数
整除.那么这样的 3 个自然数的和的最小值是多少?
【分析与解】 设这三个自然数为 A,B,C,且 A= a × b ,B= b × c ,C= c × a ,当 a 、b 、c 均是质
数时显然满足题意,为了使 A,B,C 的和最小,则质数 a 、b 、c 应尽可能的取较小值,显然当 a 、b 、
c 为 2、3、5 时最小,有 A=2×3=6, B=3×5=15,C=5×2=10.
于是,满足这样的 3 个自然数的和的最小值是 6+15+10=31.
4. 对于两个不同的整数,如果它们的积能被和整除,就称为一对 “ 好数 ” ,例如 70 与 30.那么在
1,2,…,16 这 16 个整数中,有“好数”多少对?
1 1 1
【分析与解】 设这两个数为 a 、 b ,且 a < b ,有 a b = k ×( a + b ),即 + = .
a b k
1 1 1 a = 3 a = 4
当 k =2 时,有 + = ,即( a -2)×( b -2)=22=4,有  , ,但是要求 a ≠ b .所以
a b 2 b = 6 b = 4

a = 3
只有  满足;
b = 6

1 1 1  a = 4 a = 6
当 k =3 时,有 + = ,即( a -3)×( b -3)=32=9,有  , ,但是要求 a ≠ b .所以只
a b 3 b = 12 b = 6

a=4
有 满足;
b = 12
……
逐个验证 k 的值,“好数”对有 3 与 6,4 与 12,6 与 12,10 与 15.所以“好数”对有 4 个.
6. 甲、乙两人进行下面的游戏:两人先约定一个自然数 N,然后由甲开始,轮流把
0,1,2,3,4,5,6,7,8,9 这 10 个数字中的一个填入图 28-1 的某个方格中,每一方格只能
填一个数字,但各方格所填的数字可以重复.当 6 个方格都填有数字后,就形成一个六位数.如果这
个六位数能被 N 整除,那么乙获胜;如果这个六位数不能被 N 整除,那么甲获胜.设 N 小于 15,问
当 N 取哪几个数时.乙能取胜?

【分析与解】 当 N 取 2,4,6,8,10,12,14 这 7 个偶数时,当甲将某个奇数放到最右边的方


格中,则这个六位数一定是奇数,奇数显然不能被偶数整除,所以此时乙无法取胜;
而当 N 取 5 时,当甲在最右边的方格内填人一个非 0 非 5 的数字时,则这个六位数一定不能被 5
整除,所以此时乙无法获胜:
此时还剩下 1,3,7,9,11,13 这 6 个数,
显然当 N 取 l 时,乙一定获胜;

学而思奥数网 www.aoshu.cn Page 109 of 137


www.baoshi2011.com

当 N 取 3 或 9 时,只要数字对应是 3 或 9 的倍数时,这个六位数就能被对应的 3 或 9 整除,显然


乙可以做到;
当 N 取 7,1l 或 13 时,只要前三位数字和与后三位数字和的差对应是 7,11,13 的倍数时,这
个六位数就对应是 7,11,13 的倍数,乙可以做到.
于是,当 N 取 1,3,7,9,11,13 时,乙适当的操作能保证自己一定获胜.
8. 已知 a 与 b 的最大公约数是 12, a 与 c 的最小公倍数是 300, b 与 c 的最小公倍数也是 300.那
么满足上述条件的自然数 a , b , c 共有多少组?

【分析与解】 300=12× 52 ,是 a 、b 的倍数,而 12 是 a 、b 的最大公约数,所以 a 、b 有 5 种可能,

a 12 12×5 12× 52 12 12

b 12 12 12 12×5 12× 52

由于 a 、b 中总有一个为 12,则 c = 2 x × 3 y × 5 z ,其中 x 可以取 0、1、2 中的任意一个,y 可以取

0、1 中的任意一个,这样满足条件的自然数 a 、 b 、 c 共有 5×3×2=30 组.


10.圆周上放有 N 枚棋子,如图 28-2 所示,B 点的那枚棋子紧邻 A 点的棋子.小洪首先拿走 B 点处
的 1 枚棋子,然后沿顺时针方向每隔 1 枚拿走 2 枚棋子,这样连续转了 10 周,9 次越过 A.当将要第
10 次越过 A 处棋子取走其他棋子时,小洪发现圆周上余下 20 多枚棋子.若 N 是 14 的倍数,请精确
算出圆周上现在还有多少枚棋子?

【 分 析 与 解 】 设圆周上余 a 枚棋 子,从第 9 次越过 A 处拿走 2 枚棋子到第


10 次将要越过 A 处棋子时,小洪拿了 2 a 枚棋子,所以在第 9 次将要越过 A 处棋
子时,圆周上有 3 a 枚棋子. .
依次类推,在第 8 次将要越过 A 处棋子时,圆周上有 32 a 枚棋子,…,在第 1 次将要越过 A 处棋
子时,圆周上有 3 9 a 枚棋子,在第 1 次将要越过 A 处棋子之间,小洪拿走了 2(3 9 a -1)+枚棋子,所
以 N=2(3 9 a -1)+1+3 9 a =310 a -1.
N=310 a -1=59049 a -l 是 14 的 倍 数 , N 是 2 和 7 的 公 倍 数 , 所 以 a 必 须 是 奇 数 ; 又
N=(7×8435+4) a -1=7×8435 a +4 a -1,所以 4 a -1 必须是 7 的倍数.
当 a =21,25,27,29 时,4 a -1 不是 7 的倍数,当 a =23 时,4 a -1=91=7×13,是 7 的倍数.
所以.圆周上还有 23 枚棋子.
12. 是否存在一个六位数 A,使得 A,2A,3A,…,500000A 中任意一个数的末尾 6 个数码不全
相同?
【分析与解】 显然 A 的个位数字不能为偶数,不然 500,000A 的后 6 位为 000,000;
而 A 的个位数字也不能为 5,不然 200,000A 的后 6 位为 000,000.
于是 A 的个位数字只能为 1,3,7,9.

对 于 任 何 一 个 六 位 数 A( 个 位 数 字 为 1 , 3 , 7 , 9) , 均 存 在 六 位 数 t = abcdef , 使 得 t

×A≡111,111(mod 1,000,000).

如果存在 t = abcdef >500,000,使得 t ×A≡111,111 (mod 1,000,000),那么那个 A 即为题中

所求的值.(说明见评注)

学而思奥数网 www.aoshu.cn Page 110 of 137


www.baoshi2011.com

当 t =999,999,有 A=888,889 时, t A=888,888,111,111,显然满足上面的条件.


所以 888,889 即为所求的 A.

评 注 : 如果存在 t = abcdef >500,000,使得 t ×A≡111,111(mod 1,000,000),那么

那个 A 即为题中所求的值.
这是因为如果对于上面的 A,还存在一个六位数 B,使得 B×A=111,111(mod 1,000,000),
那么有( t ×A-B ×A)=0(mod 1,000,000),即( t -B)×A≡0(mod 1,000,000).因为 A 不含有
质因数 2、5,所以( t -B)为 1,000,000 的倍数, t -B≥1,000,000,那么 t >1,000,000,与
t 为六位数矛盾.
也就是说不存在小于等于 500,000 的 t,使得 t A 的后六位为 111,111,那么也不可能使得 t A
的后 6 位相同.
14.已知 m,n,k 为自然数,m ≥ n ≥k, n 2 m +2 n -2 k 是 100 的倍数,求 m + n - k 后的最小值.
【分析与解】 方法一:首先注意到 100=22×52.
如果 n=k,那么 2m 是 100 的倍数,因而是 5 的倍数,这是不可能的.所以 n-k≥1.

2m +2n -2k =2k (2m-k +2n-k +1) 被 22 整除,所以 k≥2.


设 a =m-k, b =n-k,则 a ≥ b ,且都是整数.
2a+2b-1 被 52 整除,要求 a + b +k=m+n-k 的最小值.
不难看出 210+21-1=1025,能被 25 整除,所以 a + b +k 的最小值小于 10+l+2=13.
而且在 a =10, b =1,k=2 时,上式等号成立.
还需证明在 a + b ≤10 时,2a+2b-l 不可能被 25 整除.
有下表

a 9 8 7 6 5 4
b 1 1,2 1,2,3 1,2,3,4 1,2,3,4,5 1,2,3,4

a ≤3 时 , 2a+2b-1<8+8=16 不 能 被 52 整 除 . 其 他 表 中 情 况 , 不 难 逐 一 检 验 , 均 不 满 足
2a +2b -1 被 25 整除的要求.
因此 a + b -k 即 m+n-k 的最小值是 13.

方法二:注意到有 100=2×2×5×5,4∣ (2m +2n -2k ) .

2m +2n -2k =2k (2m-k +2n-k -1)因为2m-k +2n-k -l, 所以 k 最小为 2.

还有 25∣ (2m-k +2n-k -1) ,令 m-k=x, n-k=y

则有 2 x +2 y ≡l(mod 25)
因为 5 去除 2,22,23,24,25 余数分别为 2,4,3,1,2;余数是 4 个一周期 .于是,
x=4p+2,y=4q+1;
或者是 x=4P+3,y=4Q+3.
(1)x=4p+2,y=4q+1 时

当 x=2,y=1,于是 2m +2n -2k =24+23-22=20 不是 100 的倍数;

当 x=6,y=l,于是 2m +2n -2k =28+23-22=260 不是 100 的倍数;

学而思奥数网 www.aoshu.cn Page 111 of 137


www.baoshi2011.com

当 x=10,y=l,于是 2m +2n -2k =212+23-22=4100 是 l00 的倍数;

(2)x=4P+3,y=4Q+3

当 x=3,y=3,于是 2m +2n -2k =25+25-22=60 不是 l00 的倍数;

当 x=7,y=3,于是 2m +2n -2k =29+25-22=540 不是 l00 的倍数:

其余的将超过(1)种情况,所以,最小为 m+n-k=12+3-2=13.

第 29 讲 数论综合 4
内容概述
主要是“小升初”综合素质测试中较难的数论问题.

1.任意选取 9 个连续的正整数,即它们的乘积为 P,最小公倍数为 Q.我们知道,P 除以 Q 所得到的


商必定是自然数,那么这个商的最大可能值是多少?
【分析与解】 将 9 个连续的正整数作因式分解,如果某个质数是其中至少两个分解式的因子,那么
次数最高的那个方幂会包含在最小公倍数 Q 中,而其他方幂的乘积则出现在 P 除以 Q 的商中.显然这
样的质数必定小于 9,只可能是 2,3,5 或 7.
记 P÷Q=R,则 R 的质因数必定取自 2,3,5,7.
两个不同的 7 的倍数至少相差 7,因此在 9 个连续正整数中,最多有两个数含有质因数 7.当有两
个数是 7 的倍数是,可能它们都不能被 7×7 整除,也可能其中一个数是 7×7 的倍数,而另一个不是.
于是 R 的质因数分解式中 7 的幂次最高是 1.
类似的分析,R 中最多包含一个质因数 5.
在 9 个连续的正整数中,恰有 3 个数是 3 的倍数,其中一个数能被 9 整除,而另一两个数仅能被 3
整除,因此 R 中所包含的质因数 3 的幂次必定为 2.
在 9 个连续的正整数中,最多有 5 个数是偶数.此时,除去含有 2 的幂次最高的数外,其余的 4 的
数含有质因数 2 最多的情形是:其中有 2 个仅为 2 的倍数,有 1 个是 4 的倍数,另一个是 8 的倍数.
即 R 的质因数分解式中 2 的幂次最多是 1+1+2+3=7.
综上所述,R 的最大值是 27×32×5×7=40320.事实上,对于 9 个连续正整数 560,561,
…,568,P 除以 Q 所得到的商恰是 40320.

学而思奥数网 www.aoshu.cn Page 112 of 137


www.baoshi2011.com

2.老师在黑板上依次写了三个数 21、7、8,现在进行如下的操作,每次将这三个数中的某些数加上
2,其他数减去 1,试问能否经过若干次这样的操作后,使得:
(1)三个数都变成 12? (2)三个数变成 23、15、19?
【分析与解】 如果两个数都加上 2,那么它们的差不变;如果两个数都减去 1,那么它们的差也不变;
如果一个数加上 2,一个数减去 1,那么它们的差增大或减小 3.所以,不管怎样,它们的差增大或减
小 3 的倍数.也就是说,不管怎么操作,这两个数的差除以 3 的余数是不变的.
21 与 7 的差除以 3 的余数为 2;21 与 8 的差除以 3 的余数为 1;7 与 8 的差除以 3 的余数为
1.
(1)三个数都变成 12,那么它们的差除以 3 的余数都是 0,显然与开始给出的三个数之间差的余
数有变化,所以不满足;
(2)三个数变成 23、15、19,它们之间差除以 3 的余数依次为:
23 与 15 的差除以 3 的余数为 2;
23 与 19 的差除以 3 的余数为 1;
15 与 19 的差除以 3 的余数为 1.也就是说与开始给出的三个数之间差的余数没变化,所以满
足.

3.对于 n 个奇质数,如果其中任意奇数个数的和仍是质数,那么称这些数构成“奇妙数组”,而 n 就
是这个数组的“阶数”.例如 11,13,17 就是“奇妙数组”,因为 11,13,17 和 11+13+17=41 都
是质数.
(1)证明:“奇妙数组”的“阶数”最大值为 4;
(2)对于“阶数”为 4 的“奇妙数组”,求这 4 个质数的乘积的最小值.
【分析与解】 (1)假设 a、b、c、d、e 能组成一个 5 阶“奇妙数组”,那么 a、b、c、d 一定可以组成一个四
阶“奇妙数组 ”,考虑除以 3 的余数情况,不能存在 3 的数它们除以 3 的余数相同,并且验证只能是
1,1,2,2.则 e 除以 3 不管是余 0,1,2 都能在这五个数中找到三个数,它们的和是 3 的倍数,
且大于 3,所以无法组成 5 阶 “ 奇妙数组 ” .但是如 97,73,4l,53 满足(它们的三个数和依次为
167,191,223,2ll 均是质数).所以存在最大的 4 阶“奇妙数组”.
(2)写出所有除以 3 余 1 的质数:7,13,19,31,37,43,61,67,73,79,97;
写出所有除以 3 余 2 的质数:(2,5),11,17,23,29,41,47,53,59,71,83,89.
很容易知道 2 是不能含有,不然其他两个奇质数与 2 的和为大于 2 的偶数,显然不是质数,5 也
很容易验证不满足;

有 7 , 13 , 11 , 23 满 足 ( 和 依 次 为 47 , 4l , 43 , 31) . 它 们 的 乘 积 为
7×13×11×23=23023.所以 4 阶“奇妙数组”的 4 个数最小乘积为 23023.
评 注 : 四 阶 的 “ 奇 妙 数 组 ” 还 有 很 多 , 如 97 , 13 , 41 , 53 . 它 们 的 三 个 数 和 依 次 为
107,191,163,
151 均是质数.

学而思奥数网 www.aoshu.cn Page 113 of 137


www.baoshi2011.com

第 30 讲 几何综合 2
内容概述
勾股定理,多边形的内角和,两直线平行的判别准则,由平行线形成的相似三角形中对应线段和面
积所满足的比例关系.与上述知识相关的几何计算问题.各种具有相当难度的几何综合题.
典型问题

2.如图 30-2,已知四边形 ABCD 和 CEFG 都是正方形,且正方形 ABCD 的边长为 10 厘米,那么图


中阴影三角形 BFD 的面积为多少平 方厘米?

【分析与解】 方法一:因为 CEFG 的边长题中未给出,显然阴


影部分的面积与其有关.设正方形 CEFG 的边长为 x,有:

1 1 10x-x 2
S正方形ABCD =10 × 10=100, S正方形CEFG=x 2 , S ∆DGF = DG × GF= (10-x)x= ,
2 2 2

1 1 10x+x 2
又 S ∆ABD = ×10 ×10=50, S ∆BEF = (10+x)x= .
2 2 2
阴影部分的面积为:

ABCD + S
S正方形正方形 CEFG + S ∆DGF − S∆ABD − S∆BEF

10 x − x 2 10 x + x2
= 100 + x 2 + − 50 − = 50 (平方厘米).
2 2
方法二:连接 FC,有 FC 平行与 DB,则四边形 BCFD 为梯形.

有△DFB、△DBC 共底 DB , 等 高 , 所 以 这 两 个 三 角 形 的 面 积 相 等 , 显
1
然 ,△DBC 的 面 积 × 10 × 10 = 50 (平方厘米).
2
阴影部分△DFB 的面积为 50 平方厘米.

4. 如 图 30-
4 , ∠ A+∠B+∠C+∠D+∠E+∠F+∠G+ ∠H+∠I 等于多少度?

学而思奥数网 www.aoshu.cn Page 114 of 137


www.baoshi2011.com

【分析与解】 为了方便所述,如下图所示,标上数字,
有∠I=1800 -(∠1+∠2),而∠1=1800-∠3, ∠2=1800-∠4,有∠I=∠3+∠4-1800
同理, ∠H=∠4+∠5-1800, ∠G=∠5+∠6-1800, ∠F=∠6+∠7-1800, ∠E=∠7+∠8-1800,
∠D=∠8+∠9-1800, ∠C=∠9+∠10-1800, ∠B=∠10+∠11-1800, ∠A=∠11+∠3-
1800


∠ A+∠B+∠C+∠D+∠E+∠F+∠G+ ∠H+∠I=2×(∠3+∠4+∠5+∠6
+∠7+∠8+∠9+∠10+∠11)-9×180 0

而∠3+∠4+∠5+∠6+∠7+∠8+∠9+∠10+∠11 正是 9 边形的内角和为(9-2)×1800=12600.
所以∠A+∠B+∠C+∠D+∠E+∠F+∠G+∠H+∠I=2×12600-9×1800=9000
6.长边和短边的比例是 2:1 的长方形称为基本长方形.考虑用短边互不相同的基本长方形拼图,要
求任意两个基本长方形之间既没有重叠,也没有空隙.现在要用短边互不相同且最小短边长为 1 的 5 个
基本长方形拼接成一个更大的长方形.例如,短边长分别是 1,2,5,6,12 的基本长方形能拼接成
大 长 方 形 , 具 体 案 如 图 30-6 所 示 . 请 给 出 这 5 个 基 本 长 方 形 所 有 可 能 的 选 择 方 式 . 设
a1=1<a2<a3<a4<a5 分别为 5 条短边的长度,则我们将这种选择方式记为(a1,a2,a3,a4,a5),这里无需考
虑 5 个基本长方形的拼图方案是否惟一.

【分析与解】 我们以几个不同的 基本长方形作为分类依据,


并按边长递增的方式一一列出.

第一类情况:以 为特征的有 7 组:

学而思奥数网 www.aoshu.cn Page 115 of 137


www.baoshi2011.com

第二类情况:以 为特征的有 6 组:

第三类 情 况 有如下三组:

共 有 16 组 解,它们是:
(1,2,2.5,5,7.25 ),(1,2,2.5,5,14.5).
(1,2,2.25,2.5,3. 625),(1,2,2.25,2.5,7.25).
(1,2,5,5.5,6),(1,2,5,6,11),
(1,2,2.5,4.5,7),(1,2,2.5,4.5,14),
(1,2,5,12,14.5),(1,2,5,12,29),
(1,2,2.25,2.5,4.5),(1,2,5,6,12).

 10 20 25 
 1, , 2, ,  , (1,2,2.4,4.8,5),
 9 9 9 

 13 10 25 14   7 8 13 10 
 1, , , ,  , 1, , , ,  .
 6 3 6 3   3 3 3 13 
8.如图 30-8,ABCD 是平行四边形,面积为 72 平方厘米,E,F 分别为边 AB,BC 的中点.则图形中
阴影部分的面积为多少平方厘 米?

【分析与解】 如下图所示,连 接 EC,并在某些点处标上字母,

因 为 AE 平 行 于 DC , 所 以 四 边 形 AECD 为梯形,有 AE:DC=1:2,

所以 S ∆AEG : S∆DCG = 1: 4 ,

S ∆AGD × S∆ECG = S∆AEG × S∆DCG ,且有 S ∆AGD = S∆ECG ,所以 S ∆AEG : S∆ADG = 1: 2 ,而这两个三角形高相

同,面积比为底的比,即 EG:GD=1:2,同理 FH:HD=1:2.


1 1
有 S ∆AED = S∆AEG + S∆AGD ,而 S ∆AED = × × SY ABCD = 18 (平方厘米)
2 2

学而思奥数网 www.aoshu.cn Page 116 of 137


www.baoshi2011.com

有 EG:GD= S ∆AEG : S∆AGB ,

1 2
所以 S ∆AEG = × S∆AED = 6 (平方厘米) S ∆AGD = × S∆AED = 12 (平方厘米)
1+ 2 1+ 2
同理可得 S ∆HFC = 6 (平方厘米), S ∆DCH = 12 (平方厘米) , S ∆DCG = 4 S∆AEG = 4 × 6 = 24 (平方厘

米)

又 S ∆GHD = S∆DCG − S∆DCH =24-12=12(平方厘米)

所以原题平行四边形中空白部分的面积为 6+6+12=24(平方厘米),所以剩下的阴影部分面积为
72-24=48(平方厘米).

10.图 30-10 是一个正方形,其中所标数值的单位是厘米.问:阴影部分的面积是多少平方厘米?

【分析与解】 如下图所示,为了方便所叙,将某些点标上 字 母 , 并 连 接
BG.

设△ AEG 的面积为 x,显然△ EBG、△ BFG、△ FCG 的面积均 为 x , 则


1 100
△ ABF 的面 积为 3x , S ∆ABF = × 20 × 10 = 100 即 x = ,那 么 正方形内空
2 3
400
白 部 分 的 面 积 为 4x = . 所以原题中阴影部分面积为
3
400 800
20 × 20 − = (平方厘米).
3 3
12.如图 30-12,若图中的圆和半圆都两两相切,两个小圆和三个半圆的半径长都是 1.求阴影部分
的面积.

【分析与解】 如下图所示,左图中 的 3 个阴影部分面积相等,右图


中的 3 个阴影部分的面积也相等.我 们把左下图中的每一部分阴影称
为 A,右下图中的每一部分阴影称为 B.
1 1 2 9π
大半圆的面积为 3 A + 3B + 3 小圆的面积 = × 3 × π = .
2 2 2

学而思奥数网 www.aoshu.cn Page 117 of 137


www.baoshi2011.com

 9π 1  π
而小圆的面积为 π ,则 A + B =  −3 π ÷3 = ,
 2 2  3
π π 5
原题图中的阴影部分面积为小半圆面积与阴影 A、B 的面积和,即为 + = π
2 3 6
14.如图 30-14,将长方形 ABCD 绕顶点 C 顺时针旋转 90 度,若 AB=4,BC=3,AC=5,求 AD 边
扫过部分的面积.( π 取 3.14)

【分析与解】 如下图所示,

如下图所示,端 点 A 扫过的轨
迹 为 AA′′A′ , 端 点 D 扫过轨迹为
DD′′D′ , 而 AD 之 间的点,扫过
的轨迹在以 A、D 轨迹,AD, A′D′ 所形成的封闭图形内,且这个封闭图形的每一点都有线段 AD 上某
点扫过,所以 AD 边扫过的图形为阴影部分.

∆A′D ′C + S DDACA′ − S
显然有阴影部分面积为 S直角扇形直角扇形C ∆ACD −S ′ ,而直角三角形 A′D′C 、ACD 面积

相等.
所以

∆A′D′C + S ACA′ − S
S直角扇形直角扇形CDD扇形扇形CDD ∆ACD − S ′ =S ACA′ −S ′

90π 90π π 9
= AC 2 − CD2 = (52 − 42 ) = π = 7.065(平方厘米)
360 360 4 4

即 AD 边扫过部分的面积为 7.065 平方厘米.

学而思奥数网 www.aoshu.cn Page 118 of 137


www.baoshi2011.com

第 31 讲 图形变换
内容概述
本讲将涉及到图形的对称、平移、旋转、割补及其他等积变换,下面我们就汶些变换的预备知识及变换
本身进行学习和探讨.
AB BC CA
1.三角形 ABC 与 A′B′C ′ ,如果它们的对应边成比例,即 = = = K ,我们就称它们相似,
A′B′ B′C ′ C ′A′
记作△ABC~△ A′B′C ′ .
这个比值 K 叫做两个三角形的相似系数(注意三角形的先后顺序),如果相似系数为 1,就称这两个三
角形全等,记作△ABC≌△ A′B′C ′ .
如果一个三角形的两角分别与另一个三角形的两角对应相等,那么这两个三角形相似;
如果一个三角形的两条边与另一个三角形的两条边对应成比例 ,并且夹角相等,那么这两个三角形相
似;
如果一个三角形的三条边与另一个三角形的三条边对应成比例,那么这两个三角形相似.
(以上 3 条判定定理中,如果含有边的比例的关系,而其中的比例系数为 l,则这两个三角形全等.)
2.两条直线平行,则:

反之, 如 果
知道上面 某 种
情况的成 立,则
那两条直 线 平
行.
3.两个相似三角形的面积比值为相似系数的平方.
典型问题
2.四边形 ABCD 中,AB=30,AD=48,BC=14,CD=40. 又已知 ∠ ABD+∠BDC=900,求四边形 ABCD
的面积.

【分 析与解】 如下图,以 BD 的垂直平分线为 对称轴 L, 做 △ ABD 关于 L 的


对称图形△ A′ BD.连接 A′ C.

因 为 ∠ ABD+∠BDC=9000 而 ∠ ABD=∠ A′ DB=900, 所 以 有 ∠ A′


DB+∠BDC=900.

学而思奥数网 www.aoshu.cn Page 119 of 137


www.baoshi2011.com

那么 A′ CD 为直角三角形,由勾股定理知 A′C 2 = AB 2 + CD 2 =2500,所以 A′C = 50 .

而在△ A′ BC 中,有 A′ B=AD=48,有 482+142=2500,即 A′ B2+BC2= A′ C2, 即△ A′ BC 为直角三


角形.
1 1
有 SVA′CD + SVA′BC = 30 × 40 × +14 × 48 × = 936 .
2 2
而| S四边形ABCD = SVA′CD + SVA′BC = 936 .

评注:Ⅰ.本题以∠ABC+∠BDC=900 突破口,通过对称变换构造出与原图形相关的角三角形.这样面积
就很好解决了.

Ⅱ.对于这道题我们还可以将△BCD 作 L 的对称图形.如下:

4.如图,在三角形 ABD 中,当 AB 和 CD 的长度相等时,请求出“?”所示的角 是


多少度,给出过程.

【 分 析 与 解 】 因为 AB=CD, 于是可以将三角形 ABC 的边 BA


边与 CD 对齐, 如下图. 在下图中 有∠BCA=1100,所以∠ACD=700

于 是 ∠ AC C ′ =∠ ACD +∠ DC C ′ =∠ ACD +∠ ABC


=70 +40 =110 ;
0 0 0

即 ∠ AC C ′ =1100=∠ CC ′D ;又因为 C ′A′ 只是 CA 移动的变化,


所以 C ′A′ = CA ;则 ABC ′A′ 是一等腰梯形.
于是,∠ ADC ′ =1800-1100=700;
又∠ CDC ′ =300,所以∠ ADC =700-300=400.
6. 如 下 图 ,△ABC 是 边 长 为 1 的 等 边 三 角 形 ,△BCD 是 等 腰 三 角 形 BD=CD , 顶 角
∠BDC=1200,∠MDN=600,求△AMN 的周长.

【分析与解】 如下图, 延长 AC 至 P,使 CP=MB,连接 DP.

1 1
则 有 ∠ MBD=600+ SVADE = S正六边形DEQSRT = SVDQR
6 3
1800 − 1200
= ∠PCD;CP=BM;BD=CD,所以有△MBD≌△PCD.
2

学而思奥数网 www.aoshu.cn Page 120 of 137


www.baoshi2011.com

于 是 ∠ MDC=∠PDC ; 又 因 为 ∠ MDB+∠NDC=600, 所 以
∠PDC+∠NDC=∠NDP=600;MD=PD
在 △ MDN 、 △ PND 中 ,∠NDM=∠NDP,ND=ND,MD=PD, 于 是 △ MND≌△PND. 有
MN=PN.
因 为 NP=NP=NC+CP, 而 AM=AB-MB=AB-CP, 所 以 AM+AN+MN=(AB-CP)+AN+
(NC+CP)=AB+AN+NC=2.
即△AMN 的周长为 2.
8.下图为半径 20 厘米、圆心角为 1440 的扇形图.点 C、D、E、F、G、H、J 是将扇形的 B、K 弧线分为 8 等份
的点.求阴影部分面积之和.

【分析与解】 如下图,做出辅助 线
△KMA 与 △ ANG 形状相同(对 应角相等),大小相等(对应边相

等),有△KMA≌△ANG, SVKMA = SVANG ,而△LMA 是两个三角形的公共部分,所以上图中的阴影部分面积

相等.

所以,GNMK 与扇形 KGA 的面积相等,那么 KGEB 的面积为


2 倍扇形 KGA 的面积.

1440
扇 形 KGA 的 圆 心 角 为 ×3=540, 所 以 扇 形 面 积 为
8
54
× 200 ×π = 60π 平方厘米.
360
那么 KGEB 的面积为 60 π × 2 =120 π 平方厘 米.
如下图,做出另一组辅助线.

△JQA 与 △ ARH 形状相同(对应角相等),大小相 等(对应边

相等),有△JQA≌△ARH, SVJQA = SVARH =5△A,而△PQA 是两个三角形的公共部分,所以右图中的阴影部

分面积相等.
所以,JHRQ 与扇形 JHA 的面积相等,那么 JHDC 的面积为 2 倍扇形 JHA 的面积.

1440 18
扇形 JHA 的圆心角为 = 1800 ,所以扇形面积为 × 202 × π = 20π 平方厘米.
8 360
那么 JHDC 的面积为 20π × 2 = 40π 平方厘米.
所以,原题图中阴影部分面积为

S KGEB − S JHDC = 120π − 40π = 80π ≈80×3.14=251.2 平方厘米.

学而思奥数网 www.aoshu.cn Page 121 of 137


www.baoshi2011.com

第 32 讲 勾股定理
内容概述
1.勾股定理(毕达哥拉斯定理):直角三角形中的两直角边平方后的和等于斜边的平方.

公元前 500 年古希腊的毕达 哥拉斯发现了勾股定理后,曾宰牛百头,广设


盛筵以示庆贺.
2. 公元前 11 世纪的《周髀算经》 中提到:故折矩,以为句广三,股修四、径修五.
既方之.外半卿一矩,环而共盘.得成三、四、五.

三国时期的赵 爽注解道:句股各
自乘,并之为弦实, 开方除之,即弦.
案:弦图又可以句 股相乘为朱实二,
倍之为朱实四,以句股之差自相乘为中黄实,加差之,亦成弦实.

汉朝张苍、狄 昌寿整理的《九
章算术》第九卷为
《句股》.其中解释

学而思奥数网 www.aoshu.cn Page 122 of 137


www.baoshi2011.com

到:短面曰句,长面曰股,相与结角曰弦.句短其股,股短其弦.
句股各自乘,并,而开方除之,即弦.
中国科学院数学与系统科学研究院的徽标(右图所示)采用的就是赵爽
的弦图.2002 年在北京举行的国际数学家大会的徽标也是弦图.
1
如下,在弦图中有 S四边形EFGH =
2
( ABCD + S
S矩形矩形 MNPQ )S VC ′DG = SVADG = SVCDE

3. 伽菲尔德证法 :美国第 20 任总统 伽菲尔德对数学有浓厚的兴趣 ,在还是中


学教师时曾给出一种勾股定理的证明 方法:
1
梯 形 面 积= (上底+下底)×高
2
1
(a+b)×(a+b) =
2
1
= (a+b)2;
2
1 1 1
三个直角三角形的面积和= ab+ ab+ c2;
2 2 2
梯形面积=三个直角三角形面积和.
1 1 1 1
(a+b)2= ab+ ab+ c2,所以 a2+b2=c2.
2 2 2 2
4. 公元前 3 世纪的欧几里得在《几何原本》中给出一种证明,简叙如下:
如图,作出三个正方形,它们的边长分别为直角三角形 ABC 的三边长.连接图中的虚线段对应的点;
过 C 作 CK 平行于 AF,交 AB、FG 分别于 J、K 点.

1 1
易 证 △ AFC≌△BAE, 有 SVFAC = AF.FK= S矩形AFKJ ,
2 2
1
SVBAE = EA.CA= S正方形ACDE , 所 以 S矩形AFKJ =
2
S正方形ACDE ;

1 1 1
易 证 △ CBG≌△HBA , 有 SVCBG = BG.KG= S矩形KGBJ , SVHBA = BH.IH= S正方形CBHI , 所 以
2 2 2
S矩形KGBJ

= S正方形CBHI .

AFGB = S
而 S正方形矩形 AFKJ + S矩形正方形
KGBJ = S ACBE + S正方形CBHI .

即有 AB2=AC2+CB2.
5. 勾股数组:a=u2-v2,b=2uv,c=u2+v2 如果 a、6、c 可以如此表达,那么 a、b、c 称之为勾股数组,有
a2+b2=c2.

学而思奥数网 www.aoshu.cn Page 123 of 137


www.baoshi2011.com

如:u=2,v=l 时 a=3,b=4,c=5;u=7,v=6 时 a=13,b=84,c=85.


当然将已知的勾股数组内每个数都同时扩大若干倍得到的新的一组数还是勾股数组.
典型问题
2.智能机器猫从平面上的 O 点出发.按下列规律行走:由 O 向东走 12 厘米到 A1,由 A1 向北走 24 厘米到
A2,由 A2 向西走 36 厘米到 A3,由 A3 向南走 48 厘米到 A4,由 A4 向东走 60 厘米到 A5,…,问:智能机器猫
到达 A6 点与 O 点的距离是多少厘米?
【分析与解】 如右图所示,当智能机器猫到达 A6 点时,相对
O 点,向东走了 12-36+60=36 厘米,向北走了 24-48+72=48 厘米.
2
有 OA6 =362+482,即 OA2=60.

所以,A6 点到 O 点的距离为 60 厘米.

4.如图 32-3 所示,直角三角形 PQR 的两个直角边分别为 5 厘米,9 厘米问下图中 3 个正方形面积之和


比 4 个三角形面积之和大多少?

【 分 析 与 解 】 如右图,延长 AR,DQ,过 E,F 分别作 AR,DQ 的平


行 线 , 在正 方形 EFRQ 内交 成四 个 全等的直角三角形和一个小正方形
GHMN , 四 个 全 等 的 直 角 三 角 形 面积之和与四个白色的三角形面积
之和相等.
小正方形 HGNM 的边长为 9-5=4 厘米,所以面积为 16 平方厘米,
而另
外两个正方形 ABPR、CDQR 他的面积分别为 25,81.所以原图中 3 个
正方
形面积之和比 4 个三角形面积之和大 25+8l+16=122 平方厘米.

6.若把边长为 1 的正方形 ABCD 的四个角剪掉,得一四边形 A1BlClDl,试


5
问怎样剪,才能使剩下的图形仍为正方形,且剩下图形的面积为原来正方形面积的 ,请说明理由.(写出
9
证明及计算过程)
【分析与解】如左图所示,我们知道利用弦图,可是弦图怎么利用?设构造出的弦图中最小正方形的面
积为 x 最大正方形面积为 1,那么有剩下的正方形面
1 5 1
积为 (x+1)= ,所以 x= .
2 9 9
1
那么,最小正方形的边长为 .由于是四角对称的剪
3
1 2
去,所以有 ADl=DCl=CBl=BA1= ,AAl=BBl=CCl=DDl=
3 3
证明及计算过程略.
8.有 5 个长方形,它们的长和宽都是整数,且 5 个长和 5 个宽恰好是 1~10 这 10 个整数;现在用这 5
个长方形拼成 1 个大正方形,那么,大正方形面积的最小值为多少?
【分析与解】 注意到,5 个长、宽均不相等的长方形拼成一个正方形,只有一种拼法.(如右图所示,由
弦图联想到).
A、B、C、D 中必有一个长方形的一边长为 10,不妨设为 A,

学而思奥数网 www.aoshu.cn Page 124 of 137


www.baoshi2011.com

那么显然不能组成边长为 10 的正方形;
如果能够组成边长为 11 的正方形,那么有 11=10+1=9+2=8+3=7+4=6+5,那么大正方形的
四边必须是为 11,则剩下的两个数,它们的和为 11,为中问阴影部分的长、宽和;

评 注 :如果能够组成边长为 12 的 正 方 形 , 那 么 有
12=10+2=9+3=8+4=7+5, 剩 下 1、6 试填不满足.
对于边长为 13 的正方形 ,注意到 13=10+3=9+4=8+5=7+6,剩下 1、2,
有见下图情形,满足.

10.园林小路,曲径通幽.如图 32-7 所示, 小路由白色正方形石板和青、红两色的


三角形石板铺成.问:内圈三角形石板 的总面积大,还是外圈三角形的总面积大?
请说明理由.

【分析与解】 如图①,我们任意抽 出两块相邻的白色正方形石板,及它们


所夹成的青、红两色的三角形石板 , 如 图 ② 所 示 . 图 中 有
∠CDB+∠ADG=180 .
0

如果③,将△CDE 逆时针旋转 900,得△ C ′DG .有 A 、D 、C ′ 在同一条直线上,且△ C ′DG 与△ ADG 等

底同高,所以有 SVC ′DG = SVADG = SVCDE .

也就是说,任 意两块相邻
的白色正方形石 板,它们所夹
成的青色三角形 与红色三角
形面积相等.
注意到在原图中,除了外圈青色的两块三角形外,外圈三角形、内圈三角形一一对应.所以原图中,外
圈三角形的面积大于内圈三角形的面积,如图①所示.

第 33 讲 计数综合 2
内容概述
利用对应法求解的计数问题.所谓对应法,即建立起所考察对象和另一类对象之间的对应关系,通过
对后者的计数而求得问题的答案.与平面和立体图形相关的复杂计数问题,其他具有相当难度的计数综
合题.
典型问题

学而思奥数网 www.aoshu.cn Page 125 of 137


www.baoshi2011.com

2. 小明有 10 块大白兔奶糖,从今天起,每天至少吃一块.那么他一共有多少种不同的吃法?
[分 析 与 解 ] 我们将 10 块大白兔奶糖从左至右排成一列,如果在其中 9 个间隙中的某个位置插入
“木棍”,则将 lO 块糖分成了两部分.
我们记从左至右,第 1 部分是第 1 天吃的,第 2 部分是第 2 天吃的,…,
如:○○○|○○○○○○○表示第一天吃了 3 粒,第二天吃了剩下的 7 粒:
○○○○ | ○○○| ○○○表示第一天吃了 4 粒,第二天吃了 3 粒,第三天吃了剩下的 3 粒.
不难知晓,每一种插入方法对应一种吃法,而 9 个间隙,每个间隙可以插人也可以不插入,且相互独立,
故共有 29=512 种不同的插入方法,即 512 种不同的吃法.
4. 在 8×8 的方格表中,取出一个如图 33-1 所示的由 3 个小方格组成的“L”形,一共有多少种不
同的方法?

【分析与解】 观察发现,对于每个“L”形,都有一个点 M 与其对应,而每个 2×2 的方格中,M 点


都对应 4 个不同的“L”.

在 8×8 的 方 格 中 , 类 似 M 点 的 交 叉 点 有 7×7=49 个(不包括边


上 的 交 叉 点 ) . 所 以 共 有 “ L” 形 4×49=196 种不同的取法.

评 注 :通过上面两个范例我们 知道,当直接去求一个集合元素的个数
较为困难的时候 ,可考虑采用相等 的原则,把问题转化成求另一个集合的
元素个数

6. 有一批规格相同的均匀圆棒,每根划分成相同的 5 节,每节用红、黄、蓝 3 种颜色中的一种来涂.问可以
得到多少种着色方式不同的圆棒?
【分析与解】 如图 每根原棒的 5 节记为 A、B、C、D、E,特别得注意到原棒可

左右倒置,即 有可能与 是同种情况.

不难得知,当原棒上的 5 节对称时,即 与
是同种情况.

① ,其中 A 有 3 种颜色可选,B 也有 3 种颜色可选,C 还是有 3 种颜色可选,故有

3×3×3=27 种不同的染法.
② 考虑不对称 时 则 A 有 3 种原色可选,B、C、D、E 也各有 3
种颜色可选,于是有
3×3×3×3×3=243 种不同的染法.

所以,其中不对称有 243-27=216 种,不对称 的 与


重复计算

了,而对称 的 没有重复计算.

学而思奥数网 www.aoshu.cn Page 126 of 137


www.baoshi2011.com

所以,共有 216÷2+27=135 种实质不同的着色方式.


8. 如图 33-3,八面体有 12 条棱,6 个顶点.一只蚂蚁从顶点 A 出发,沿棱爬行,要求恰好经过每个顶点一
次.问共有多少种不同的走法?
【分析与解】 A → B,A → D,A → E,A → F,这 4 类走法,每类走法的种数一样多,所以只用考察 A →
B 的后续步骤有多少种:
B → E → C → D → F,B → E → C → F → D,B → E → D → F → C,B → E → D → C → F,
B → F → D → E → C,B → F → D → C → E,B → F → C → E → D,B → F → C → D → E,
B → C → E → D → F,B → C → F → D → E(从 B → C 后三步只能是顺时针或逆时针,只用 2 种).
共 10 种.

所 以 从 A 点 出 发 共 有 10×4=40 种不同的满足题中条件的
走法
10. 某 玩 具 厂 生 产 大 小 一 样 的 正 方 体形状的积木,每个面分别涂上红、黄、
蓝三种颜色中的一种 ,每色各涂两个 面.当两个积木经过适当的翻动以后,
能使各种颜色的面所在位置相同时,它们就被看作是同一种积木块.试说明:最多能涂成多少种不同的积
木块?
【分析与解】 总可以使下底面为红色.
如果上底面也是红色,通过翻过,可以使前面为黄色,左面不是黄色,这时后面可以是黄色,也可以是
蓝色,有 2 种.
如果上底面不是红色,通过旋转,可以使后面为红色.这时又分两种情况:
(1)前面与上面同色,可以同为黄色,也可以同为蓝色,有 2 种.
(2)前面与上面不同色,通过翻动,可以使上面为黄色,前面为蓝色这时右面可以是黄色,也可以是蓝色,
有 2 种.
因此,共可涂成 2+2+2=6 种不同的积木块.
12.有 8 个队参加比赛,采用如图 33-4 所示的淘汰制方式.问在比赛前抽签时,可以得到多少种实质不同
的比赛安排表?

【分析与解】 我们标 上字母,如下图.

8
如果全排列为 P8 =8!因 为 A,B;B,A 实质赛程一样;同理

C/D,E/F,G/H,I/J,K/L,M/N 均是,所以除以 7 个 2.
于是,共有 8!÷27=315 种实质不同的赛程安排.
14. 游乐园的门票 1 元 1 张,每人限购 1 张.现在有 10 个小朋友排队购票,其中 5 个小朋友只有 1 元的
钞票,另外 5 个小朋友只有 2 元的钞票,售票员没有准备零钱.问有多少种排队方法,使售票员总能找得开
零钱?
【分析与解】 方法一:按第一个带 2 元钞票的小朋友前面有几个小朋友来确定排队的方案,共有五个
方案:
① 带 1 元的 5 个小朋友都排在前边,即 1111l22222,只有 1 种情况;

学而思奥数网 www.aoshu.cn Page 127 of 137


www.baoshi2011.com

⑦ 带 1 元 的 小 朋 友 有 4 个 排 在 前 面 , 即
1111212222,1111221222,1111222122,1111222212,共有 4 种情况;
③ 带 1 元的小朋友有 3 个排在前边,如 1112112222,…,共有 9 种情况;
④ 带 1 元的小朋友有 2 个排在前边,如 1121112222,…,共有 14 种情况;
⑤ 带 1 元的小朋友只有 1 个排在前边,如 1211112222,…,共有 14 种情况.
五个方案共有 1+4+9+14+14=42(种)情况.
因 为 10 个 小 朋 友 互 不 相 同 , 所 以 每 种 情 况 有 5 ! ×5 ! =14400( 种 ) 排 队 方 法 , 总 共 有
42×14400=604800 种排队方法,使售票员总能找得开零钱.
方法二:如下左图,先将拿 1 元的小朋友看成相同的,2 元的小朋友看成相同的.在下图中,每条小横线
代表拿 l 元的小朋友,每条小竖线代表拿 2 元的小朋友.
从 A 到 B 的不论在网格中的何点均有横线数不小于竖线
数.
相当于求 A 到 B 的走法:

我们再由上右图知:从 A → B 的走法有 42 种.
因为各个小朋友都是不同的,所以共有 42×5!×5!=42×120×120=604800 种情况.
评 注 :游乐园的门票 1 元 1 张,每人限购 1 张.现在有 10 个小朋友排队购票,其中 n 个小朋友只有 1

( 2n ) !× n!
元的钞票,另外 n 个小朋友只有 2 元的钞票,售票员没有准备零钱.则有 种排队方法,使售票员
( n+1) !
总能找得开零钱.

第 34 讲 最值问题

学而思奥数网 www.aoshu.cn Page 128 of 137


www.baoshi2011.com

内容概述
均值不等式,即和为定值的两数的乘积随着两数之差的增大而减小.各种求最大值或最小值的问
题,解题时宜首先考虑起主要作用的量,如较高数位上的数值,有时局部调整和枚举各种可能情形也
是必要的.
典型问题
2.有 4 袋糖块,其中任意 3 袋的总和都超过 60 块.那么这 4 袋糖块的总和最少有多少块?
【分析与解】 方法一:设这 4 袋为 A、B、C、D,为使 4 袋糖块的总和最少,则每袋糖应尽量平均,
有 A、B、C 袋糖有 20、20、21 块糖.
则当 A、B、D 三袋糖在一起时,为了满足条件,D 袋糖不少于 21 块,验证 A、B、C、D 这 4 袋糖依
次有 20,20,2l,2l 时满足条件,且总和最少.
这 4 袋糖的总和为 20+20+21+21=82 块.
方法二:设这 4 袋糖依次有 a、b、c、d 块糖,
 a + b + c ≥ 61①

 a + b + d ≥ 61② 1
有 , ① +②+③+④ 得:3(a+b+c+d ) ≥ 244,所以 a+b+c+d≥81 ,因为
 a + c + d ≥ 61③ 3
 b + c + d ≥ 61④
a+b+c+d 均是整数,所以 a+b+c+d 的和最小是 82.
a + b + c〉 60 ①

 a+b+d〉 60 ②
评 注 : 不 能 把 不 等 式 列 为  , 如 果 这 样 将 ① +②+③+④ 得 到
 a+c+d〉 60 ③
 b+c+d〉 60 ④
3(a+b+c+d)>240,a+b+c+d>80,因为 a、b、c、d 均是整数,所以 a+b+c+d 的和最小是 81.至于为什么会出现这
种情况.如何避免,希望大家自己解决.
4.用 1,3,5,7,9 这 5 个数字组成一个三位数 ABC 和一个两位数 DE,再用 O,2,4,6,8 这
5 个数字组成一个三位数 FGH 和一个两位数 IJ.求算式 ABC×DE-FGH×IJ 的计算结果的最大值.
【分析与解】 为了使 ABC×DE-FGH×IJ 尽可能的大,ABC×DE 尽可能的大,FGH×IJ 尽可能的
小.
则 ABC×DE 最大时,两位数和三位数的最高位都最大,所以为 7、9,然后为 3、5,最后三位数
的个位为 1,并且还需这两个数尽可能的接近,所以这两个数为 751,93.
则 FGH×IJ 最小时,最高位应尽可能的小,并且两个数的差要尽可能的大,应为 468×20.
所以 ABC×DE-FGH×IJ 的最大值为 751×93-468×20=60483.
评注:类似的还可以算出 FGH×IJ-ABC×DE 的最大值为 640×82-379×15=46795.

6.将 6,7,8,9,10 按任意次序写在一圆周上,每相邻两数相乘,并将所得 5 个乘积相加,那么


所得和数的最小值是多少?

【分析与解】 我们从对结果影响最大的数上人手,然后考虑次大的,所以我
们首先考虑 10,为了让和数最小,10 两边的数必须为 6 和 7.
然后考虑 9,9 显然只能放到图中的位置,最后是 8,8 的位置有两个位置可放,而且也不能立即
得到哪个位置的乘积和最小,所以我们两种情况都计算.

学而思奥数网 www.aoshu.cn Page 129 of 137


www.baoshi2011.com

8×7+7×10+10×6+6×9+9×8=312;
9×7+7×10+10×6+6×8+8×9=313.
所以,最小值为 312.
8.一个两位数被它的各位数字之和去除,问余数最大是多少?

【分析与解】设这个两位数为 ab =lOa+b,它们的数字和为 a+b,因为 lOa+b=(a+b)+9a,所以

lOa+b≡9a(mod a+b),
设最大的余数为 k,有 9a≡k(mod a+b).
特殊的当 a+b 为 18 时,有 9a=k+18m,因为 9a、18m 均是 9 的倍数,那么 k 也应是 9 的倍数
且小于除数 18,即 0,9,也就是说余数最大为 9;
所以当除数 a+b 不为 18,即最大为 17 时,

 m=7+9t
:余数最大为 16,除数 a+b 只能是 17,此时有 9a=15+17m,有  (t 为
a=15+17t
可取 0 的自然数),而 a 是一位数,显然不满足;
:余数其次为 15,除数 a+b 只能是 17 或 16,

 m=6+9t
除数 a+b=17 时,有 9a=15+17m,有  ,(t 为可取 0 的自然数),a 是一位数,显然也
a=13+17t
不满足;

 m=3+9t
除数 a+b=16 时,有 9a=15+16m,有  (t 为可取 0 的自然数),因为 a 是一位数,所
a=7+16t
以 a 只能取 7,对应 b 为 16-7=9,满足;
所以最大的余数为 15,此时有两位数 79÷(7+9)=4……15.
10.用 1,2,3,4,5,6,7,8,9 这 9 个数字各一次,组成一个被减数、减数、差都是三位数的
正确的减法算式,那么这个算式的差最大是多少?
【分析与解】考虑到对差的影响大小,我们先考虑百位数,为了让差最大,被减数的百位为 9,减数

的百位为 1,如果差的百位为 8,那算式就是如下形式: 剩下的 6 个数字为

2、3、4、5、6、7,因为百位数字为 8,所以我们可以肯定被减数的十位数字比减数要大,而且至少大
2,因为 1 已经出现在算式中了,算式的可能的形式如下:

得数的十位只可能是减数和被减数的十位数字之差,或者小 1,可能的算式形式如下:

但这时剩下的数都无法使算式成立.再考虑差的百位数字为 7 的情况,这时我们可以肯定减数的

学而思奥数网 www.aoshu.cn Page 130 of 137


www.baoshi2011.com

十位数比被减数要大,为了使差更大,我们希望差值的十位为 8,因此,算式可能的形式为:

再考虑剩下的三个数字,可以找到如下几个算式:

,所以差最大为 784.

12. 4 个不同的真分数的分子都是 1,它们的分母有 2 个是奇数、2 个是偶数,而且 2 个分母是奇数的


分数之和与 2 个分母是偶数的分数之和相等.这样的奇数和偶数很多,小明希望这样的 2 个偶数之和
尽量地小,那么这个和的最小可能值是多少?
1 1 1 1
【分析与解】 设这四个分数为上 、 、 、 (其中 m、n、a、b 均为非零自然数)
2m 2n 2a+1 2b+1
1 1 1 1 1 1 1 1
有 + = + ,则有 - = - ,
2m 2n 2a+1 2b+1 2m 2b+1 2a+1 2n
我们从 m=1,b=1 开始试验:
1 1 1 1 1 1 1 1 1 1
= + = + , = + = + ,
2 6 3 4 4 3 12 4 6 6
1 1 1 1 1 1 1 1 1 1
= + = + , = + = + ,
4 20 5 8 8 5 30 6 10 10
1 1 1 1 1
= + = + ,﹍
6 5 10 12 12
1 1 1
我们发现, 和 分解后具有相同的一项 ,而且另外两项的分母是满足一奇一偶,满足题中条
5 6 10
件:
1 1 1 1
+ = + ,所以最小的两个偶数和为 6+10=16.
5 15 6 10
14.有 13 个不同的自然数,它们的和是 100.问其中偶数最多有多少个?最少有多少个?
【分析与解】 13 个整数的和为 100,即偶数,那么奇数个数一定为偶数个,则奇数最少为 2 个,
最多为 12 个;对应的偶数最多有 11 个,最少有 1 个.
但是我们必须验证看是否有实例符合.
当 有 11 个 不 同 的 偶 数 , 2 个 不 同 的 奇 数 时 , 11 个 不 同 的 偶 数 和 最 小 为
2+4+6+8+10+12+14+16+18+20+22=132,而 2 个不同的奇数和最小为 1+3=4.它们的和
最小为 132+4=136,显然不满足:
当 有 9 个 不 同 的 偶 数 , 4 个 不 同 的 奇 数 时 , 9 个 不 同 的 偶 数 和 最 小 为
2+4+6+8+10+12+14+16+18=90,而 4 个不同的奇数和最小为 1+3+5+7=16,还是大于
100,仍然不满足;
当 有 7 个 不 同 的 偶 数 , 6 个 不 同 的 奇 数 时 , 7 个 不 同 的 偶 数 和 最 小 为
2+4+6+8+10+12+14=56 , 6 个 不 同 的 奇 数 和 为 1+3+5+7+9+11 : 36 , 满 足 , 如
2,4,6,8,10,12,22,1,3,5,7,9,11 的和即为 100.
类 似 的 可 知 , 最 少 有 5 个 不 同 的 偶 数 , 8 个 不 同 的 奇 数 , 有
2,4,8,10,16,1.3.5,7,9,11,13,15 满足.
所以,满足题意的 13 个数中,偶数最多有 7 个,最少有 5 个.

学而思奥数网 www.aoshu.cn Page 131 of 137


www.baoshi2011.com

第 35 讲 构造与论证 1
内容概述
各种探讨给定要求能否实现,设计最佳安排和选择方案的组合问题.这里的最佳通常指某个量达
到最大或最小.解题时,既要构造出取得最值的具体实例,又要对此方案的最优性进行论证.论证中
的常用手段包括抽屉原则、整除性分析和不等式估计.
典型问题
2.有 3 堆小石子,每次允许进行如下操作:从每堆中取走同样数目的小石子,或是将其中的某一石子
数是偶数的堆中的一半石子移入另外的一堆.开始时,第一堆有 1989 块石子,第二堆有 989 块石子,
第三堆有 89 块石子.问能否做到:
(1)某 2 堆石子全部取光?
(2)3 堆中的所有石子都被取走?
【分析与解】 (1)可以,如(1989,989,89) → (1900,900,0) → (950,900,950) →
(50,0,50) → (25,25,50) → (O,0,25).
(2)因为操作就两种,每堆取走同样数目的小石子,将有偶数堆石子堆中一半移至另一堆,所
以每次操作石子总数要么减少 3 的倍数,要么不变.
现在共有 1989+989+89=3067,不是 3 的倍数,所以不能将 3 堆中所有石子都取走.
4.在某市举行的一次乒乓球邀请赛上,有 3 名专业选手与 3 名业余选手参加.比赛采用单循环方式进行,
就是说每两名选手都要比赛一场.为公平起见,用以下方法记分:开赛前每位选手各有 10 分作为底
分,每赛一场,胜者加分,负者扣分,每胜专业选手一场加 2 分,每胜业余选手一场加 1 分;专业选
手每负一场扣 2 分,业余选手每负一场扣 1 分.问:一位业余选手最少要胜几场,才能确保他的得分
比某位专业选手高?
【分析与解】 当一位业余选手胜 2 场时,如果只胜了另两位业余选手,那么他得 10+2-3=9(分).
此时,如果专业选手间的比赛均为一胜一负,而专业选手与业余选手比赛全胜,那么每位专业选手的
得分都是 10+2-2+3=13(分).所以,一位业余选手胜 2 场,不能确保他的得分比某位专业选手高.
当一位业余选手胜 3 场时,得分最少时是胜两位业余选手,胜一位专业选手,得 10+2+2-
2=12(分).此时,三位专业选手最多共得 30+0+4=34(分),其中专业选手之间的三场比赛共得 0
1
分,专业选手与业余选手的比赛最多共得 4 分.由三个人得 34 分,34÷3=11 ,推知,必有人得分
3
不超过 11 分.
也就是说,一位业余选手胜 3 场,能确保他的得分比某位专业选手高.

6.如图 35-1,将 1,2,3,4,5,6,7,8,9,10 这 10 个数分别填入图中的 10 个圆圈内,使任


意连续相邻的 5 个圆圈内的各数之和均不大于某个整数 M.求 M 的最小值并完成你的填图.

【分析与解】 要使 M 最小,就要尽量平均的填写,因为如果有的连续 5 个圆圈内的数


特别小,有的特别大,那么 M 就只能大于等于特别大的数,不能达到尽量小的目的.

学而思奥数网 www.aoshu.cn Page 132 of 137


www.baoshi2011.com

因为每个圆圈内的数都用了 5 次,所以 10 次的和为 5×(1+2+3+…+10)=275.


每次和都小于等于朋,所以 IOM 大于等于 275,整数 M 大于 28.
下面来验证 M=28 时是否成立,注意到圆圈内全部数的总和是 55,所以肯定是一边五个的和是
28,一边是 27.因为数字都不一样,所以和 28 肯定是相间排列,和 27 也是相问排列,也就是说数
组每隔 4 个差值为 l,这样从 1 填起,容易排出适当的填图.
8.1998 名运动员的号码依次为 1 至 1998 的自然数.现在要从中选出若干名运动员参加仪仗队,使
得剩下的运动员中没有一个人的号码等于另外两人的号码的乘积.那么,选为仪仗队的运动员最少有
多少人?
【分析与解】 我们很自然的想到把用得比较多的乘数去掉,因为它们参与的乘式比较多,把它们
去掉有助于使剩下的构不成乘式,比较小的数肯定是用得最多的,因为它们的倍数最多,所以考虑先
把它们去掉,但关键是除到何处?
考虑到 44 的平方为 1936,所以去到 44 就够了,因为如果剩下的构成了乘式,那么乘式中最小
的数一定小于等于 44,所以可以保证剩下的构不成乘式.因为对结果没有影响,所以可以将 1 保留,
于是去掉 2,3,4,…,44 这 43 个数.
但是,是不是去掉 43 个数为最小的方法呢?构造 2×97,3×96,4×95,…,44×45,发现这
43 组数全不相同而且结果都比 1998 小,所以要去掉这些乘式就至少要去掉 43 个数,所以 43 位最
小值,即为所求.
10.在 10×19 方格表的每个方格内,写上 0 或 1,然后算出每行及每列的各数之和.问最多能得到多
少个不同的和数?
【分析与解】首先每列的和最少为 0,最多是 10,每行的和最少是 0,最多是 19,所以不同的和
最多也就是 0,1,2,3,4,…,18,19 这 20 个.
下面我们说明如果 0 出现,那么必然有另外一个数字不能出现.
如果 0 出现在行的和中,说明有 1 行全是 0,意味着列的和中至多出现 0 到 9,加上行的和至多
出现 10 个数字,所以少了一种可能.
如果 0 出现在列的和中,说明在行的和中 19 不可能出现,所以 0 出现就意味着另一个数字不能出
现,所以至多是 19,下面给出一种排出方法.

12.在 1000×1000 的方格表中任意选取 n 个方格染为红色,都存在 3 个红色方格,它们的中心构


成一个直角三角形的顶点.求 n 的最小值.
【分析与解】 首先确定 1998 不行.反例如下:

其次 1999 可能是可以的,因为首先从行看,1999 个红点分布在 1000


行中,肯定有一些行含有 2 个或者以上的红点,因为含有 0 或 1 个红点的行最
多 999 个,所以其他行含有红点肯定大于等于 1999-999=1000,如果是大
于 1000,那么根据抽屉原理,肯定有两个这样红点在一列,那么就会出现红
色三角形;
如果是等于 1000 而没有这样的 2 个红点在一列,说明有 999 行只含有 1

学而思奥数网 www.aoshu.cn Page 133 of 137


www.baoshi2011.com

个红点,而剩下的一行全是红点,那也肯定已经出现直角三角形了,所以 n 的最小值为 1999.


14.在图 35-2 中有 16 个黑点,它们排成了一个 4×4 的方阵.用线段连接其中 4 点,就可以画出各
种不同的正方形.现在要去掉某些点,使得其中任意 4 点都不能连成正方形,那么最少要去掉多少个
点?

【分析与解】 至少要除去 6 个点,如下所示为几种方法:

第 36 讲 构造与论证 2
内容概述
组合证明题,在论证中,有时需进行分类讨论,有时则要着眼于极端情形,或从整体把握.若干
点及连接它们的一些线段组成图,与此相关的题目称为图论问题,这里宜从特殊的点或线着手进行分
析.各种以染色为内容,或通过染色求解的组合问题,基本的染色方式有相间染色与条形染色.
典型问题
2.甲、乙、丙三个班人数相同,在班级之间举行象棋比赛.各班同学都按 l,2,3,4,…依次编号.
当两个班比赛时,具有相同编号的同学在同一台对垒.在甲、乙两班比赛时,有 15 台是男、女生对垒;
在乙、丙班比赛时,有 9 台是男、女生对垒.试说明在甲、丙班比赛时,男、女生对垒的台数不会超过
24.并指出在什么情况下,正好是 24 ?
【分析与解】 不妨设甲、乙比赛时,1~15 号是男女对垒,乙、丙比赛时.在 1~15 号中有 a 台男女
对垒,15 号之后有 9-a 台男女对垒(0≤a≤9)
甲、丙比赛时,前 15 号,男女对垒的台数是 15-a(如果 1 号乙与 1 号丙是男女对垒,那么 1 号甲与
1 号丙就不是男女对垒),15 号之后,有 9-a 台男女对垒.所以甲、丙比赛时,男女对垒的台数为
15-a+9-a=24-2a≤24.
仅在 a=0,即必须乙、丙比赛时男、女对垒的号码,与甲、乙比赛时男、女对垒的号码完全不同,甲、
丙比赛时,男、女对垒的台数才等于 24.
4.将 15×15 的正方形方格表的每个格涂上红色、蓝色或绿色.证明:至少可以找到两行,这两行中
某一种颜色的格数相同.
【分析与解】 如果找不到两行的某种颜色数一样,那么就是说所有颜色的列与列之问的数目不同.
那么红色最少也会占:
0+1+2+…+14=105 个格子.
同样蓝色和绿色也是,这样就必须有至少:
3×(0+l+2+…+14)=315 个格子.
但是,现在只有 15×15=225 个格子,所以和条件违背,假设不成立,结论得证.

学而思奥数网 www.aoshu.cn Page 134 of 137


www.baoshi2011.com

6. 4 个人聚会,每人各带 2 件礼品,分赠给其余 3 个人中的 2 人.试证明:至少有 2 对人,每对人是


互赠过礼品的.
【分析与解】将这四个人用 4 个点表示,如果两个人之间送过礼,就在两点之间连一条线.
由于每人送出 2 件礼物,图中共有 4×2=8 条线,由于每人礼品都分赠给 2 个人,所以每两点之间
至多有 1+1=2 条线.
四点间,每两点连一条线,一共 6 条线,现在有 8 条线,说明必有两点之间连了 2 条线,还有另外两
点(有一点可以与前面的点相同)之间也连了 2 条线.
即为所证结论。
8.若干台计算机联网,要求:
① 任意两台之间最多用一条电缆连接;
② 任意三台之间最多用两条电缆连接;
③ 两台计算机之间如果没有电缆连接,则必须有另一台计算机和它们都连接有电缆.若按此要求最
少要用 79 条电缆.
问:(1)这些计算机的数量是多少台?
(2)这些计算机按要求联网,最多可以连多少条电缆?
【分析与解】将机器当成点,连接电缆当成线,我们就得到一个图,如果从图上一个点出发,可以沿
着线跑到图上任一个其它的点,这样的图就称为连通的图,条件③表明图是连通图.
我们看一看几个点的连通图至少有多少条线.可以假定图没有圈(如果有圈,就在圈上去掉一条线),
从一点出发,不能再继续前进,将这一点与连结这点的线去掉.考虑剩下的 n-1 个点的图,它仍然是
连通的.用同样的办法又可去掉一点及一条线.这样继续下去,最后只剩下一个点.因此 n 个点的连
通图至少有 n-1 条线(如果有圈,线的条数就会增加),并且从一点 A 向其他 n-1 个点各连一条线,这
样的图恰好有 n-1 条线.
因此,(1)的答案是 n=79+1=80,并且将一台计算机与其他 79 台各用一条线相连,就得到符合
要求的联网.
下面看看最多连多少条线.

在这 80 个点(80 台计算机)中,设从 A1 引出的线最多,有 k 条,与 A1 相连的点是 B1 , B2 ,…, Bk

由于条件, B1 , B2 …, Bk 之间没有线相连.

设与 A1 不相连的点是 A2 , A3 …, Am ,则 m+k=80,而 A2 , A3 …, Am 每一点至多引出 k 条线,

图中至多有 mk 条线,因为 B40 4 × m × k = (m + k ) 2 ≤ (m + k ) 2 = 6400

所以 m×k≤1600,即连线不超过 1600 条.

另一方面,设 80 个点分为两组: A1 , A2 …, A40 ; B1 , B2 …, B40 第一组的每一点与第二组的

每一点各用一条线相连,这样的图符合题目要求,共有 40×40=1600 条线.


10.在一个 6×6 的方格棋盘中,将若干个 1×1 的小方格染成红色.如果随意划掉 3 行 3 列,在剩下
的小方格中必定有一个是红色的.那么最少要涂多少个方格?
【分析与解】方法一:显然,我们先在每行、每列均涂一个方格,使之成为红色,如图 A 所示,但是
在图 B 中,划去 3 行 3 列后,剩下的方格没有红色的,于是再将两个方格涂成红色(依据对称性,应将
2 个方格同时涂成红色),如图 C 所示,但是图 D 的划法,又使剩下的方格没有红色,于是再将两个方
格涂成红色(还是由于对称的缘故,将 2 个方格涂成红色),得到图 E,图 E 不管怎么划去 3 行 3 列,
都能使剩下的方格含有红色的.
这时共涂了 10 个方格.

学而思奥数网 www.aoshu.cn Page 135 of 137


www.baoshi2011.com

方法二:一方面,图 F 表明无论去掉哪三行哪三列总会留下一个涂红的方格.

另一方面,如果只涂 9 个红色方格,那么红格最多的三行至少有 6 个红格(否则第三多的行只有 1


个红格,红格总数≤5+3=8),去掉这三行至多还剩 3 个红格,再去掉三列即可将这三个红格也去掉.
综上所述,至少需要将 10 个方格涂成红色.
12. 证明:在 6×6×6 的正方体盒子中最多可放入 52 个 1×l×4 的小长方体,这里每个小长方体的面
都要与盒子的侧面平行.
【分析与解】 先将 6× 6×6 的正方体盒子视为实体,那么 6×6×6 的正方体可分成 216 个小正方体,
这 216 个小正方体可以组成 27 个棱长为 2 的正方体.我们将这 27 个棱长为 2 的正方体按黑白相间
染色,如下图所示.

其中有 14 个黑色的,13 个白色的,而一个白色的 2×2×2 的正方体可以对应的放人 4 个每个面都


与盒子侧面平行的 1×l×4 的小长方体,所以最多可以放入 13×4=52 个 1×1×4 的小长方体.
评注:6×6×6 的正方体的体积为 216,1×1×4 的小长方体的体积为 4,所以可放入的小正方体数目不超过
216÷4=54 个.
14.用若干个 l×6 和 1×7 的小长方形既不重叠,也不留孔隙地拼成一个 11×12 的大长方形,最少
要用小长方形多少个?
【分析与解】我们先通过面积计算出最优情况:
11×12=132,设用 1×6 的小长方形 x 个,用 1×7 的小长方形 y 个,有 6 x + 7 x = 132 .

学而思奥数网 www.aoshu.cn Page 136 of 137


www.baoshi2011.com

 x = 1 + 7t
解得:  (t 为可取 0 的自然数),共需 x+y=19+t 个小长方形.
 y = 18 − 6t
(1)当 t=0 时,即 x+y=1+18=19,表示其中的 1×6 的小长方形只有 1 个,剩下的 18 个小长方
形都是
l×7 的.
大长方形中无论是 1 行还是 1 列,最多都只能存在 1 个 l×7 的小长方形,所以在大长方形中最多只
能无重叠的同时存在 16 个 l×7 的小长方形.
现在却存在 18 个 1×7 的小长方形,显然不满足;
(2)当 t=l 时,即 x+y=8+12=20,有如下分割满足,所以最少要用小长方形 20 个.

学而思奥数网 www.aoshu.cn Page 137 of 137

You might also like